STUDY-MODE - Practice Exam 1 Flashcards

1
Q

As described in the DSM-5, _______________ is the likely diagnosis for an adult client who has experienced a depressed mood, a poor appetite, and impaired concentration for most of the day on most days for over two years.
Select one:

A. Dysthymic Disorder
B. Mood Disorder NOS
C. Persistent Depressive Disorder
D. Unspecified Depressive Disorder

A

Answer C is correct: The client’s symptoms are consistent with a diagnosis of Persistent Depressive Disorder.

Answer A: Dysthymic Disorder is not a DSM-5 diagnosis.

Answer B: Mood Disorder NOS is not a DSM-5 diagnostic category.

Answer D: Unspecified Depressive Disorder applies when a clinician does not want to indicate the reason why a client’s symptoms do not meet the criteria for a specific depressive disorder.

The correct answer is: Persistent Depressive Disorder

How well did you know this?
1
Not at all
2
3
4
5
Perfectly
2
Q
Your new client, a college freshman, says she sometimes has an "out of body" experience in which she is watching what she is doing from outside herself. She describes a recent experience in which she was in her dorm room writing a paper when she realized she couldn't feel her fingers on the computer keyboard or her feet on the floor and then suddenly felt like she was watching herself from the ceiling. The client says that these episodes make her feel like she's "going crazy" and are interfering with her ability to attend class and complete course assignments. The client's symptoms are most suggestive of a DSM-5 diagnosis of:
Select one:

A. Delusional Disorder
B. Dissociative Fugue
C. Depersonalization/Derealization Disorder
D. Depersonalization Disorder

A

Answer C is correct: Depersonalization/Derealization Disorder is characterized by recurrent episodes of depersonalization (a sense of unreality, detachment, or being an outside observer of one’s thoughts, feelings, etc.) and/or derealization (a sense of unreality or detachment involving one’s surroundings) that cause clinically significant distress or impaired functioning.

Answer A: Delusional Disorder involves one or more delusions that last at least one month. As defined in the DSM-5, a delusion is “a false belief based on incorrect inference about external reality that is firmly held despite what almost everyone else believes and despite what constitutes incontrovertible and obvious proof or evidence to the contrary” (p. 819).

Answer B: A dissociative fugue is characterized by apparently purposeful travel with an inability to recall some or all of one’s past and is included in the DSM-5 as a specifier for Dissociative Amnesia.

Answer D: Depersonalization Disorder is not a DSM-5 diagnosis.

The correct answer is: Depersonalization/Derealization Disorder

How well did you know this?
1
Not at all
2
3
4
5
Perfectly
3
Q

The most likely DSM-5 diagnosis for a person who experiences genital pain during sexual intercourse that causes significant distress and cannot be better explained by a non-sexual mental disorder, relationship distress, or a medical condition is which of the following?
Select one:

A. Dyspareunia
B. Vaginismus
C. Genito-Pelvic Pain/Penetration Disorder
D. Other Specified Sexual Dysfunction

A

Answer C is correct: The symptoms listed in this questions are consistent with the DSM-5 diagnosis of Genito-Pelvic Pain/Penetration Disorder.

The correct answer is: Genito-Pelvic Pain/Penetration Disorder

How well did you know this?
1
Not at all
2
3
4
5
Perfectly
4
Q

Older adults are at a particularly high risk for ________, with the risk being increased by medical illness and surgery.

Select one:
A. depersonalization
B. derealization
C. dementia
D. delirium
A

Advancing age is one of the factors that increases the risk for delirium.

a. Incorrect See explanation for Answer D.
b. Incorrect See explanation for Answer D.
c. Incorrect See explanation for Answer D.
d. CORRECT Several factors place people at higher risk for delirium. People over 60 are usually cited as the group at highest risk in general. Moreover, the risk for this population increases as the result of a medical illness or surgery. Other high risk factors include cardiotomy, burns, and rapid withdrawal from alcohol or a sedative, hypnotic, or anxiolytic.

The correct answer is: delirium

How well did you know this?
1
Not at all
2
3
4
5
Perfectly
5
Q

A person with seasonal affective disorder (SAD) is most likely to exhibit which of the following symptoms?

Select one:

a. overeating and weight gain
b. loss of appetite without weight loss
c. insomnia
d. a craving for salty foods

A

Your answer is incorrect.
Common symptoms of SAD are listed in the Abnormal Psychology chapter of the written study materials.

a. CORRECT - People with SAD usually experience hypersomnia, increased appetite and weight gain, and a craving for carbohydrates.
b. Incorrect - This is not characteristic of SAD.
c. Incorrect - Hypersomnia (not insomnia) is characteristic of SAD.
d. Incorrect - People with SAD often experience a craving for carbohydrates.

The correct answer is: overeating and weight gain

How well did you know this?
1
Not at all
2
3
4
5
Perfectly
6
Q

Restlessness, psychomotor agitation, flushed face, diuresis, rambling speech, and muscle twitching are most suggestive of which of the following?
Select one:

A. Alcohol Withdrawal
B. Caffeine Intoxication
C. Cocaine Intoxication
D. Hyperthyroidism

A

It is sometimes difficult to distinguish between substance-induced disorders and medical conditions.

a. Incorrect Alcohol Withdrawal is characterized by tremor, nausea and vomiting, autonomic hyperactivity, depressed mood and irritability, transient illusions and hallucinations, and insomnia.
b. CORRECT The symptoms listed in the question are characteristic of Caffeine Intoxication.
c. Incorrect Cocaine Intoxication is marked by euphoria, grandiosity, hypervigilance, impaired judgment, rambling and incoherent speech, perspiration, chills, and visual or tactile hallucinations.

d. Incorrect Hyperthyroidism, an endocrine disorder, produces weight loss, increased appetite, intolerance to heat, tremors, and rapid heart rate.
The correct answer is: Caffeine Intoxication

How well did you know this?
1
Not at all
2
3
4
5
Perfectly
7
Q

Research investigating high-risk situations associated with relapse for a variety of addictive and other problem behaviors has shown that which of the following accounts for nearly three-fourths of relapse incidents?
Select one:

A. urges and temptations
B. negative and positive emotional states and negative physical states
C. social pressure, interpersonal conflicts, and negative physical states
D. negative emotional states, interpersonal conflicts, and social pressure

A

Of the various precipitants of relapse, negative emotional states have been found to be the most frequent cause.

a. Incorrect According to Marlatt and Gordon (1985), urges and temptations account for about 11% of relapses among alcoholics (and for similar rates for other types of addictions).
b. Incorrect Although negative emotional states are important determinants of relapse, positive emotional states and negative physical states are less predictive than interpersonal conflicts and social pressure (response d).
c. Incorrect This response can be eliminated because it doesn’t include negative emotional states.
d. CORRECT Marlatt and Gordon report that, for alcoholics, 38% of incidents of relapse are related to negative emotional states; 18% to interpersonal conflicts; and 18% to social pressure. Together, these three account for nearly three-fourths of relapse incidents.

The correct answer is: negative emotional states, interpersonal conflicts, and social pressure

How well did you know this?
1
Not at all
2
3
4
5
Perfectly
8
Q
John Conger (1956) offered the tension-reduction model of alcohol consumption as an explanation for:
Select one:

A. satiation.
B. habituation.
C. addiction.
D. tolerance.

A

J. Conger’s model provides an explanation for alcohol addiction (Reinforcement theory and the dynamics of alcoholism, Quarterly Journal of Studies on Alcohol, 17, 296-305, 1956).

a. Incorrect See explanation for response c.
b. Incorrect See explanation for response c.
c. CORRECT Conger’s theory proposes that drinking is reinforced by its tension reduction properties (i.e., drinking recurs because it removes anxiety and stress) and that this effect eventually leads to addiction.
d. Incorrect See explanation for response c.

The correct answer is: addiction.

How well did you know this?
1
Not at all
2
3
4
5
Perfectly
9
Q

A woman and her husband, who have been married for two years, come for marital therapy. The woman complains about her husband’s lack of interest in family activities and in their 9-month-old child, his lack of affection toward her, and his lack of interest in sex. The husband, who says he did not want to come to therapy at all, states that he is a “loner,” that he has always had few friends, and that he has never really been interested in his family. He seems quite aloof and emotionally unresponsive. Your tentative diagnosis is:
Select one:

a. Avoidant Personality Disorder.
b. Schizoid Personality Disorder.
c. Schizoaffective Disorder.
d. Schizotypal Personality Disorder.

A

Your answer is correct.
The husband’s symptoms appear to be long-term and experienced by him as ego-syntonic, thereby suggesting a personality disorder.

a. Incorrect - An individual with Avoidant Personality Disorder desires close interpersonal relationships but avoids them in order to avoid being hurt.
b. CORRECT - The husband’s symptoms are most suggestive of Schizoid Personality Disorder which is characterized by detachment from social relationships and a restricted range of emotional expression.
c. Incorrect - Schizoaffective Disorder involves a combination of schizophrenic and affective symptoms. This man does not exhibit the psychotic symptoms (hallucinations and delusions) that must be present for this diagnosis.
d. Incorrect - The husband does not seem to have the cognitive or perceptual distortions and eccentricities that are characteristic of Schizotypal Personality Disorder.

The correct answer is: Schizoid Personality Disorder.

How well did you know this?
1
Not at all
2
3
4
5
Perfectly
10
Q

A young woman has just received a diagnosis of Schizophrenia. Which of the following family members is most likely to receive the same diagnosis?
Select one:

a. her father
b. her grandmother
c. her identical twin
d. her biological brother

A

Your answer is correct.
The research has consistently confirmed a genetic contribution to Schizophrenia.

a. Incorrect - See explanation for response c.
b. Incorrect - See explanation for response c.
c. CORRECT - The closer the genetic similarity, the higher the concordance rate for Schizophrenia. The woman’s identical twin is most similar to her in terms of genetic make-up and, therefore, most likely to also receive the diagnosis.

d. Incorrect - See explanation for response c.
The correct answer is: her identical twin

How well did you know this?
1
Not at all
2
3
4
5
Perfectly
11
Q

__________ is a cause of delirium and may result from kidney or liver failure or from diabetes mellitus, hypo- or hyperthyroidism, vitamin deficiency, electrolyte imbalance, severe dehydration, or a number of other conditions.
Select one:

a. Gerstmann’s syndrome
b. Cushing’s syndrome
c. Wernicke’s encephalopathy
d. Metabolic encephalopathy

A

Your answer is incorrect.
If you are unfamiliar with the disorder being asked about by this question, you may have been able to identify the correct answer through the process of elimination.

a. Incorrect - Gerstmann’s syndrome is caused by damage to certain areas of the parietal lobe and involves a combination of finger agnosia, right-left confusion, agraphia, and acalculia.
b. Incorrect - Cushing’s syndrome is caused by excessively high levels of cortisol resulting from the use of glucocorticoids or an abnormality in the pituitary or adrenal gland.
c. Incorrect - Wernicke’s encephalopathy is caused by lesions in certain areas of the brain~ especially the thalamus and mammillary bodies~ often as the result of a thiamine deficiency.
d. CORRECT Metabolic encephalopathy is a state of global brain dysfunction. Its symptoms range from impairments in executive functioning to agitated delirium to coma.

The correct answer is: Metabolic encephalopathy

How well did you know this?
1
Not at all
2
3
4
5
Perfectly
12
Q

The symptoms of Oppositional Defiant Disorder are categorized in three groups in the DSM-5. These groups are:
Select one:

A. destruction of property, deceitfulness or theft, and serious violations of rules.
B. negativistic, defiant, and hostile behavior.
C. deceitfulness/dishonesty, irritability/aggressiveness, and failure to conform to social norms.
D. angry/irritable mood, argumentative/defiant behavior, and vindictiveness.

A

Answer D is correct: Angry/irritable mood, argumentative/defiant behavior, and vindictiveness are the three categories of symptoms included in the DSM-5 for Oppositional Defiant Disorder.

Answer A: These are three of the four categories of symptoms included in the DSM-5 for Conduct Disorder (the fourth is aggression to people and animals).

Answers B and C: See explanation for answer D.

The correct answer is: angry/irritable mood, argumentative/defiant behavior, and vindictiveness.

How well did you know this?
1
Not at all
2
3
4
5
Perfectly
13
Q

Alcohol Withdrawal is characterized by:
Select one:

a. dysphoric mood, vivid dreams, insomnia or hypersomnia, and increased appetite.
b. hand tremor, insomnia, hallucinations, and seizures.
c. incoordination, nystagmus, impaired memory, and mood lability.
d. dysphoric mood, pupillary dilation, insomnia, and fever.

A

Your answer is incorrect.

Answer B is correct: Alcohol Withdrawal is diagnosed in the presence of at least two characteristic symptoms within several hours to a few days following cessation or reduction of alcohol consumption, e.g., hand tremor; insomnia; transient illusions or hallucinations; and generalized tonic-clonic seizures.

Answer A: These symptoms are characteristic of Stimulant Withdrawal.

Answer C: These are symptoms of Alcohol Intoxication.

Answer D: These symptoms are associated with Opioid Withdrawal.

The correct answer is: hand tremor, insomnia, hallucinations, and seizures.

How well did you know this?
1
Not at all
2
3
4
5
Perfectly
14
Q

Which of the following suggests that a DSM-5 diagnosis of Anorexia Nervosa is more appropriate than a diagnosis of Bulimia Nervosa?
Select one:

A. the presence of episodes of binging and purging
B. a disturbance in body image
C. menstrual irregularities
D. persistent restriction of energy intake

A

Answer D is correct: This question is asking which symptom is characteristic of Anorexia but not Bulimia. One of the DSM-5 diagnostic criteria for Anorexia (but not Bulimia) is a “restriction of energy intake relative to requirements, leading to a significantly low body weight” (p. 338).

Answer A: While binging and purging are classic signs of Bulimia Nervosa, some individuals with Anorexia engage in binging and purging, and the DSM-5 includes binge-eating/purging type as a specifier for Anorexia.

Answer B: Anorexia Nervosa and Bulimia Nervosa both involve a disturbance in the perception of one’s body weight or shape.

Answer C: Menstrual irregularity is an associated feature of both disorders.

The correct answer is: persistent restriction of energy intake

How well did you know this?
1
Not at all
2
3
4
5
Perfectly
15
Q

The majority of older adults who complete suicide have a psychiatric disorder at the time of their death, with __________ being the most common diagnosis.
Select one:

A. Major Depressive Disorder
B. Substance Use Disorder
C. Major Neurocognitive Disorder
D. Generalized Anxiety Disorder

A

Answer A is correct. The research has been fairly consistent in identifying Major Depressive Disorder as a significant risk factor for suicide among older adults. In their review of psychological autopsy studies of older adults who completed suicide, Y. Conwell, P. R. Duberstein, and E. D. Caine report that Major Depressive Disorder was identified as the most common psychiatric diagnosis [Risk factors for suicide in later life, Biological Psychiatry, 52, 193-204, 2002]. While Substance Use Disorders, Major Neurocognitive Disorder, and Anxiety Disorders were also linked to suicide in these studies, they were present in a smaller number of cases.

The correct answer is: Major Depressive Disorder

How well did you know this?
1
Not at all
2
3
4
5
Perfectly
16
Q

During the postpartum period, approximately _____ of women experience postpartum depression.
Select one:

A. 1 to 2%
B. 10 to 15%
C. 25 to 30%
D. 45 to 50%

A

Postpartum depression is a mood disorder that begins within several weeks following delivery.

a. Incorrect See explanation for response b.
b. CORRECT Ten to 15% is the rate most often reported in the literature for postpartum depression in the United States. Factors associated with an increased risk for postpartum depression include having difficulty getting pregnant, having twins or triplets, having premature labor and delivery, experiencing pregnancy and birth complications, and having a baby during adolescence. See, e.g., CDC, Prevalence of self-reported postpartum depression - 17 states, 2004-2005, Morbidity and Mortality Weekly Report, 57(14), 361-366, 2008.
c. Incorrect See explanation for response b.

d. Incorrect See explanation for response b.
The correct answer is: 10 to 15%

How well did you know this?
1
Not at all
2
3
4
5
Perfectly
17
Q

Maxine M. states she believes that messages on a particular radio station are being broadcast especially to her. Maxine is displaying which of the following?
Select one:

A. illusion
B. hallucination
C. delusion
D. blocking

A

All of the phenomena listed in the answers are characteristic of Schizophrenia and other disorders, but only one describes the woman’s symptom.

a. Incorrect An illusion is a distorted perception or memory.
b. Incorrect An hallucination is a false perception. A person is hallucinating, for example, if she hears voices that are not really there.
c. CORRECT A delusion is a false belief that persists despite evidence to the contrary. The woman’s delusion (her interpretation of an external event as though it has special significance for her) is a “delusion of reference.”
d. Incorrect Blocking is an abrupt, involuntary interruption in the flow of speech or thought.

The correct answer is: delusion

How well did you know this?
1
Not at all
2
3
4
5
Perfectly
18
Q

A 35-year old client has a history of relationship problems. Although he usually makes a good first impression, his friendships don’t last very long. He attributes this problem to the fact that other people are jealous of what he has accomplished. The client has a great idea that will “revolutionize the field of telemarketing,” but, so far, no one has recognized the potential importance of his contribution. He left his wife six months ago and spends very little time with his two-year old son. He has been busy “doing what he wants to do,” which has included moving into an expensive condominium, buying a new sports car, and dating as many attractive women as possible. These characteristics are most suggestive of:

Select one:

a. Histrionic Personality Disorder.
b. Narcissistic Personality Disorder.
c. Schizotypal Personality Disorder.
d. Delusional Disorder, grandiose type.

A

Your answer is correct.
The man’s primary symptoms are grandiosity and lack of empathy.

a. Incorrect - A diagnosis of Histrionic Personality Disorder requires a pattern of excessive emotionality and attention-seeking.
b. CORRECT - These are core features of Narcissistic Personality Disorder.
c. Incorrect - A diagnosis of Schizotypal Personality Disorder requires the presence of pervasive social and interpersonal deficits and eccentricities in cognition, perception, and behavior.
d. Incorrect - A diagnosis of Delusional Disorder is made in the presence of one or more non bizarre delusions with unimpaired functioning apart from the impact of the delusions.

The correct answer is: Narcissistic Personality Disorder.

How well did you know this?
1
Not at all
2
3
4
5
Perfectly
19
Q

Linehan’s (1993) dialectical behavior therapy (DBT) for borderline personality disorder is based on the assumption that this disorder is due to:
Select one:

A. a boundary disturbance.
B. learned helplessness.
C. emotion dysregulation.
D. pathologic internalized object relations.

A

A key assumption underlying DBT is that the central problem in borderline personality disorder (BPD) is a dysfunctional emotion regulation system, which may be the result of genetic factors, events that occurred during prenatal development, and/or early trauma that adversely affected the brain.

a. Incorrect Boundary disturbances are viewed as the cause of neurotic behavior by Gestalt therapists.
b. Incorrect Learned helplessness has been linked to depression.
c. CORRECT Linehan has described BPD as resulting from emotion dysregulation, which is viewed as due to a combination of a biological predisposition to react to even low levels of stress and other forms of provocation with intense and long-lasting emotions and an emotion-invalidating environment. According to Linehan, most of the behaviors associated with BPD are attempts to regulate intense emotions or the undesirable outcomes of emotion dysregulation.

d. Incorrect Kernberg identified pathologic internalized object relations as the etiological factor in BPD.
The correct answer is: emotion dysregulation.

How well did you know this?
1
Not at all
2
3
4
5
Perfectly
20
Q

Cataplexy is often triggered by which of the following?
Select one:

A. smells associated with a traumatic event
B. physical fatigue
C. strong emotions
D. stimuli associated with a feared object or event

A

Cataplexy is a brief episode of a sudden loss of muscle tone that is associated with narcolepsy. a. Incorrect See explanation for response c.

b. Incorrect See explanation for response c.
c. CORRECT Cataplexy occurs in the majority of individuals who have narcolepsy and is triggered by strong emotions (most often positive emotions such as joking and laughing but also by anger).

d. Incorrect See explanation for response b.
The correct answer is: strong emotions

How well did you know this?
1
Not at all
2
3
4
5
Perfectly
21
Q

Meta-analyses of research on the treatment of pediatric acute lymphoblastic leukemia with cranial radiation or chemotherapy have found that:
Select one:

A. either treatment alone is associated with decreased intellectual functioning and lower academic achievement.
B. only cranial radiation is associated with decreased intellectual functioning and lower academic achievement.
C. only chemotherapy is associated with decreased intellectual functioning and lower academic achievement.
D. they are associated with decreased intellectual functioning and lower academic achievement only when used in combination.

A

A number of studies have confirmed that radiation and chemotherapy improve survival rates but are associated with a number of negative consequences.

a. CORRECT Either treatment alone has substantial negative effects on intellectual functioning and academic achievement, although the effects vary from individual to individual and may be less severe for chemotherapy. In addition, the negative effects are largely irreversible and may worsen over time. See, e.g., C. C. Peterson et al., A meta-analysis of the neuropsychological sequelae of chemotherapy-only treatment for pediatric acute lymphoblastic leukemia, Pediatric Blood Cancer, 51, 99-104, 2008.
b. Incorrect See explanation above.
c. Incorrect See explanation above.
d. Incorrect See explanation above.

The correct answer is: either treatment alone is associated with decreased intellectual functioning and lower academic achievement.

How well did you know this?
1
Not at all
2
3
4
5
Perfectly
22
Q

Which of the following has been found to be the strongest predictor of Intellectual Disability with an unknown etiology?
Select one:

A. paternal health
B. female gender
C. large family size
D. low birth weight

A

Answer D is correct: Croen et al. (2001) investigated individual and family characteristics of children with a mild or severe Intellectual Disability with an unknown etiology and found low birth weight to be the strongest predictor of both levels of disability.

Answers A, B, and C: See explanation for answer D.

The correct answer is: low birth weight

How well did you know this?
1
Not at all
2
3
4
5
Perfectly
23
Q

Based on the initial evaluation of a 15-year-old boy, a clinician believes the nature and number of his symptoms suggest a DSM-5 diagnosis of Attention-Deficit/Hyperactivity Disorder, predominantly inattentive presentation. Before assigning the diagnosis, the clinician will want to confirm that some symptoms were present before the boy was _____ years of age and that he exhibits them in at least _____ settings.
Select one:

A. 7; 2
B. 12; 2
C. 6; 3
D. 10; 3

A

Answer B is correct: For a diagnosis of ADHD, the DSM-5 requires that “several inattentive or hyperactive-impulsive symptoms were present prior to age 12” (p. 60) and that symptoms are present in at least two settings.

The correct answer is: 12; 2

How well did you know this?
1
Not at all
2
3
4
5
Perfectly
24
Q

A dimensional approach to diagnosis is based on a(n) ________ of attributes.

Select one:

a. quantification
b. categorization
c. exclusion
d. dichotomization

A

There are two basic approaches to diagnosis - categorical classification and dimensional classification. The DSM utilizes a categorical approach, which involves determining whether or not a person meets the criteria for a given diagnosis.

a. CORRECT - When using a dimensional approach~ the individual is ranked on a quantitative dimension (e.g., on a scale from 1 to 10) on each symptom or other characteristic.
b. Incorrect - See explanation for response a.
c. Incorrect - See explanation for response a.

d. Incorrect - See explanation for response a.
The correct answer is: quantification

How well did you know this?
1
Not at all
2
3
4
5
Perfectly
25
Q

As described in the DSM-5, which of the following are symptoms of Stimulant Intoxication?
Select one:

A. nystagmus, numbness, and muscle rigidity
B. blurred vision, tremor, and depressed reflexes
C. pupillary dilation, nausea, and muscular weakness
D. fatigue, increased appetite, and vivid dreams

A

Answer C is correct: Symptoms of Stimulant Intoxication include tachycardia or bradycardia, pupillary dilation, elevated or lowered blood pressure, perspiration or chills, nausea or vomiting, weight loss, psychomotor agitation or retardation, muscular weakness, respiratory depression or cardiac arrhythmias, confusion, seizures, and coma.

Answer A: These symptoms are characteristic of Phencyclidine Intoxication.

Answer B: These symptoms are characteristic of Inhalant Intoxication.

Answer D: These are symptoms of Stimulant Withdrawal, not Stimulant Intoxication.

The correct answer is: pupillary dilation, nausea, and muscular weakness

How well did you know this?
1
Not at all
2
3
4
5
Perfectly
26
Q

Use of which of the following substances is NOT likely to produce a Substance-Induced Psychotic Disorder?
Select one:

A. alcohol
B. cannabis
C. cocaine
D. opioids

A

Answer D is correct: The DSM-5 identifies 10 classes of drugs that are associated with Substance-Induced Disorders but only eight are associated with Substance-Induced Psychotic Disorder: alcohol; cannabis; phencyclidine; other hallucinogen; inhalant; sedative, hypnotic, or anxiolytic; amphetamine or other stimulant; and cocaine.

Answers A, B, and C: See explanation for answer D.

The correct answer is: opioids

How well did you know this?
1
Not at all
2
3
4
5
Perfectly
27
Q

A 16-year-old suddenly exhibits a change in behavior that includes irritability, concentration problems at school, a decreased need for sleep, and sexual promiscuity. Her mother says that the girl has had behavioral problems at school for the past few years but that her current symptoms are new and developed about ten days ago. Based on this information, the most likely diagnosis is:

Select one:

a. ADHD.
b. Conduct Disorder.
c. Bipolar I Disorder.
d. Bipolar II Disorder.

A

The girl’s symptoms and their sudden onset are most suggestive of Bipolar Disorder.

a. Incorrect - For a diagnosis of ADHD, there would have to be some evidence of hyperactivity-impulsivity and/or inattention in at least two settings.
b. Incorrect - A diagnosis of Conduct Disorder requires a persistent pattern of antisocial behavior. Although the question mentions that the girl has had “behavioral problems,” there is not sufficient evidence to conclude that they meet the criteria for this diagnosis.
c. CORRECT - The girl’s symptoms are suggestive of mania which, in adolescents, is sometimes preceded by a history of behavioral problems.
d. Incorrect - A diagnosis of Bipolar II disorder requires a history of both major depressive and hypomanic episodes.

The correct answer is: Bipolar I Disorder.

How well did you know this?
1
Not at all
2
3
4
5
Perfectly
28
Q

A 12-year-old boy’s mother says her son is very argumentative, doesn’t do what he is asked to do, and frequently bullies and threatens his sister and classmates at school. Also, in the past year, the boy has skipped school at least a half dozen times, ran away from home for two days on three different occasions, and was expelled from school for throwing a rock through a school window. The mother has been divorced for four years, and she says that these problems started several months after she remarried two years ago. Based on these symptoms, the most likely diagnosis for the boy is:
Select one:

a. Child or Adolescent Antisocial Behavior.
b. Adjustment Disorder with disturbance of conduct.
c. Oppositional Defiant Disorder.
d. Conduct Disorder.

A

Answer D is correct: The diagnosis of Conduct Disorder requires a persistent pattern of behavior that violates the basic rights of others and/or age-appropriate social norms or rules with three or more characteristic symptoms, e.g. bullying, threatening, or intimidating others; being frequently truant from school; running away from home at least twice; and vandalizing school property.

Answer A: Child or Adolescent Antisocial Behavior is included in the DSM-5 with Other Conditions That May Be a Focus of Clinical Attention and may apply when the individual’s symptoms do not meet the criteria for a mental disorder.

Answer B: A diagnosis of Adjustment Disorder is assigned when the individual does not meet the diagnostic criteria for another mental disorder.

Answer C: Oppositional Defiant Disorder involves a recurrent pattern of an angry/irritable mood, argumentative/defiant behavior, or vindictiveness. The boy’s symptoms are more severe than those associated with Oppositional Defiant Disorder.

The correct answer is: Conduct Disorder.

How well did you know this?
1
Not at all
2
3
4
5
Perfectly
29
Q

The DSM-5 requires which of the following for a diagnosis of Autism Spectrum Disorder?
Select one:

A. an onset of symptoms prior to three years of age
B. an onset of symptoms during the early developmental period
C. apparent normal development for at least six months followed by a loss of previously acquired skills
D. apparent normal development for at least two years followed by a loss of previously acquired skills

A

Answer B is correct: For the diagnosis of Autism Spectrum Disorder, the DSM-5 requires that “symptoms must be present in the early developmental period” (p. 50).

Answers A, C, and D: See explanation above.

The correct answer is: an onset of symptoms during the early developmental period

How well did you know this?
1
Not at all
2
3
4
5
Perfectly
30
Q

Renaldo, age 20, is afraid he’s going to be fired from his job as an inventory stocker at a large warehouse because he frequently “loses track” of inventory and does not follow-through on his boss’s instructions. He says he’s always been forgetful and never did well in school because he had a hard time focusing on his schoolwork. Based on this information, the best DSM-5 diagnosis for Renaldo is which of the following?
Select one:

A. Attention-Deficit/Hyperactivity Disorder, combined presentation
B. Attention-Deficit/Hyperactivity Disorder, in partial remission
C. Other Specified Attention-Deficit/Hyperactivity Disorder, with insufficient inattention symptoms
D. Attention-Deficit/Hyperactivity Disorder, predominantly hyperactive/impulsive presentation

A

Answer C is correct: Of the diagnoses listed in the answers, Other Specified Attention-Deficit/Hyperactivity Disorder is the best since Renaldo’s symptoms involve problems related to attention but it’s unclear if he has the minimum number of symptoms (at least six for children and five for individuals ages 17 and older) required for a diagnosis of Attention-Deficit/Hyperactivity Disorder, predominantly inattentive presentation.

Answer A: The information provided in the question does not indicate that Renaldo has symptoms of hyperactivity or impulsivity, so Attention-Deficit/Hyperactivity Disorder, combined presentation can be eliminated as the correct answer.

Answer B: There is no information provided in the question to suggest that Renaldo met the full diagnostic criteria for ADHD in the past and it’s not clear how many symptoms he has now, so it is not possible to conclude that he is currently in remission.

Answer D: See explanation for answers A and C.

The correct answer is: Other Specified Attention-Deficit/Hyperactivity Disorder, with insufficient inattention symptoms

How well did you know this?
1
Not at all
2
3
4
5
Perfectly
31
Q

In their study of outpatient mental health services in Los Angeles County, Sue et al. (1991) found that, while clients from all groups showed improvement at the end of therapy, __________ had the best outcomes.
Select one:

A. African Americans
B. Anglo Americans
C. Asian Americans
D. Hispanic Americans

A

Answer D is correct. Sue et al. (1991) and others have found a few consistent differences in outcomes for members of different racial/ethnic groups. Sue et al. found that Hispanic Americans had the best outcomes, followed by Anglos, Asian Americans, and, lastly, African Americans.

The correct answer is: Hispanic Americans

  1. Hispanic
  2. Anglos
  3. Asian
  4. African American
How well did you know this?
1
Not at all
2
3
4
5
Perfectly
32
Q

Jung’s theory differs from Freud’s in all of the following ways except:
Select one:

A. Jung’s perspective reflects a more optimistic view of human nature.
B. Jung promoted a more active interchange between analyst and patient.
C. Jung conceptualized the psyche as continuing to evolve throughout the lifespan.
D. Jung replaced the concept of the ego with the concepts of the self and the ideal-self.

A

Jung’s analytic psychology differs from Freud’s psychotherapy in a number of important ways. For Jung, for example, the basic elements of the psyche are the conscious (which includes the ego) and the unconscious (which includes a personal and collective component).

a. Incorrect This is true about Jung’s perspective. He described people as being more creative and goal-directed than Freud.
b. Incorrect This also describes Jung’s approach.
c. Incorrect Jung placed a great deal of emphasis on changes that occur during adulthood, especially in midlife.
d. CORRECT The ego is an important part of Jung’s theory, and, although he did make use of the notion of the “self,” he considered the self to be the archetype for the ego.

The correct answer is: Jung replaced the concept of the ego with the concepts of the self and the ideal-self.

How well did you know this?
1
Not at all
2
3
4
5
Perfectly
33
Q

According to Atkinson, Morten, and Sue’s (1993) Racial/Cultural Identity Development Model, the final stage of identity development is characterized by:
Select one:

A. assimilating into the dominant culture.
B. developing a self-identity that is consistent with one’s racial/cultural background.
C. adopting an identity that is independent of the minority and majority cultures.
D. adopting a multicultural perspective.

A

Atkinson et al.’s (1993) model distinguishes between five stages:

CONFORMITY
DISSONANCE
RESISTANCE-IMMERSION 
INTROSPECTION
INTEGRATIVE AWARENESS 

a. Incorrect Atkinson et al. do not view assimilation as the outcome of successful identity development.
b. Incorrect Although successful identity development does involve developing an ethnic identity, that identity is not necessarily reflective of the individual’s cultural/racial background in all ways. Instead, the individual chooses which elements he/she wants to incorporate into his/her identity.
c. Incorrect Atkinson’s model does not propose that being “culture-free” is an optimal state.
d. CORRECT Like most cultural/racial identity development models, the Atkinson et al. model describes the final stage as one of integration - i.e., of accepting one’s own minority culture while also recognizing and incorporating the positive contributions of the majority culture. They refer to this as adopting a multicultural perspective.

The correct answer is: adopting a multicultural perspective.

How well did you know this?
1
Not at all
2
3
4
5
Perfectly
34
Q

A husband and wife avoid conflicts with one another by focusing their attention on helping their child overcome his difficulties at school. As described by Minuchin, this is an example of:
Select one:

A. reframing.
B. detouring.
C. stable coalition.
D. triangulation.

A

Minuchin distinguished between three types of rigid triads that help alleviate tension between two family members -

DETOURING
STABLE COALITION
TRIANGULATION

a. Incorrect Reframing is a paradoxical technique that involves providing an alternative meaning for a behavior so that it can be seen from a new perspective.
b. CORRECT Detouring occurs when parents avoid the tension between them by either blaming or overprotecting a child.
c. Incorrect A stable coalition occurs when a parent and child consistently “gang up” against the other parent.
d. Incorrect Triangulation occurs when both parents attempt to get the child to side with him or her.

The correct answer is: detouring.

How well did you know this?
1
Not at all
2
3
4
5
Perfectly
35
Q

Caplan (1970) distinguished between four types of mental health consultation. Which of the following is an example of the type he labeled “consultee-centered case consultation”?
Select one:

A. A consultant is hired by a colleague to resolve a diagnostic dilemma she is having with a current client.
B. A consultant is hired by a school to help school administrators evaluate the effectiveness of an existing remedial program for at-risk students.
C. A consultant monitors a support group for administrators to help them develop better interpersonal skills.
D. A consultant helps a teacher acquire the behavior modification skills she needs to reduce disruptions in her classroom.

A

As its name implies, “consultee-centered case consultation” targets the consultee.

a. Incorrect This is an example of client-centered case consultation.
b. Incorrect This is an example of program-centered administrative consultation.
c. Incorrect This is an example of consultee-centered administrative consultation.
d. CORRECT In this situation, the consultant is helping the consultee improve her skills so that she is better able to deal with future problems in the classroom. The focus of the consultation is on the consultee rather than on particular individuals who the consultee will be in contact with.

The correct answer is: A consultant helps a teacher acquire the behavior modification skills she needs to reduce disruptions in her classroom.

How well did you know this?
1
Not at all
2
3
4
5
Perfectly
36
Q

Which of the following is likely to be least effective when working with American Indian or Alaskan Native therapy clients?
Select one:

A. promoting a value free environment in therapy
B. adopting a holistic perspective that regards illness as a result of multiple factors
C. incorporating traditional healers and practices into the treatment process
D. adopting a client-centered approach that avoids highly directive techniques

A

Although psychologists must be careful not to stereotype members of specific cultural groups, it is always important to take a client’s cultural background into consideration as part of the treatment planning process.

a. CORRECT Promoting a “value-free” environment is similar to adopting a “color-blind” perspective and is inappropriate when working with members of American Indian and Alaskan Native communities. A more effective approach is to establish credibility and trust by demonstrating familiarity with and respect for the client’s culture.
b. Incorrect This would be an appropriate approach when working with American Indians and Alaskan Natives who tend to view illness holistically - i.e., as being the result of multiple factors, including disharmony with nature.
c. Incorrect Incorporating elders, medicine people, and other traditional healers as well as traditional healing practices and ceremonies into the treatment process is considered an essential component of successful treatments for members of American Indian and Alaskan Native communities.
d. Incorrect A collaborative, problem-solving, client-centered approach is appropriate for members of these communities.

The correct answer is: promoting a value free environment in therapy

How well did you know this?
1
Not at all
2
3
4
5
Perfectly
37
Q

African-American adolescents who exhibit a “blended bicultural” behavioral pattern:
Select one:

A. are situated midway between ethnic and American culture and consider themselves part of neither

B. acknowledge their American heritage but are more strongly influenced by their ethnic background

C. do not distinguish between ethnic and American cultures

D. view themselves as having integrated American and ethnic identities

A

Biculturalism among African-American and Mexican-American adolescents was examined by J. S. Phinney and M. Devich-Navarro (Variations in bicultural identification among African-American and Mexican-American adolescents, Journal of Research in Adolescence, 7(1), 3-32, 1997). These investigators found that the adolescents in their sample exhibited one of three patterns: blended biculturalism, alternating biculturalism, or separated.

a. Incorrect This describes MARGINALISATION. None of the adolescents in Phinney and Devich-Navarro’s sample fit this pattern.
b. Incorrect Phinney and Devich-Navarro refer to this as ALTERNATING BICULTURALISM.
c. Incorrect This is referred to as a FUSED IDENTITY. Phinney and Navarro found no evidence of this pattern in their sample.
d. CORRECT As defined by these authors, this pattern represents blended BICULTURALISM.

The correct answer is: view themselves as having integrated American and ethnic identities.

How well did you know this?
1
Not at all
2
3
4
5
Perfectly
38
Q

Which of the following is a challenge that may arise from mandating Evidence Based Therapies?
Select one:

A. The financial cost of training all of the individuals needed.
B. The limited number of trainers with which to prepare those that are gaining skills in Evidence Based Therapies.
C. The limited number of Evidence Based Trainings.
D. The varied type and intensity of training that differs across regions.

A

The correct answer is D. Many community-based training initiatives in evidence-based psychosocial treatments have acquired significant support due to the evidence that some EBTs can be conveyed effectively into non-research and community-based settings. Although many large health organizations such as the National Health Service in the United Kingdom, the United States Department of Veterans Affairs and state mental health systems have mandated the implementation of EBTs, the intensity and type of training varies considerably across these fields.

Answers A, B and C are incorrect as the main difficulty is the lack of consistency across the field, not the expense, number of trainers or lack of evidenced based therapies.

The correct answer is: The varied type and intensity of training that differs across regions.

How well did you know this?
1
Not at all
2
3
4
5
Perfectly
39
Q

Manuel is a 21-year old college student who moved to the United States from Mexico with his family when he was four years old. Manuel has many Anglo friends at school and usually dates Anglo women, but, at home, he speaks Spanish and participates in the cultural traditions practiced by his family. When asked about his ethnic identity, Manuel says, “I’m American and Hispanic, but I guess I consider myself mostly Hispanic.” Manuel is best described as:
Select one:

A. fused.
B. separated.
C. assimilated.
D. bicultural.

A

Different experts have labeled the different levels or categories of acculturation somewhat differently, but most distinguish between at least four levels -

ASSIMILATED
INTEGRATED (bicultural)
SEPARATED
MARGINAL

a. Incorrect The term “fused” refers to the melting pot position in which the two separate cultures cannot be distinguished.
b. Incorrect Separation occurs when the individual is immersed in his/her own ethnic culture and is not part of the larger (dominant) culture.
c. Incorrect An assimilated individual has given up his/her own ethnic culture and identifies with the larger (dominant) culture.
d. CORRECT The bicultural individual has adopted both cultures. Biculturalism can be alternating (as in Manuel’s case) or blended.

The correct answer is: bicultural.

How well did you know this?
1
Not at all
2
3
4
5
Perfectly
40
Q
For Yalom (1985), in group therapy,\_\_\_\_\_\_\_\_\_\_ is the analogue of the client-psychotherapist relationship in individual therapy and is a key factor in therapy outcome.
Select one:

A. cohesiveness
B. family re-enactment
C. universality
D. identification

A

Answer A is correct. If you think about Yalom’s initial stages in group therapy, you might recall that it is the development of cohesiveness that moves the group forward into the working stage. Although Yalom believes that group therapy provides a number of curative factors, he emphasizes the importance of group cohesiveness and describes it as the analogue to the therapist-client relationship in individual therapy.

The correct answer is: cohesiveness

How well did you know this?
1
Not at all
2
3
4
5
Perfectly
41
Q

You are beginning a therapy group for individuals dealing with mid-life issues. From your experiences with new groups, you know that, during the beginning stages of group therapy:
Select one:

A. group members will talk through the therapist and seemingly ignore one another.
B. group members will interact with one another and more or less ignore the therapist.
C. group members will act quite hostile toward the therapist and resist his/her suggestions.
D. cliques will begin to develop within the group.

A

In the beginning of group therapy, clients rely more on the therapist. Later, as the group members become closer, group cohesion develops and members interact more with one another.

a. CORRECT Initially, group members ignore each other and direct all of their comments and questions to the therapist.
b. Incorrect As response A suggests, interactions between group members will be sparse at the onset of therapy.
c. Incorrect Although this may be true in some cases, most group members perceive the therapist as their ally.
d. Incorrect Cliques begin to develop, if at all, after the beginning stages of group therapy.

The correct answer is: group members will talk through the therapist and seemingly ignore one another.

How well did you know this?
1
Not at all
2
3
4
5
Perfectly
42
Q

Bateson, Jackson, Haley, and Weakland (1956) identified double-bind communication as an etiological factor for which of the following disorders?
Select one:

A. Anorexia
B. Schizophrenia
C. Antisocial Personality Disorder
D. Narcissistic Personality Disorder

A

As described by Bateson et al. (1956), double-bind communication occurs when a family member receives contradictory injunctions (messages) from another family member and is unable to comment on those injunctions or escape from them.

a. Incorrect See explanation for response b.
b. CORRECT The title of Bateson et al.’s 1956 paper was Toward a Theory of Schizophrenia. Their double-bind hypothesis provided an alternative to psychodynamic theory regarding the development of this disorder and proposed that the recipient of double-bind communication learns to escape the discomfort it creates by responding with similarly distorted and contradictory messages. Eventually, the individual becomes unable to understand the true meaning of his/her own messages and those of others.
c. Incorrect See explanation for response b.
d. Incorrect See explanation for response b.

The correct answer is: Schizophrenia

How well did you know this?
1
Not at all
2
3
4
5
Perfectly
43
Q

When treating a couple in which the husband has physically abused his wife and it has been clearly determined that the abuse can be classified as instrumental (vs. expressive), the best initial approach is:
Select one:

A. to ensure that the husband and wife are physically separated and to provide them with separate therapy.
B. to have the husband and wife sign a “no-violence contract” and to provide them with separate therapy.
C. to have the husband and wife sign a “no-violence contract” and to provide them with conjoint therapy.
D. to provide combined individual and group therapy for both the husband and wife.

A

The key to this question is noting that the abuse is instrumental rather then expressive and knowing that therapists often recommend conjoint treatment only in the latter case (see, e.g., R. N. Mack, Spouse Abuse: A Dyadic Approach, in G. R. Weeks, Treating couples, New York: Bruner/Mazel, 1989).

a. CORRECT In cases of instrumental abuse (which is brutal, dangerous, and committed with little provocation), the woman’s physical safety is the priority. Therefore, the best course of action is to make sure the husband and wife are physically separated and provided with separate therapy.
b. Incorrect See explanations for responses a and c.
c. Incorrect This is usually the treatment-of-choice in cases of expressive abuse, in which the abuse is related to the emotional life of the couple and occurs within the context of escalating conflict.
d. Incorrect See explanations for responses a and c.

The correct answer is: to ensure that the husband and wife are physically separated and to provide them with separate therapy.

How well did you know this?
1
Not at all
2
3
4
5
Perfectly
44
Q

In Freudian psychoanalysis, clarification:
Select one:

A. reduces the clients resistance.
B. links current behavior to unconscious processes.
C. brings a confusing or unclear issue into focus.
D. helps the client gain control over his/her inner conflicts.

A

The process of Freudian psychoanalysis is described as involving a combination of:

CONFRONTATION
CLARIFICATION
INTERPRETATION
WORKING THROUGH

a. Incorrect This does not accurately describe the function of clarification.
b. Incorrect This is a better description of interpretation than of clarification.
c. CORRECT Clarification is used to clarify an issue by, for example, separating a key issue from less important ones.
d. Incorrect This is part of the working through process.

The correct answer is: brings a confusing or unclear issue into focus.

How well did you know this?
1
Not at all
2
3
4
5
Perfectly
45
Q

Family therapy is probably contraindicated in which of the following situations?
Select one:

A. The presenting problem involves long-term marital conflict.
B. Family members attribute their problems to one of the members.
C. One of the family members has severe depression.
D. One of the family members has anorexia or bulimia.

A

Family therapy is generally contraindicated when one family member’s disturbance, while affecting the family, is basically unrelated to family processes or structure or is so disruptive that it would interfere with the success of therapy.

a. Incorrect Family therapy would be appropriate in this situation.
b. Incorrect Family therapy would be appropriate in this situation.
c. CORRECT Depression has been linked to a variety of factors including heredity, biochemical abnormalities, learned helplessness, hostility or aggression turned inward, and illogical schemata. It has not, however, been consistently linked with any particular family factor; and family therapy, therefore, would not be particularly useful as a treatment for this disorder.
d. Incorrect Family therapy would be appropriate in this situation since eating disorders (especially anorexia and bulimia) have been linked to family factors.

The correct answer is: One of the family members has severe depression.

How well did you know this?
1
Not at all
2
3
4
5
Perfectly
46
Q

In their meta-analysis of research evaluating psychotherapy outcomes for children and adolescents, Weisz and his colleagues (1995) obtained the largest effect size for:
Select one:

A. female children.
B. male children.
C. female adolescents.
D. male adolescents.

A

The Weisz et al. meta-analysis examined the effects of type of disorder, type of therapy, therapist experience, and child age and gender on psychotherapy outcomes [J. R. Weisz et al., Effects of psychotherapy with children and adolescents revisited: A meta-analysis of treatment outcome studies, Psychological Bulletin, 117(3), 450-468, 1995].

a. Incorrect See explanation for response c.
b. Incorrect See explanation for response c.
c. CORRECT Weisz et al. report an effect size of .86 for female adolescents. In contrast, they report an effect size of .37 for male adolescents, .44 for female children, and .48 for male children.
d. Incorrect See explanation for response c.

The correct answer is: female adolescents.

How well did you know this?
1
Not at all
2
3
4
5
Perfectly
47
Q

A strategic family therapist is working with a family in which the husband and wife have avoided their own conflicts by focusing their attention on the problems of their 9-year old son. The therapist in this situation is most likely to:
Select one:

A. educate the parents about “triangulation.”
B. obtain a detailed family history from the husband and wife to identify the pattern of interactions in their families of origin.
C. give the husband and wife a homework assignment designed to foster recognition of the conflicts that exist between them.
D. use “circular questions” to help family members gain insight into each other’s perspectives.

A

As its name implies, strategic family therapy involves the use of “strategies.”

a. Incorrect This is not characteristic of strategic family therapy.
b. Incorrect This is not characteristic of strategic family therapy.
c. CORRECT Of the responses given, this one is the best example of the techniques used by a strategic family therapist. The therapist is assigning the couple a task that will help them face their own conflicts.
d. Incorrect This is also not typical of strategic family therapy.

The correct answer is: give the husband and wife a homework assignment designed to foster recognition of the conflicts that exist between them.

How well did you know this?
1
Not at all
2
3
4
5
Perfectly
48
Q

Based on her review of the existing models of gay and lesbian identity development, Sophie (1985/86) developed a general stage theory of lesbian identity development, which she tested on several women who were currently experiencing changes with regard to their sexual orientation. Which of the following accurately presents Sophie’s four stages?

Select one:
A. identity confusion, identity comparison, identity acceptance, and identity pride
B. sensitization, identify confusion, identity assumption, and identity integration
C. first awareness, testing and exploration, identity acceptance, and identity integration
D. pre-encounter, encounter, immersion-emersion, and internalization

A

Sophie reviewed several of the prominent stage theories to derive her own general model [A critical examination of stage theories of lesbian identity development, Journal of Homosexuality, 12(2), 39-51, 1985/86].

a. Incorrect These are four of the six stages of Cass’s (1984) stage model of gay and lesbian identity development. In order, the six stages are identity confusion, identity comparison, identity tolerance, identity acceptance, identity pride, and identity synthesis.
b. Incorrect These are the four stages of Troiden’s (1988) gay/lesbian identity development model.
c. CORRECT These are the four stages described by Sophie.
d. Incorrect These are the four stages of Cross’s (1991) Black racial identity development model.

The correct answer is: first awareness, testing and exploration, identity acceptance, and identity integration

How well did you know this?
1
Not at all
2
3
4
5
Perfectly
49
Q

Although it is necessary to avoid stereotypes when providing services to members of ethnic and racial minority groups, some generalizations are often made. For example, when working with Hispanic and Hispanic American clients, it is important to keep in mind all of the following except:
Select one:

A. different levels of acculturation among Hispanics can influence their perceptions of and responses to therapy.
B. family therapy is often contraindicated because of the hierarchical nature of the Hispanic family.
C. behavioral and other active, problem-oriented therapies are usually more acceptable than insight-oriented therapies.
D. sex-roles tend to be demarcated clearly and are fairly rigid.

A

Some generalizations about the treatment of Hispanic and Hispanic American clients have been made in the literature, and you should be familiar with them for the licensing exam.

a. Incorrect This is true. There are different levels of acculturation not only between families but also within families (i.e., younger members of the family tend to be more acculturated to the white middle-class way of life).
b. CORRECT Although Hispanic families do tend to be hierarchical, this does not preclude them from family therapy. In fact, family therapy is often the treatment-of-choice because of the close relationships between family members.
c. Incorrect This is true not only for Hispanics but also members of other minority groups.
d. Incorrect This is also generally true.

The correct answer is: family therapy is often contraindicated because of the hierarchical nature of the Hispanic family.

How well did you know this?
1
Not at all
2
3
4
5
Perfectly
50
Q

Don Jackson, Murray Bowen, and a number of pioneers in the field of family therapy had a psychodynamic background, which they incorporated, to some degree, in their approach to family therapy. However, it is the psychoanalyst _______________ who is generally identified as the first to formally adapt and apply psychoanalytic principles to the understanding of the family.
Select one:

A. Nathan Ackerman
B. Carl Whitaker
C. Richard Stuart
D. Robert Liberman

A

Nathan Ackerman, a psychoanalyst and child psychiatrist, began investigating the role of family dynamics in the 1930s; and, in the 1940s, he started developing clinical techniques that applied psychoanalytic principles to the treatment of preschool children and their families.

a. CORRECT Ackerman is considered a pioneer in the assessment and treatment of families and the first to integrate the psychoanalytic approach with a systems approach in assessing and treating families.
b. Incorrect Whitaker applied experiential (humanistic) principles to family therapy.
c. Incorrect Stuart was an early contributor to behavioral family therapy.
d. Incorrect Liberman is a major figure in behavioral family therapy.

The correct answer is: Nathan Ackerman

How well did you know this?
1
Not at all
2
3
4
5
Perfectly
51
Q

__________ described “personal constructs” as bipolar dimensions of meaning that determine how a person perceives, interprets, and predicts events.
Select one:

A. William Glasser
B. George Kelly
C. Victor Frankl
D. Fritz Perls

A

This question should have been easy to answer as long as you’re familiar with the individual who developed Personal Construct Therapy.

a. Incorrect William Glasser is associated with Reality Therapy.
b. CORRECT Kelly’s Personal Construct Therapy is a constructivist approach that focuses on how people “construe” events and proposes that construing involves reliance on personal constructs.
c. Incorrect Victor Frankl is associated with Existential Therapy.
d. Incorrect Fritz Perls is associated with Gestalt Therapy.

The correct answer is: George Kelly

How well did you know this?
1
Not at all
2
3
4
5
Perfectly
52
Q

Helms’s (1995) White Racial Identity Development Model distinguishes between six statuses, with each status being associated with a different information processing strategy. For example, the “flexibility and complexity” strategy is characteristic of the __________ status.
Select one:

A. internalization
B. integrative awareness
C. reintegration
D. autonomy

A

Helms’s six identity statuses are:

CONTACT
DISINTEGRATION
REINTEGRATION
PSEUDO-INDEPENDENCE 
IMMERSION/EMERSION
AUTONOMY

a. Incorrect See explanation for response d.
b. Incorrect See explanation for response d.
c. Incorrect See explanation for response d.
d. CORRECT Not surprisingly, information processing flexibility and complexity are characteristic of the final status in Helms’s model. The attributes of the autonomy status include awareness of one’s own “whiteness” and role in perpetuating racism and a commitment to abandoning a sense of White entitlement.

The correct answer is: autonomy

How well did you know this?
1
Not at all
2
3
4
5
Perfectly
53
Q

Data published by Hans Eysenck in 1952:
Select one:

A. confirmed the effectiveness of behavioral treatments for a variety of psychological disorders.
B. confirmed the “Dodo bird” hypothesis, which predicts that various types of therapy are equally effective for a variety of psychological disorders.
C. challenged the notion of “spontaneous remission.”
D. challenged the effectiveness of conventional psychotherapeutic interventions.

A

Eysenck’s 1952 publication sparked research on psychotherapy outcomes.

a. Incorrect See explanation for response d.
b. Incorrect This sounds more like the Smith, Glass, and Miller (1980) meta-analysis, which found relatively few differences between various therapeutic approaches.
c. Incorrect This is just the opposite of what Eysenck concluded.
d. CORRECT Eysenck reported that 66% of patients in eclectic therapy and 44% in psychoanalytic psychotherapy improved versus 72% of those with similar problems who did not receive therapy.

The correct answer is: challenged the effectiveness of conventional psychotherapeutic interventions.

How well did you know this?
1
Not at all
2
3
4
5
Perfectly
54
Q

According to Irvin Yalom (1985), in group therapy, increasing intimacy and willingness to self-disclose and greater adherence to group norms are attributable to which of the following?
Select one:

A. cohesiveness
B. universality
C. identification
D. group transferences

A

Yalom describes the beginning of a new therapy group as involving three formative stages.

a. CORRECT According to Yalom, the third stage of group therapy is characterized by a growing sense of group cohesiveness, which leads to greater participation in the group and increased adherence to group norms.
b. Incorrect See explanation for response a.
c. Incorrect See explanation for response a.
d. Incorrect See explanation for response a.

The correct answer is: cohesiveness

How well did you know this?
1
Not at all
2
3
4
5
Perfectly
55
Q

As described by Sue and Sue (1999), “playing it cool” and “Uncle Tom Syndrome” are:
Select one:

A. signs of an internal locus of control.
B. survival mechanisms.
C. characteristics of the immersion-emersion stage of racial/cultural identity development.
D. signs of functional (versus cultural) paranoia.

A

D. W. Sue and D. Sue describe “playing it cool” and the “Uncle Tom Syndrome” as reactions of African Americans to long-term racial oppression (Counseling the culturally different, New York, John Wiley & Sons, 1999).

a. Incorrect See explanation for response b.
b. CORRECT Playing it cool helps African Americans conceal feelings that might be unacceptable to Whites and that, if expressed, could lead to harm and exploitation. The Uncle Tom Syndrome refers to the use of passivity as a defense mechanism.
c. Incorrect See explanation for response b.
d. Incorrect See explanation for response b.

The correct answer is: survival mechanisms.

How well did you know this?
1
Not at all
2
3
4
5
Perfectly
56
Q

In the book, A Theory of Multicultural Counseling and Therapy, Sue, Ivey, and Pedersen (1996) describe three multicultural counseling competencies. These are:
Select one:

A. ability, knowledge, and sensitivity.
B. flexibility, sensitivity, and awareness.
C. awareness, knowledge, and skills.
D. self-awareness, expertise, and strategies.

A

The three multicultural counseling competencies described by Sue, Ivey, and Pedersen (1996) were originally identified by D. W. Sue et al. (Position paper: Cross-cultural counseling competencies, The Counseling Psychologist, 10, 45-52, 1982).

a. Incorrect See explanation for response c.
b. Incorrect See explanation for response c.
c. CORRECT The first competency is AWARENESS of one’s own values, beliefs, and stereotypes regarding human behavior. The second competency is KNOWLEDGE about the worldview of minority and culturally-different clients. Finally, the third competency is possession of culturally-appropriate counseling SKILLS.
d. Incorrect See explanation for response c.

The correct answer is: awareness, knowledge, and skills.

How well did you know this?
1
Not at all
2
3
4
5
Perfectly
57
Q

Which of the following statements regarding sexual relationships between a therapist and a former client is most consistent with the APA’sEthics Code?
Select one:

A. A sexual relationship between a therapist and a former (but not current) client is ethical.
B. A therapist may have a sexual relationship after two years have passed since termination of the therapeutic relationship.
C. A therapist may have a sexual relationship after two years have passed since termination, the circumstances of the therapy and/or termination were not exploitative, and the client’s mental status and/or personal history does not show susceptibility to exploitation.
D. A sexual relationship between a therapist and a former client is always unethical.

A

Standard 10.08(a) addresses this issue. It states: “Psychologists do not engage in sexual intimacies with former clients/patients for at least two years after cessation or termination of therapy.” In addition Standard 10.08(b) states: “Psychologists do not engage in sexual intimacies with former clients/patients even after a two-year interval except in the most unusual circumstances. Psychologists who engage in such activity … bear the burden of demonstrating that there has been no exploitation, in light of all relevant factors ….”
a. Incorrect See explanation for response c.

b. Incorrect See explanation for response c.
c. CORRECT This answer is most consistent with the admonitions set forth in Standard 10.08.
d. Incorrect See explanation for response c.

The correct answer is: A therapist may have a sexual relationship after two years have passed since termination, the circumstances of the therapy and/or termination were not exploitative, and the client’s mental status and/or personal history does not show susceptibility to exploitation.

How well did you know this?
1
Not at all
2
3
4
5
Perfectly
58
Q

Which of the following is most consistent with ethical requirements regarding publication credit for publications that are substantially based on a student’s doctoral dissertation?
Select one:

A. Authorship credit must reflect each individual’s relative contribution to the research project.
B. Authorship credit must be mutually agreed upon by all individuals who contributed to the project.
C. The student must be listed as first author except under exceptional circumstances.
D. The student must always be listed as first author.

A

This issue is addressed in Standard 8.12(c) of the APA’s Ethics Code. a. Incorrect This is the general rule for authorship credit. However, the standards for publications based primarily on a student’s dissertation project differ.

b. Incorrect This is not consistent with the requirements of the Ethics Code.
c. CORRECT Standard 8.12(c) states that “Except under exceptional circumstances, a student is listed as principal author on any multiple-authored article that is substantially based on the student’s doctoral dissertation.” Note that timeliness is one consideration when determining if a circumstance is “exceptional.” As an example, the University of Victoria provides the following authorship guidelines for research conducted by graduate students: “If a student does not produce the first draft of the manuscript within e.g., 12 months of defending the thesis or dissertation, unless another arrangement has been negotiated in advance, the supervisor can take over the lead and be listed as the principal author. The student, however, remains an author of the article” (web.uvic.ca/psyc/GradAdvisor/GradRegs_Sept2004.pdf).
d. Incorrect The statement “except under exceptional circumstances” in Standard 8.12(c) suggests that there are times when it would be acceptable not to list the student as the principal author.

The correct answer is: The student must be listed as first author except under exceptional circumstances.

How well did you know this?
1
Not at all
2
3
4
5
Perfectly
59
Q

You go out for a drink with a psychologist who is a co-worker of yours at a mental health clinic. After several beers, he accidentally lets it slip that he is currently having a sexual relationship with one of his clients. Your best course of action would be to:
Select one:

A. discuss the matter with the clinic director as soon as possible.
B. discuss the issue with the psychologist when he has not been drinking.
C. make an appointment with the client to discuss the issue with her.
D. file a formal complaint against the psychologist with the Ethics Committee.

A

As noted in the Ethics Code, any attempt at resolving a potential ethical violation by a psychologist must include consideration of the client’s confidentiality.
a. Incorrect Although this may be appropriate, note that this answer does not mention the issue of confidentiality. Consequently, it is not the best answer of those given.

b. CORRECT This is the best response since it is most consistent with Standard 1.04 (Informal Resolution of Ethical Violations) of the Ethics Code and does not violate the client’s confidentiality.
c. Incorrect This would not be an appropriate course of action since there is nothing in the question to imply that you already have a therapeutic relationship with the client.
d. Incorrect See explanations for responses a and b.

The correct answer is: discuss the issue with the psychologist when he has not been drinking.

How well did you know this?
1
Not at all
2
3
4
5
Perfectly
60
Q

Dr. Dither is hired by the court to conduct an evaluation of a family as part of a child protection matter. Based on Dr. Dither’s evaluation and other evidence, the court determines that the child can remain at home with his family who will be provided with supervision and counseling. Six months after the case is heard in court, Dr. Dither is contacted by the mother who says she would like to begin therapy with him for issues unrelated to the child protection matter. Dr. Dither should:
Select one:

A. agree to see the woman in therapy since the child protection matter has been resolved.
B. agree to see the woman in therapy only if her problem is, in fact, unrelated to the child protection matter.
C. agree to see the woman in therapy since his familiarity with her situation will be an advantage.
D. refer the woman to a colleague.

A

In most situations, multiple relationships are to be avoided.
a. Incorrect See explanation for response d.

b. Incorrect See explanation for response d.
c. Incorrect See explanation for response d.
d. CORRECT This answer is most consistent with ethical guidelines that require psychologists to avoid multiple relationships. For example, Guideline 7 of the APA’s Guidelines for Child Custody Evaluations in Family Law Proceedings identifies providing psychotherapy to prior child custody examinees as a multiple relationship that should be avoided.

The correct answer is: refer the woman to a colleague.

How well did you know this?
1
Not at all
2
3
4
5
Perfectly
61
Q

The participants in a research study you are conducting at a university are all students in an introductory psychology class. As part of the study, you administer a test to each student that has been found to be an excellent predictor of suicide risk. You discover that one of the student’s scores on the test indicate that he is at high risk for suicide. Your best initial course of action would be to:
Select one:

A. contact the student’s instructor immediately.
B. contact the student to suggest that he see a therapist at the counseling center.
C. discuss your concerns with the student immediately.
D. re-administer the test to the student to confirm its results.

A

Immediate action is the appropriate response to situations involving a high risk for suicide.
a. Incorrect See explanation for response c.

b. Incorrect Referral to a therapist might be the next step after talking with the student.
c. CORRECT Of the responses given, this is the best one. It implies an immediate action that would help determine if the student is actually at risk.
d. Incorrect There’s no information in this question suggesting that the test needs to be re-administered.

The correct answer is: discuss your concerns with the student immediately.

How well did you know this?
1
Not at all
2
3
4
5
Perfectly
62
Q

A 15-year old client who you have been seeing for several months says he’s been thinking about killing himself. In response to your concern, he says he’s “just kidding” and asks that you not tell his parents. Your best course of action would be to:
Select one:

A. tell the boy that you need to contact his parents and then do so.
B. tell the boy that you won’t tell his parents as long as he signs a “no suicide” contract.
C. continue to discuss the matter to determine if the boy is, in fact, at risk for suicide.
D. do nothing since he was “just kidding” but ask him about the issue during the next session.

A

This is pretty straightforward. The boy’s welfare is your first concern, and you’d want to make sure that he is not at risk for suicide.

a. Incorrect See explanation for response c.
b. Incorrect See explanation for response c.
c. CORRECT Suicide threats and ideation should always be taken seriously. Before notifying his parents, however, you’d want to determine how serious his thoughts about suicide are.
d. Incorrect See explanation for response c.

The correct answer is: continue to discuss the matter to determine if the boy is, in fact, at risk for suicide.

How well did you know this?
1
Not at all
2
3
4
5
Perfectly
63
Q

Murphy, DeBernardo, and Shoemaker’s (1998) survey of psychologists about the effects of managed care on their professional practice found that the most frequently cited negative effect was:
Select one:

A. increased demands due to utilization review.
B. some panels being closed to psychologists.
C. income decline due to decreased volume of patients.
D. increased need for supervision.

A

This question is referring to a survey of APA Division 42 members [M. J. Murphy, C. R., DeBernardo, & W. E. Shoemaker, Impact of managed care on independent practice and professional ethics: A survey of independent practitioners, Professional Psychology: Research and Practice, 29(1), 43-51, 1998]. The results of that survey are described in the Ethical Manual in the written study materials.

a. CORRECT Increased demand due to utilization review was the most frequently cited negative effect, with 87% of respondents referring to the increase in paperwork and 83% referring to the increase in phone contacts with third-party payors.
b. Incorrect Thirty-eight percent of respondents cited some panels being closed to psychologists.
c. Incorrect Forty percent of respondents cited a decline in income due to a decrease in the volume of patients.
d. Incorrect Increased need for supervision was not one of the effects cited by respondents.

The correct answer is: increased demands due to utilization review.

How well did you know this?
1
Not at all
2
3
4
5
Perfectly
64
Q

Dr. D. Centt is conducting a study to investigate the effectiveness of a brief behavioral treatment for reducing self-mutilation in young women. The treatment involves 20 hourly sessions that will be administered once a week for 20 weeks. Dr. Centt randomly assigns 40 women with a history of self-mutilation to either the treatment group or the no-treatment control group. After women in the treatment group have completed only six sessions, he finds that the treatment has nearly eliminated their self-mutilating behaviors. In contrast, nearly all of the women in the control group report multiple incidents of self-mutilation since the study began. As an ethical psychologist, Dr. Centt’s best course of action would be to:
Select one:

A. continue the study as long as the women in the control group signed informed consents at the onset of their participation.
B. continue the study but offer the treatment to the women in the control group at the end of the study.
C. discontinue the study and offer the treatment to the women in the control group immediately.
D. try to get the results of the study published as soon as possible so that the effectiveness of the treatment is recognized.

A

This issue is addressed by A. J. Kimmel in Ethical issues in behavioral research, Blackwell Publishers, Cambridge, MA, 1996.

a. Incorrect See explanation for response c.
b. Incorrect See explanation for response c.
c. CORRECT Kimmel concludes that, when the effectiveness of an intervention is unknown, it is acceptable to include a “no-treatment control group” in a research study. However, as soon as the treatment is shown to be effective (especially when it is effective for a serious disorder or problem), the treatment should be made immediately available to individuals in the control group.
d. Incorrect See explanation for response c.

The correct answer is: discontinue the study and offer the treatment to the women in the control group immediately.

How well did you know this?
1
Not at all
2
3
4
5
Perfectly
65
Q

The Examination for Professional Practice in Psychology (EPPP) is best described as:
Select one:

A. a measure of basic knowledge of psychology.
B. a predictor of job proficiency.
C. an aptitude test.
D. a way to protect the public from incompetent psychologists.

A

The licensing exam measures knowledge of several areas in the field of psychology.

a. CORRECT The EPPP is a measure of knowledge that candidates seeking licensure should have.
b. Incorrect This would be nice, but the test has not been validated as a measure of job proficiency.
c. Incorrect This is not an accurate description of the licensing exam.
d. Incorrect This describes licensure but not the licensing exam.

The correct answer is: a measure of basic knowledge of psychology.

How well did you know this?
1
Not at all
2
3
4
5
Perfectly
66
Q

In the context of managed health care, “gatekeeper” refers to the:
Select one:

A. peer review board.
B. HMO.
C. clinical supervisor.
D. primary care physician.

A

Managed care reduces health costs in several ways including the use of a “gatekeeper” who makes referrals to specialists.

a. Incorrect See explanation for response d.
b. Incorrect See explanation for response d.
c. Incorrect See explanation for response d.
d. CORRECT The gatekeeper is the person who makes referrals to a psychotherapist or other specialist or the person who evaluates the individual to determine if additional care is needed. Very often, the gatekeeper is the individual’s primary care physician.

The correct answer is: primary care physician.

How well did you know this?
1
Not at all
2
3
4
5
Perfectly
67
Q

A former client leaves a voicemail message requesting that you send a copy of her clinical records to her new therapist. She leaves the new therapist’s address and phone number. You should:
Select one:

A. send the records to the therapist as requested by the client.
B. send the records to the client rather than to the therapist.
C. contact the client to clarify what information should be sent to the therapist and to obtain a verbal or written consent from the client.
D. contact the therapist to identify what information is relevant to the clients current therapy and then send that information to the therapist.

A

Before releasing any information to the client’s new therapist, you’d want to discuss the release with her to make sure she understands what information is being released. In addition, to be consistent with ethical obligations, you’d want to obtain a verbal or written consent before releasing the information.
a. Incorrect See explanation for response c.

b. Incorrect See explanation for response c.
c. CORRECT You would not want to provide another professional with confidential client information unless you had discussed this with the client and obtained a signed written consent or a verbal consent (which you would document in her file).
d. Incorrect Although contacting the therapist to determine what information is needed might be appropriate, you should not do so without first getting the client’s consent to communicate directly with the therapist. Therefore, answer b is a better response.

The correct answer is: contact the client to clarify what information should be sent to the therapist and to obtain a verbal or written consent from the client.

How well did you know this?
1
Not at all
2
3
4
5
Perfectly
68
Q

A colleague at the mental health clinic where you are employed tells you that one of his clients has admitted to sexually abusing his daughter. The colleague says he is not going to report the abuse since it was mild in nature and the client has expressed remorse, promised not to do it again, and is willing to continue therapy. In addition, he feels that reporting the abuse would be detrimental to the community since the client is a widely-known, well-respected person. You should:
Select one:

A. attempt to discuss the matter further with the colleague and encourage him to make a report to the appropriate child abuse reporting authority.
B. file a complaint against the colleague with APA or the state licensing board.
C. contact the appropriate child abuse reporting authority and provide them with information about the abuse and with the colleague’s name.
D. contact the appropriate child abuse reporting authority and provide them with information about the abuse but refuse to give them the colleague’s name if they request it.

A

This issue is addressed by Leland Swenson in Psychology and the law for the helping pofessions, Brooks/Cole, Pacific Grove, CA, 1997.

a. Incorrect See explanation for response c.
b. Incorrect See explanation for response c.
c. CORRECT Swenson notes that “Any required reporter who knows that another required reporter has not reported must report the evidence of abuse as well as identify the nonreporter” (p. 413).
d. Incorrect See explanation for response c.

The correct answer is: contact the appropriate child abuse reporting authority and provide them with information about the abuse and with the colleague’s name.

How well did you know this?
1
Not at all
2
3
4
5
Perfectly
69
Q

An evaluation of a defendant’s competence in a criminal court case is conducted to determine his/her mental status:
Select one:

A. at the time of the evaluation.
B. at the time the crime was committed.
C. at the time of the arrest.
D. prior to committing the crime.

A

This question is apparently asking about competence to stand trial.

a. CORRECT A psychologist may be asked to evaluate a person’s current mental status to determine if he/she is competent to stand trial (i.e., is able to understand court procedures and the charge against him/her).
b. Incorrect A psychologist would determine the person’s status at the time of the crime in order to support the insanity defense.
c. Incorrect See explanation above.
d. Incorrect See explanation above.

The correct answer is: at the time of the evaluation.

How well did you know this?
1
Not at all
2
3
4
5
Perfectly
70
Q

You receive a phone call from the attorney of one of your current therapy clients. The attorney asks you to forward information from the client’s file for a court case the client is currently involved in and tells you that the client has signed a release. While reviewing the client’s file, you encounter a note that you made indicating that the client said he would never want you to release personal information about him under any circumstances. As an ethical psycholgist, you will:
Select one:

A. contact the client to discuss the matter with him before taking any further action.
B. release only information to the attorney that you believe is relevant to the client’s case.
C. release the requested information to the attorney since the client has signed a release.
D. refuse to release the records without a court order.

A

The situation described in this question is not directly addressed by the APA’s Ethics Code. However, one of the answers is most consistent with the “spirit” of the Code.

a. CORRECT This response is most consistent with the requirements of the Ethics Code. Specifically, Standard 3.04 states that psychologists “take reasonable steps to avoid harming their clients/patients.” Contacting the client prior to releasing the requested information to the attorney would help ensure that the client is not harmed.
b. Incorrect Releasing only relevant information is always the best action but, before making a decision about releasing the requested information, you’d want to discuss the issue with the client given that he said he would never want you to release personal information under any circumstance. Therefore, answer a is the best response.
c. Incorrect Because of the client’s statement to you about the release of personal information, you would want to check with him to make sure he understands what information will be released and how that information will be used. You would also want to see a copy of the release before providing the attorney with the requested information.
d. Incorrect There is no information given in the question suggesting that it would be necessary to obtain a court order before releasing the requested information to the attorney.

The correct answer is: contact the client to discuss the matter with him before taking any further action.

How well did you know this?
1
Not at all
2
3
4
5
Perfectly
71
Q

You receive a court order requesting that you provide information about a 19-year-old client who is currently the defendant in a court case. You discuss the request with the client but he refuses to sign a waiver of confidentiality. You should:
Select one:

A. refuse to provide the court with the requested information until you obtain a waiver from the client.
B. refuse to provide the court with the requested information until you obtain a waiver from the client’s attorney or legal guardian.
C. provide the court only with information that you believe will not be harmful to the client.
D. provide the court with the requested information.

A

The key term in this question is “court-ordered.”

a. Incorrect See explanation for response d.
b. Incorrect See explanation for response d.
c. Incorrect See explanation for response d.
d. CORRECT Therapist-client privilege implies that, in most circumstances, a psychologist does not reveal confidential client information in the context of a legal proceeding without the consent of the client or his/her legal representative. There are exceptions to privilege, however, and a court order is one of these exceptions. (If the information is damaging to the client, the normal procedure is to request that it be read by the judge in camera - i.e., in the privacy of the judge’s chambers.)

The correct answer is: provide the court with the requested information.

How well did you know this?
1
Not at all
2
3
4
5
Perfectly
72
Q

With regard to the care and use of animals in research, a psychologist who is the principal investigator:
Select one:

A. can designate which employees or assistants are responsible for the animals.
B. can designate which employees or assistants are responsible for the animals as long as the psychologist has provided them with appropriate training.
C. is ultimately responsible for the animals him/herself.
D. shares equal responsibility for the animals with the employees and assistants.

A

The Ethics Code is not “crystal clear” about this issue, but it does seem to imply that the psychologist maintains responsibility for the care and treatment of animal subjects.

a. Incorrect See explanation for response c.
b. Incorrect See explanation for response c.
c. CORRECT Standard 8.09(b) states that “Psychologists trained in research methods and experienced in the care of laboratory animals supervise all procedures involving animals and are responsible for ensuring appropriate consideration of their comfort, health, and humane treatment.” In light of this statement, this response is the best one.
d. Incorrect See explanation for response c.

The correct answer is: is ultimately responsible for the animals him/herself.

How well did you know this?
1
Not at all
2
3
4
5
Perfectly
73
Q

Which of the following is generally considered the best defense for a psychologist in cases of litigation?
Select one:

A. licensure
B. adequate records
C. informed consent
D. liability insurance

A

A review of the literature on risk management reveals consensus among the experts that good record-keeping is a key defense in cases of malpractice.

a. Incorrect See explanation for response b.
b. CORRECT S. Knapp, for instance, notes that “An axiom of risk management is that if it is not documented, it did not occur” (Professional liability and risk management in an era of managed care, in D. T. Marsh and R. O. Magee [Eds.], Ethical and legal issues in professional practice with families, New York, John Wiley and Sons, 1997).
c. Incorrect See explanation for response b.
d. Incorrect See explanation for response b.

The correct answer is: adequate records

How well did you know this?
1
Not at all
2
3
4
5
Perfectly
74
Q

A married couple’s insurance provides reimbursement for individual therapy but not for marital therapy. The psychologist has determined, however, that marital therapy is the appropriate form of treatment for the couple. The psychologist should:
Select one:

A. treat them in individual therapy since it is covered by the insurance.
B. bill for individual therapy while treating them as a couple.
C. appeal to the insurance company for coverage for marital therapy.
D. none of the above.

A

The psychologist should not mislead the insurance company about the appropriate form of treatment for the couple since doing so would constitute insurance fraud.

a. Incorrect This would not be appropriate because individual therapy is not indicated.
b. Incorrect Standard 6.06 of the Ethics Code states: “In their reports to payors for services or sources of research funding, psychologists take reasonable steps to ensure the accurate reporting of the nature of the service provided or research conducted, the fees, charges, or payments, and where applicable, the identity of the provider, the findings, and the diagnosis.”
c. CORRECT The psychologist should make a request to the insurance company for marital therapy.
d. Incorrect See explanation above.

The correct answer is: appeal to the insurance company for coverage for marital therapy.

How well did you know this?
1
Not at all
2
3
4
5
Perfectly
75
Q

A 23-year old client of yours is currently the defendant in a court case. The client’s attorney contacts you, requesting that you provide him with some information about the client from your files. The information will be helpful in preparing the client’s defense. You:
Select one:

A. release the information since, to do so, is in the “best interests of the client.”
B. release the information because privilege is waived in this situation.
C. release only the information you believe is relevant to the case.
D. discuss the release with the client before taking any additional action.

A

The release of confidential client information ordinarily requires a waiver from the client.

a. Incorrect See explanation for response d.
b. Incorrect See explanation for response d.
c. Incorrect See explanation for response d.
d. CORRECT Of the responses given, this is the best one. Privilege may or may not be waived in this situation and, even if it is, you should discuss the release of confidential information (even to the client’s attorney) with the client and, in most cases, obtain a waiver.

The correct answer is: discuss the release with the client before taking any additional action.

How well did you know this?
1
Not at all
2
3
4
5
Perfectly
76
Q

According to the APA’s Specialty Guidelines for Forensic Psychology (APA, 2012), accepting contingent fees for professional services in a legal proceeding:
Select one:

A. is acceptable.
B. is acceptable only if the fees are “fair and equitable.”
C. should be avoided unless doing so does not interfere with the psychologist’s objectivity.
D. should ordinarily be avoided.

A

This issue is addressed in Paragraph 5.02 of the Specialty Guidelines.

a. Incorrect See explanation for response d.
b. Incorrect See explanation for response d.
c. Incorrect See explanation for response d.
d. CORRECT This answer is most consistent with the provisions of Paragraph 5.02 which states: “Forensic practitioners seek to avoid undue influence that might result from financial compensation or other gains. Because of the threat to impartiality presented by the acceptance of contingent fees and associated legal prohibitions, forensic practitioners strive to avoid providing professional services on the basis of contingent fees.”

The correct answer is: should ordinarily be avoided.

How well did you know this?
1
Not at all
2
3
4
5
Perfectly
77
Q

During her third therapy session, Mrs. Maitalain, age 32, reveals that her husband has “forced her” to have sex with him two or three times in the last six months. She says that she usually agrees to have sexual relations with him but sometimes doesn’t feel like it because she’s too tired; and, in those circumstances, he sometimes uses threats and physical violence to force her to do so. Her therapist should:
Select one:

A. inform Mrs. Maitalain that he is legally required to report these incidents to the appropriate authorities in order to protect her from future harm.
B. make a report to the appropriate authorities only if he believes that Mrs. Maitalain is at high risk for future harm from her husband.
C. maintain Mrs. Maitalains confidentiality but discuss her options in this situation.
D. encourage Mrs. Maitalain to bring her husband to the next therapy session.

A

In most situations, psychologists must maintain the confidentiality of their clients unless the client waives his/her right to confidentiality or a breach of confidentiality is mandated by law.

a. Incorrect See explanation for response c.
b. Incorrect See explanation for response c.
c. CORRECT Psychologists ordinarily maintain the confidentiality of a client who is the victim of spousal or partner abuse, and the primary intervention in this situation would be to ensure the safety of Mrs. Maitalain, which would include discussing her options. An exception is when there is reason to suspect that children are also at risk. Also note that some jurisdictions mandate reporting of certain types of abuse for “vulnerable adults,” but there is no information given in this question that indicates that Mrs. Maitalain is a vulnerable adult.
d. Incorrect See explanation for response c.

The correct answer is: maintain Mrs. Maitalains confidentiality but discuss her options in this situation.

How well did you know this?
1
Not at all
2
3
4
5
Perfectly
78
Q

A psychologist has just started seeing a third grader who has been exhibiting behavioral problems at school and home. The psychologist suspects that the child has dyslexia but has had no training in the evaluation of this disorder. The child’s parents want the psychologist to conduct the evaluation because of the good rapport she has with the child. The psychologist should:
Select one:

A. conduct the evaluation only after informing the parents of their options and obtaining an informed consent.
B. conduct the evaluation only after attending a workshop on Learning Disorders.
C. conduct the evaluation only if available assessment instruments do not require extensive training to administer.
D. refer the child to a colleague who has experience in Learning Disorders for the evaluation.

A

This question addresses the issue of competence. The Ethics Code clearly prohibits psychologists from working outside their scope of the competence and expertise.

a. Incorrect See explanation for response d.
b. Incorrect See explanation for response d.
c. Incorrect See explanation for response d.
d. CORRECT Although psychologists can (and should) add to their skills, doing so requires special care so that the welfare of a client is not jeopardized. Mislabeling a child as learning disabled (or not learning disabled) can have very serious and long-lasting consequences, so this response is the best of those given.

The correct answer is: refer the child to a colleague who has experience in Learning Disorders for the evaluation.

How well did you know this?
1
Not at all
2
3
4
5
Perfectly
79
Q

It would be acceptable to include client testimonials in an advertisement or brochure for your services as long as:
Select one:

A. the testimonials are not from current therapy clients.
B. the testimonials have not been directly solicited by you from current or past clients.
C. the testimonials are not from current clients or others who are susceptible to exploitation.
D. the testimonials have not been solicited from current clients or from others who are vulnerable to undue influence.

A

This question requires you to be familiar with the language of the Ethics Code. Testimonials are not entirely prohibited but should be obtained and used with caution.

a. Incorrect This is a good answer but not as complete as response d.
b. Incorrect Direct solicitation from a former client is not necessarily prohibited.
c. Incorrect This is close but, if you read carefully, it’s not a completely accurate reflection of the requirements of the Ethics Code.
d. CORRECT This is closest to the actual language of Standard 5.05 of the Ethics Code and, consequently, is the best answer.

The correct answer is: the testimonials have not been solicited from current clients or from others who are vulnerable to undue influence.

How well did you know this?
1
Not at all
2
3
4
5
Perfectly
80
Q

The mother of a previous client of yours asks for a copy of her daughter’s records. The daughter saw you for two years and died three months ago when she was 38. You should:
Select one:

A. send the woman a copy of the records as requested.
B. send the woman only that information you believe will not be harmful.
C. allow the woman to inspect the records in your office.
D. not release any information without appropriate authorization.

A

Although the laws vary from jurisdiction to jurisdiction, in general, a psychologist should not release information after a client’s death without proper authorization (e.g., from the executor of the estate or the client’s legal representative).

a. Incorrect See explanation for response d.
b. Incorrect See explanation for response d.
c. Incorrect See explanation for response d.
d. CORRECT This is the best course of action. See, e.g., R. I. Simon, Clinical psychiatry and the law, American Psychiatric Press, Washington, DC, 1992.

The correct answer is: not release any information without appropriate authorization.

How well did you know this?
1
Not at all
2
3
4
5
Perfectly
81
Q

Dr. Nillson, a behavioral psychologist, is teaching clinical psychology at a university. She does not use, nor can she tolerate the use of, psychodynamic principles and interventions in psychotherapy. She refuses to teach psychodynamic therapy in her classes and actually makes fun of it in front of her students during informal gatherings. Her behavior is:
Select one:

A. unethical she has an obligation to provide accurate and objective information about the topic she is teaching.
B. ethical she has the freedom of choice to teach whatever she wants in her classes.
C. acceptable she would be poorly trained to teach psychodynamics and psychotherapy anyway.
D. ethical but she should not be making fun of any psychological practice in front of her students.

A

Dr. Nillson’s behavior violates Standard 7.03 of the Ethics Code, which calls for accuracy in teaching, as well as General Principle C (Integrity), which encourages “accuracy, honesty, and truthfulness.”

a. CORRECT Dr. Nillson’s behavior is in violation of the Ethics Code.
b. Incorrect As a psychologist, Dr. Nillson has an ethical duty to teach objectively, fully, and accurately.
c. Incorrect Dr. Nillson is ethically obligated to make sure her choice of topics is not biased by her own personal beliefs. If she is unqualified to teach certain areas, she could arrange to have someone else do so.
d. Incorrect Dr. Nillson’s behavior is biased and, therefore, is unethical.

The correct answer is: unethical she has an obligation to provide accurate and objective information about the topic she is teaching.

How well did you know this?
1
Not at all
2
3
4
5
Perfectly
82
Q

The review process used by managed care organizations (MCOs) to ensure that providers meet and maintain the requirements to participate in a health plan is referred to as:
Select one:

A. concurrent review.
B. credentialing.
C. quality management.
D. clinical audit.

A

For the exam, you want to be familiar with the managed care terms listed in the answers to this question.

a. Concurrent review is the term used for utilization reviews conducted during the course of treatment.
b. CORRECT Credentialing is a formal process for determining whether or not a provider meets and maintains qualification standards and provides some legal protection for the MCO.
c. Quality management is another name for quality assurance.
d. Clinical audit is a method of quality assurance.

The correct answer is: credentialing.

How well did you know this?
1
Not at all
2
3
4
5
Perfectly
83
Q

A psychologist who obtained a Ph.D. in organizational psychology wants to change her specialty to clinical psychology. To meet the requirements of APA’s guidelines regarding a change in specialty, the psychologist must:
Select one:

A. complete an internship in clinical psychology under the supervision of a licensed clinical psychologist.
B. complete appropriate doctoral-level classes and supervised post-doctoral training.
C. obtain a second Ph.D. in clinical psychology from an accredited university or professional school.
D. obtain appropriate supervision of her clinical practice.

A

This issue is addressed in the APA’s General Guidelines for Providers of Psychological Services and Specialty Guidelines for Clinical Psychologists.

a. Incorrect This would not be adequate.
b. CORRECT Section 1.7 of the Specialty Guidelines states that “professional psychologists who wish to qualify as clinical psychologists meet the same requirements with respect to subject matter and professional skills that apply to doctoral and postdoctoral education and training in clinical psychology.”
c. Incorrect This would not be sufficient.
d. Incorrect This would not be sufficient.

The correct answer is: complete appropriate doctoral-level classes and supervised post-doctoral training.

How well did you know this?
1
Not at all
2
3
4
5
Perfectly
84
Q

An insurance company is conducting a peer review and requests that you provide it with information about a current client whose fee is being paid by the company. In this situation, you are best advised to:
Select one:

A. provide the company with the requested information only if the client is willing for you to do so.
B. provide the company with only that information you believe to be pertinent to the review.
C. provide the company with relevant information after ensuring that it is aware of the need to maintain confidentiality.
D. provide the company with the requested information because the client waived confidentiality when he/she signed the insurance form.

A

Although this issue is not explicitly addressed in the Ethics Code, the “spirit” of the Code dictates that confidential information be treated with care.

a. Incorrect See explanation for response c.
b. Incorrect See explanation for response c.
c. CORRECT When a client’s therapy fee has been paid by an insurance company, it is likely that the client has already signed a waiver on his/her insurance form. In addition, therapists are expected to cooperate with peer reviews. Therefore, this is the best response. A therapist would be required to provide information to the insurance company as requested, but the therapist should release only relevant information and should take steps to ensure that confidentiality will be safeguarded.
d. Incorrect See explanation for response c.

The correct answer is: provide the company with relevant information after ensuring that it is aware of the need to maintain confidentiality.

How well did you know this?
1
Not at all
2
3
4
5
Perfectly
85
Q

Except under certain conditions, psychologists are ethically obligated to maintain a client’s confidentiality. Potential limitations on confidentiality should be:
Select one:

A. discussed at the onset of the professional relationship.
B. discussed at the onset of the professional relationship and when feasible thereafter as needed.
C. discussed and documented in writing as soon as feasible.
D. discussed when the psychologist deems it to be appropriate.

A

None of the responses given are technically “wrong,” but only one reflects the actual language of the Ethics Code and, therefore, is the best answer.

a. Incorrect See explanation for response b.
b. CORRECT This is the language of Standard 4.02: Discussing the Limits of Confidentiality. Documenting the discussion in writing (response c) may be a good idea but is not required.
c. Incorrect See explanation for response b.
d. Incorrect See explanation for response b.

The correct answer is: discussed at the onset of the professional relationship and when feasible thereafter as needed.

How well did you know this?
1
Not at all
2
3
4
5
Perfectly
86
Q

In the context of Holland’s theory of vocational choice, a high degree of differentiation:
Select one:

A. reflects an uncommon or unusual pattern of interests.
B. reflects a common pattern of interests.
C. increases the predictability of the person-environment interaction.
D. decreases the predictability of the person-environment interaction.

A

Holland used the term differentiation to describe the extent to which a person has clearly defined interests. A person who scores high on one of his interest scales and low on all other scales is highly differentiated.

a. Incorrect See explanation for response c.
b. Incorrect See explanation for response c.
c. CORRECT According to Holland, the importance of a good “fit” between a person’s interests and the characteristics of the job is most important for those who are highly differentiated.
d. Incorrect See explanation for response c.

The correct answer is: increases the predictability of the person-environment interaction.

How well did you know this?
1
Not at all
2
3
4
5
Perfectly
87
Q

According to Janis (1972), groupthink is most likely to occur when:
Select one:

A. a group is highly cohesive.
B. a group has a “laissez-faire” leader.
C. group members have low levels of organizational commitment.
D. group members are overly influenced by outside opinions.

A

Groupthink was originally described by Arthur Janis to describe several failed foreign policy decisions including the Kennedy administration’s decision to invade Cuba.

a. CORRECT Although Janis did not believe that high cohesiveness alone necessarily leads to groupthink, he considered it to be the primary condition and proposed that cohesiveness is likely to lead to poor decision making when it is combined with certain other conditions including insulation from outside opinions, the presence of external threats, and highly directive leadership.
b. Incorrect See explanation for response a.
c. Incorrect See explanation for response a.
d. Incorrect See explanation for response a.

The correct answer is: a group is highly cohesive.

How well did you know this?
1
Not at all
2
3
4
5
Perfectly
88
Q

In their research on terrorist groups, McCauley and Segal (1987) found that, when people come together because of a shared grievance, their views often become increasingly extreme as they continue to interact in isolation from possible moderating influences. This finding supports which of the following?
Select one:

A. deindividuation
B. social trap
C. group polarization
D. group contagion

A

This question is asking about a situation in which the views of group members become increasingly extreme due to their participation in the group.

a. Incorrect Individuation occurs when a person feels “anonymous” as a member of a group.
b. Incorrect A social trap is a conflict that occurs when behavior has positive short-term consequences but negative long-term consequences.
c. CORRECT Group polarization refers to the tendency of people to make more extreme decisions when acting as members of a group than they would have made as individuals.
d. Incorrect Group contagion refers to the transmission of emotion through a crowd or group.

The correct answer is: group polarization

How well did you know this?
1
Not at all
2
3
4
5
Perfectly
89
Q

The three components of a needs assessment are:
Select one:

A. microsystem, macrosystem, and exosystem.
B. unfreezing, changing, refreezing.
C. worker satisfaction, worker performance, and organizational effectiveness.
D. organization analysis, task analysis, and person analysis.

A

A needs assessment is conducted in organizations to determine training needs.

a. Incorrect These are systems in Bronfenbrenner’s ecological model.
b. Incorrect These are the stages in Lewin’s model of planned change.
c. Incorrect These are not the components of a needs assessment.
d. CORRECT The systems approach to needs assessment involves three types of analysis - organization, task/job, and person.

The correct answer is: organization analysis, task analysis, and person analysis.

How well did you know this?
1
Not at all
2
3
4
5
Perfectly
90
Q

Which of the following is most relevant to equity theory?
Select one:

A. Rosenthal effect
B. social comparison theory
C. drive theory
D. the buffering hypothesis

A

Equity theory predicts that motivation is a function of the comparisons workers make between their own input/outcome ratio and those of workers doing similar jobs.

a. Incorrect The Rosenthal effect is another name for the self-fulfilling prophecy effect, which isn’t really relevant to equity theory.
b. CORRECT According to equity theory, motivation is a result of our social comparisons. Additional information on equity theory is provided in the Industrial-Organizational Psychology chapter of the written study materials.
c. Incorrect Drive theory is also a theory of motivation but isn’t as relevant to equity theory as social comparison theory.
d. Incorrect The buffering hypothesis predicts that the perception of adequate social support buffers people against the negative effects of stress.

The correct answer is: social comparison theory

How well did you know this?
1
Not at all
2
3
4
5
Perfectly
91
Q
During the sixth week of the semester, students in an introductory psychology class meet on a Saturday morning to discuss their class project. In the first meeting, a couple of the students say they think the instructor is boring and a "bad" teacher. The other students say they disagree and that he "isn't so bad." At the end of their meeting, however, one of the students says, "Let's take a vote - how many of you agree that our instructor is one of the worst teachers you've ever had?" and every student raises his or her hand. The students' agreement about the instructor's lack of ability is an example of which of the following?
Select one:

A. social facilitation
B. group polarization
C. the fundamental attribution bias
D. negative framing

A

In this situation, the evaluation of the instructor has become more extreme for some group members as a result of their participation in the group.

a. Incorrect Social facilitation occurs when performance on a task is improved as the result of the presence of other people.
b. CORRECT Group polarization refers to the tendency of groups to make more extreme decisions than individual group members would make alone. Of the responses given, group polarization best “fits” the situation described in this question.
c. Incorrect The fundamental attribution bias refers to the tendency to make dispositional attributions when judging the behavior of others.
d. Incorrect Negative framing refers to presenting a problem to a person in negative terms.

The correct answer is: group polarization

92
Q

The Americans with Disabilities Act:
Select one:

A. does not address the issue of drug testing.
B. permits drug testing only after a job offer has been made.
C. permits drug testing before or after a job offer has been made.
D. prohibits drug testing until after a job applicant has been hired and there is reason to suspect drug use.

A

The Americans with Disabilities Act protects the rights of people with physical and mental disabilities.

a. Incorrect See explanation for response c.
b. Incorrect See explanation for response c.
c. CORRECT Although ADA permits only post-offer, pre-employment medical exams, it explicitly excludes drug testing as a medical exam. In other words, drug testing is not prohibited at any time during pre-employment or employment.
d. Incorrect See explanation for response c.

The correct answer is: permits drug testing before or after a job offer has been made.

93
Q

Incentive/reward theory implies that which of the following is most important for ensuring worker motivation?
Select one:

A. clearly tying rewards and punishments to effective performance
B. making jobs interesting, attractive, and satisfying
C. explicitly linking individual goals to organizational goals
D. allowing workers to identify their preferred benefits

A

The implications of incentive/reward and other motivational theories are summarized by R. A. Katzell and D. E. Thompson in Work motivation: Theory and practice, American Psychologist, 45(2), 144-153, 1990.

a. Incorrect This is an implication of reinforcement theory, which is different from incentive/reward theory in its premises and implications.
b. CORRECT Incentive/reward theory is broader than reinforcement theory and emphasizes the features of the job and work environment that maximize worker interest and satisfaction.
c. Incorrect This is not a key implication of incentive/reward theory.
d. Incorrect This is also not an implication of incentive/reward theory.

The correct answer is: making jobs interesting, attractive, and satisfying

94
Q

Research about job satisfaction has most consistently found that:
Select one:

A. job satisfaction is inversely related to turnover and absenteeism.
B. job satisfaction is inversely related to age.
C. job satisfaction is inversely related to education level.
D. job satisfaction levels are the same for males and females.

A

There has been a myriad of research investigating the relationship between job satisfaction and various measures of job performance. The most consistent finding is that job satisfaction is inversely related to absenteeism and turnover.

a. CORRECT Job satisfaction has consistently been found to be inversely correlated with both turnover and absenteeism (although it appears to have a slightly stronger relationship with turnover than absenteeism).
b. Incorrect Conversely, job satisfaction tends to increase with age.
c. Incorrect Conversely, individuals with higher levels of education tend to have higher levels of satisfaction.
d. Incorrect The results of research investigating the differences between males and females have been inconsistent. Some studies indicate that males tend to be more satisfied, while others indicate that females are more satisfied.

The correct answer is: job satisfaction is inversely related to turnover and absenteeism.

95
Q

In mediation:
Select one:

A. the mediator considers the preferences of all involved parties but his/her decision or problem solution is binding.
B. the mediator makes a formal recommendation following a review of the facts, but the recommendation may or may not be accepted by the involved parties.
C. the mediator facilitates the flow of information between the involved parties and helps them progress toward an acceptable compromise.
D. the mediator provides the “final vote” that is needed to break a deadlock between the involved parties.

A

Mediation is often distinguished from fact-finding and arbitration.

a. Incorrect This sounds more like arbitration.
b. Incorrect This sounds a more like fact-finding than mediation.
c. CORRECT In mediation, a neutral third party (the mediator) uses various tactics to facilitate voluntary agreement between disputants. Mediators can make recommendations, but they have no formal power and cannot impose their solution or decision.
d. Incorrect This does not describe the role of a mediator.

The correct answer is: the mediator facilitates the flow of information between the involved parties and helps them progress toward an acceptable compromise.

96
Q

In the context of Porter and Lawler’s (1968) expectancy theory, __________ refers to the belief that meeting job performance goals will lead to certain outcomes.
Select one:

A. valence
B. instrumentality
C. vigor
D. expectancy

A

Expectancy theory predicts that motivation is a function of three beliefs: expectancy, instrumentality, and valence. For additional information on expectancy theory, see the Industrial-Organizational Psychology chapter of the written study materials.

[EIV]

a. Incorrect See explanation for response b.
b. CORRECT Instrumentality refers to the belief that good performance will be rewarded.
c. Incorrect See explanation for response b.
d. Incorrect See explanation for response b.

The correct answer is: instrumentality

97
Q

Group decisions tend to be better than decisions made by individuals when:
Select one:

A. the task requires a high degree of creativity.
B. the group is highly cohesive.
C. the group consists of members with complementary expertise.
D. the group has a directive leader.

A

The research has shown that the effectiveness of group decision-making is affected by a number of factors (e.g., groupthink, the risky shift).

a. Incorrect Individual decisions tend to be better than group decisions for tasks that require a high degree of creativity.
b. Incorrect A high degree of cohesiveness can lead to groupthink.
c. CORRECT Not surprisingly, groups are more effective when members have different skills and knowledge.
d. Incorrect A directive leader can lead to groupthink.

The correct answer is: the group consists of members with complementary expertise.

98
Q

From the perspective of the situational leadership model, a “telling” leadership style is most effective when an employee is:
Select one:

A. low in ability and low in willingness.
B. low in ability and high in willingness.
C. high in ability and low in willingness.
D. high in ability and high in willingness.

A

The situational leadership model distinguishes between four leadership styles and proposes that the optimal style depends on the employee’s maturity level, which is determined by a combination of his/her ability and willingness to take responsibility.

A. Telling
B. Selling
C. Participating
D. Delegating

a. CORRECT Hersey and Blanchard recommend a telling style when the employee is low in both ability and willingness. Even if you’re not familiar with the theory, it makes sense that this type of employee would need to be told what to do.
b. Incorrect A selling style is best for this type of employee.
c. Incorrect A participating style is best for employees high in ability and low in willingness.
d. Incorrect A delegating style is optimal for this type of employee.

The correct answer is: low in ability and low in willingness.

99
Q

A problem with bounded rationality is that decision makers often examine possible alternatives only until a solution that meets minimal requirements is found and then stop looking for better alternatives. This is referred to as:
Select one:

A. minimizing.
B. polarizing.
C. suboptimizing.
D. satisficing.

A

Research examining the strengths and weaknesses of various decision-making strategies has found that bounded rationality often fails because of the problem described in this question.

a. Incorrect See explanation for response d.
b. Incorrect See explanation for response d.
c. Incorrect See explanation for response d.
d. CORRECT This is the term used in the literature for the phenomenon described in this question.

The correct answer is: satisficing.

100
Q

According to McGregor (1960), Theory X managers assume that:
Select one:

A. employees have an inherent dislike of work and will avoid it when they can.
B. employees are inherently motivated to work and are capable of self-control and self-direction.
C. the most effective management style depends on the nature of the organizational climate.
D. the most effective management style depends on the nature of the task and certain characteristics of the employees.

A

As described by McGregor (1960), Theory X and Theory Y managers are distinguished by their different assumptions about employee characteristics.

a. CORRECT Knowing that Theory X managers have a pessimistic view of employees would have enabled you to identify this as the correct response.
b. Incorrect This is more consistent with the assumptions of Theory Y managers who have more positive views of employees.
c. Incorrect McGregor’s theory does not identify factors that determine the most effective management style. Instead, McGregor proposed that a Theory Y style is most likely to have the greatest benefits for an organization.
d. Incorrect See explanations above.

The correct answer is: employees have an inherent dislike of work and will avoid it when they can.

101
Q

In a study designed to evaluate the correlates of achievement motivation, a psychologist would be likely to find that, in comparison to people with a low need for achievement, people with a high need for achievement will prefer which of the following kinds of tasks?
Select one:

A. tasks of low difficulty
B. tasks of moderate difficulty
C. tasks of high difficulty
D. tasks representing a range of difficulty levels

A

Achievement-oriented people tend to prefer moderate to moderately difficult tasks.

a. Incorrect You probably could have eliminated this response on the basis of common sense. People who are achievement-oriented would not get too much gratification from success on easy tasks.
b. CORRECT This might not be the answer you’d guess if you’re totally unfamiliar with the research on this topic, but, in fact, it is the correct answer.
c. Incorrect Somewhat surprisingly, it is people with a low need for achievement who choose more difficult tasks.
d. Incorrect If you had eliminated answer “a” for the correct reason, that should have helped you eliminate this answer too since tasks of varying difficulty levels would include easy tasks.

The correct answer is: tasks of moderate difficulty

102
Q

Research investigating the impact of group heterogeneity and task performance has generally found that:
Select one:

A. group heterogeneity is always preferable to group homogeneity regardless of the type of task.
B. group homogeneity is always preferable to group heterogeneity regardless of the type of task.
C. group heterogeneity is preferable on disjunctive tasks but can actually be detrimental on some other types of tasks.
D. group heterogeneity is preferable on conjunctive tasks but can actually be detrimental on some other types of tasks.

A

A good “rule-of-thumb” for answering questions is to eliminate responses that contain absolutes (e.g., “always” or “never”).

a. Incorrect See explanation for response c.
b. Incorrect See explanation for response c.
c. CORRECT Groups that are heterogeneous with regard to members’ skills, experience, and other characteristics have been found to be particularly beneficial for disjunctive tasks in which only one member needs to identify the correct response or solution. For a discussion of this issue, see D. T. Kenrick, S. L. Heuberg, & R. B. Cialdini, Social psychology: Unraveling the mystery, Boston, Allyn & Bacon, 2001.
d. Incorrect Heterogeneity can actually be detrimental on conjunctive tasks in which group success depends on the ability of each member to perform his/her role effectively (and the roles do not require heterogeneity).

The correct answer is: group heterogeneity is preferable on disjunctive tasks but can actually be detrimental on some other types of tasks.

103
Q

Joshua is dissatisfied with the amount of his bonus and he believes his other co-workers received more than he did. This greatly angers him. Joshua is concerned with what type of justice?
Select one:

A. Compensatory
B. Procedural
C. Corrective
D. Distributive

A

The correct answer is D. In fairness research, there is a focus on what a person receives (distributive justice) and how perceived goods are allocated (procedural justice), and then there is the interpersonal treatment received by individuals as the justice is carried out. For the most part, organizational justice is concerned with the ways in which employees decide if they been treated fairly in their jobs and the conditions in which those determinations influence other work-related variables.

Answer A is incorrect as compensatory justice tends to be more for criminal matters and how punishment is administered to the convict.

Answer B is incorrect as procedural justice would have been in this case the amount of money that the company set aside for bonuses as opposed to how much each individual receives.

Answer C is incorrect as this is a type of justice that has more to do with criminal punishment than social fairness.

The correct answer is: Distributive

104
Q

The concepts of “job relatedness” and “business necessity” are associated with which of the following?
Select one:

A. adverse impact
B. truth in testing
C. comparable worth
D. personnel training

A

Job relatedness and business necessity are conditions that may permit the use of a selection procedure that is having an adverse impact for members of a protected group.

a. CORRECT If a selection or other employment procedure is found to be having adverse impact, the employer may be able to continue using the procedure if he/she can demonstrate that it is job related and a business necessity. See, e.g., the Americans with Disabilities Act and the Federal Uniform Guidelines on Employee Selection Procedures.
b. Incorrect See explanation for response a.
c. Incorrect See explanation for response a.
d. Incorrect See explanation for response a.

The correct answer is: adverse impact

105
Q

When using equity theory to evaluate an employee’s motivation, you would be most interested in which of the following?
Select one:

A. the employee’s level of intrinsic motivation
B. the employee’s self-efficacy beliefs
C. the employee’s perceptions regarding the pay and benefits received by workers performing similar work duties as the employee
D. the employee’s perceptions regarding the likelihood that successful performance will be rewarded with desirable outcomes

A

Equity theory proposes that a worker compares his or her input/outcome ratio to the ratio of another worker performing the same or a similar job. When the ratios seem equal to the worker, the worker is motivated to continue performing at the same level. If the ratios are unequal, this has an effect on motivation, performance, and other job outcomes.

a. Incorrect Intrinsic motivation is not addressed by equity theory.
b. Incorrect Self-efficacy beliefs are not addressed by equity theory.
c. CORRECT This pertains to the predictions of equity theory.
d. Incorrect This sounds like one of the components of expectancy theory.

The correct answer is: the employee’s perceptions regarding the pay and benefits received by workers performing similar work duties as the employee

106
Q

An assumption underlying process consultation is that:
Select one:

A. attitude change precedes behavior change.
B. behavior change precedes attitude change.
C. motivation is a proximal and primary cause of performance.
D. motivation and performance have a reciprocal relationship.

A

Process consultation is an organizational development (OD) strategy that, as its name suggests, focuses on organizational processes - or, more specifically, the social processes that are a normal part of the job.

a. Incorrect This is the opposite of what is true. A distinguishing characteristic of process consultation is its assumption that behavior change precedes attitude change.
b. CORRECT Process consultation focuses on identifying and altering the overt behaviors that are interfering with normal social processes. It is based on the assumption that behavior change is the priority and precedes attitude change.
c. Incorrect This is not a focus (or prediction) of process consultation.
d. Incorrect This is not a focus (or prediction) of process consultation.

The correct answer is: behavior change precedes attitude change.

107
Q

Job applicants complain that the biographical information blank (BIB) they are required to complete includes many items that seem entirely unrelated to their ability to do the job and argue that these items are an invasion of privacy. The concerns of these applicants suggest that the BIB lacks:
Select one:

A. face validity.
B. construct validity.
C. social validity.
D. predictive validity.

A

The BIB items do not seem to be measuring what they are intended to measure - in other words, they are lacking “face validity.”

a. CORRECT Face validity is not a type of validity in the same sense that content, construct, and criterion-related validity are (it is not empirically- or theoretically-determined). However, face validity is often desirable because it maximizes the motivation and interest of applicants or examinees.
b. Incorrect See explanation for response a.
c. Incorrect See explanation for response a.
d. Incorrect See explanation for response a.

The correct answer is: face validity.

108
Q

Herzberg’s two-factor theory classifies job security, pay and benefits, relationships with co-workers, working conditions, and company policies as:
Select one:

A. quality factors.
B. quantity factors.
C. motivator factors.
D. hygiene factors.

A

The “two factors” in Herzberg’s two-factor theory are hygiene factors and motivator factors. According to Herzberg, hygiene factors contribute to dissatisfaction when they’re inadequate but do not contribute to satisfaction or motivation. In contrast, motivator factors do not contribute to dissatisfaction but contribute to satisfaction and motivation when they’re adequate.

a. Incorrect See explanation for response d.
b. Incorrect See explanation for response d.
c. Incorrect See explanation for response d.
d. CORRECT As described by Herzberg, hygiene factors include the factors listed in this question, while motivator factors include the nature of the work itself and opportunities for responsibility, achievement, and promotion.

The correct answer is: hygiene factors.

109
Q

The rational-economic model of decision-making views decision-makers as attempting to make:
Select one:

A. satisficing decisions.
B. bounded decisions.
C. representative decisions.
D. optimal decisions.

A

The rational-economic model assumes that decision-makers are perfect and rational.

a. Incorrect This better describes the assumption underlying the administrative model of decision-making.
b. Incorrect “Bounded rationality” is also associated with the administrative model, which does not view decision-makers as entirely perfect and rational.
c. Incorrect This is not associated with the rational-economic model.
d. CORRECT The rational-economic model assumes that decision-makers will consider all possible alternatives and choose the optimal one.

The correct answer is: optimal decisions.

110
Q

To reduce a five-year old’s nighttime fear of the dark, the most effective treatment would be:
Select one:

A. coping self-statements and positive imagery.
B. in vivo exposure with response prevention.
C. stress inoculation.
D. covert sensitization.

A

A number of cognitive-behavioral techniques have been used to alleviate anxiety, fears, and other maladaptive behaviors in children and adults. Obviously, some techniques are less appropriate for children than others.

a. CORRECT Several studies have confirmed that coping skills training–which incorporates positive self-statements and positive imagery–is a useful technique for eliminating nighttime fear of the dark.
b. Incorrect This would not be a useful technique for young children.
c. Incorrect Although certain aspects of stress inoculation might be useful with young children, this technique in its entirety is better suited for adults.
d. Incorrect Covert sensitization is a type of aversive counterconditioning and would not be useful for this purpose.

The correct answer is: coping self-statements and positive imagery.

111
Q

Which aspect of long-term memory contains memories of one’s personal experiences?
Select one:

A. procedural
B. episodic
C. semantic
D. implicit

A

Long-term memory can be divided into two types - procedural and declarative - and declarative memory can be further divided into semantic and episodic memory.

a. Incorrect Procedural memory contains memories of skilled responses and actions (e.g., riding a bicycle).
b. CORRECT Episodic (autobiographical) memory consists of memories for personal events (e.g., first date, graduation from college).
c. Incorrect Semantic memory contains memories of general knowledge.
d. Incorrect Implicit memory refers to memories that are recalled without conscious effort.

The correct answer is: episodic

112
Q
When a person with an optimistic attribution style does poorly on an exam in a class he ordinarily does well in, that person will most likely say which of the following?
Select one:

A. I didn’t study hard enough.
B. I was unlucky.
C. The teacher gave a very hard test this time.
D. The teacher always grades on the curve.

A

According to Seligman’s (1990) theory of learned optimism, the attributions of optimistic people are just the opposite of those made by depressed and other pessimistic people.

a. Incorrect This is an internal attribution and is not consistent with Seligman’s theory.
b. Incorrect This isn’t as good of a response as response c.
c. CORRECT Of the responses given, this one fits best with Seligman’s theory which proposes that optimistic people tend to make external, specific, and unstable (temporary) attributions for negative events.
d. Incorrect This is a global (rather than specific) attribution.

The correct answer is: The teacher gave a very hard test this time.

113
Q

Thermal (temperature) biofeedback would be most effective as a treatment for:
Select one:

A. hyperventilation.
B. Raynauds disease.
C. stuttering.
D. TMJ.

A

Thermal (temperature) biofeedback provides feedback on skin temperature, which indicates changes in blood-flow.

a. Incorrect Respiration biofeedback is useful for alleviating hyperventilation.
b. CORRECT Raynaud’s disease is a disorder of the blood vessels that limits circulation to certain areas of the body (usually the fingers and toes). It is triggered by cold or emotional stress and causes a change in color of the skin with coldness and numbness followed by throbbing, tingling, or stinging pain upon warming or relief of the stress. Thermal biofeedback has been found to be an effective treatment for Raynaud’s disease and for a number of other disorders including hypertension and migraine headache.
c. Incorrect Galvanic skin response (GSR) training has been found useful for treating stuttering.
d. Incorrect EMG biofeedback has been found to be effective for treating temporomandibular joint (TMJ) syndrome.

The correct answer is: Raynauds disease.

114
Q

If an unconditioned stimulus is repeatedly presented just prior to a conditioned stimulus in order to replace an undesirable response with a more desirable one, which of the following will most likely occur?
Select one:

A. The target response will eventually be eliminated.
B. The target response will be suppressed (but not eliminated).
C. The target response will paradoxically increase.
D. The target behavior will not change in frequency.

A

A careful reading of this question reveals that it is describing “backwards conditioning.”

a. Incorrect See explanation for response d.
b. Incorrect See explanation for response d.
c. Incorrect See explanation for response d.
d. CORRECT Backwards conditioning rarely works: For classical conditioning to be effective, the conditioned stimulus must be presented before the unconditioned stimulus.

The correct answer is: The target behavior will not change in frequency.

115
Q

Prospective memory:
Select one:

A. is an aspect of working memory.
B. contains memories of one’s personal experiences.
C. refers to “remembering to remember.”
D. refers to “knowing about knowing.”

A

Prospective memory refers to the ability to “remember to remember” (e.g., to remember an appointment in the future).

a. Incorrect Working memory is an aspect of short-term memory.
b. Incorrect This refers to episodic memory.
c. CORRECT Remembering that you have a doctor’s appointment on Thursday at 3 p.m. depends on prospective memory.
d. Incorrect See explanation above.

The correct answer is: refers to “remembering to remember.”

116
Q

John Watson produced a phobia in Albert B. by using which of the following procedures?
Select one:

A. physically confining the child so that he could not move
B. requiring the child to make difficult stimulus discriminations
C. pairing an aversive US with a neutral CS
D. providing punishment noncontingent on behavior

A

John Watson applied Pavlov’s classical conditioning model to human behavior. In his most famous study, Watson taught Albert B., an 11-month-old child, to fear a white rat.

a. Incorrect Albert B.’s phobia was established through classical conditioning and, thus, did not involve confining him.
b. Incorrect Pavlov showed that this procedure produced “experimental neurosis”; it is not the procedure used by Watson.
c. CORRECT Albert B.’s phobia was created by pairing a US (loud noise) with a CS (white rat) so that the CS alone eventually elicited a startle response.
d. Incorrect Watson’s research with Albert did not involve the application of punishment. The use of punishment to decrease a response is an operant conditioning technique.

The correct answer is: pairing an aversive US with a neutral CS

117
Q

For the treatment of hypertension, biofeedback:
Select one:

A. is generally ineffective.
B. is effective only when combined with medication.
C. is about equally as effective as relaxation training or self-monitoring of blood pressure.
D. is more effective than relaxation training or self-monitoring of blood pressure.

A

The research on biofeedback is far from consistent, but biofeedback does appear to have positive effects for several problems including hypertension.

a. Incorrect See explanation for response c.
b. Incorrect See explanation for response c.
c. CORRECT The research has shown that, for most disorders that it has been applied to, biofeedback has beneficial effects, but these effects do not exceed those associated with other treatments. For hypertension, biofeedback seems to be about as effective as relaxation training and self-monitoring.
d. Incorrect See explanation for response c.

The correct answer is: is about equally as effective as relaxation training or self-monitoring of blood pressure.

118
Q

Bob, Al, Sally, and Jane all study a list of nonsense syllables for 30 minutes. Afterwards, Bob takes a nap, Al goes to a movie, Sally studies arithmetic, and Jane goes grocery shopping. Based on your familiarity with the research on memory, you would predict that, when Bob, Al, Sally, and Jane are asked to recall the list of syllables, ________ will recall the most syllables?
Select one:

A. Bob
B. Al
C. Sally
D. Jane

A

In an early evaluation of the trace decay theory of forgetting, Jenkins and Dallenbach (1924) had subjects who had memorized a list of nonsense syllables either sleep or stay awake before being asked to recall the syllables. Those who slept recalled more syllables.

a. CORRECT The results of the Jenkins and Dallenbach study suggest that Bob, who slept after studying the list of syllables, will exhibit the greatest recall.
b. Incorrect See explanation for response a.
c. Incorrect See explanation for response a.
d. Incorrect See explanation for response a.

The correct answer is: Bob

119
Q

According to Miller (1956), the average number of “chunks” of information that can be retained in short-term memory is:
Select one:

A. 5 and 2
B. 5 to 9 (7 +/- 2)
C. 9 and 2
D. 11 and 2

A

Answer B is correct. Short-term memory is very limited in terms of duration and capacity, but the latter can be expanded by “chunking” information into meaningful units. Miller proposed that short-term memory can hold between 5 and 9 units of information and that the ability to hold larger amounts of information is due to chunking information into groups of related items.

The correct answer is: 5 to 9 (7 +/- 2)

120
Q

According to _______________, depression is attributable to deficits in self-monitoring, self-evaluation, and self-reinforcement.
Select one:

A. Bandura’s self-efficacy model
B. Rehm’s self-control theory
C. Seligman’s learned helplessness model
D. Swann’s self-verification theory

A

There are several models of depression, but only one identifies all three of the components listed in the question as the factors underlying this disorder.

a. Incorrect Bandura’s self-efficacy model has not been widely applied to the understanding or treatment of depression. However, it implies that depression and other undesirable or maladaptive behaviors are due to a low sense of self-efficacy.
b. CORRECT According to Rehm’s self-control theory, individuals with depression selectively attend to negative events and to the immediate consequences of their behavior; make inaccurate internal attributions about their behaviors; and have low rates of self-reinforcement.
c. Incorrect Seligman’s learned helplessness model attributes depression to prior exposure to uncontrollable negative events, which leads to a sense of helplessness.
d. Incorrect Self-verification theory is not a theory of depression. It predicts that people tend to seek information that confirms the self-concept.

The correct answer is: Rehm’s self-control theory

121
Q

A researcher interested in retention of information in short-term memory shows participants a list of five letters (“C-S-J-Z-M”) and asks them to memorize the list. She then instructs the participants to count backwards by 3’s from 99. After six seconds, 12 seconds, and 18 seconds, the researcher asks the participants to recall the list. The reason why participants are instructed to count backwards is:
Select one:

A. to increase the effects of retroactive interference.
B. to deter the “trace decay” process.
C. to keep them from rehearsing the list of letters.
D. to maximize their concentration.

A

Information is retained in short-term memory for a very short period unless it is rehearsed or repeated.

a. Incorrect Retroactive interference occurs when recently learned material interferes with the recall of previously learned material, and it is most likely to occur when recently and previously learned material are similar. Counting backwards is not similar to memorizing letters, so counting backwards will not increase the effects of retroactive interference.
b. Incorrect Trace decay theory proposes that memories fade over time. Counting backwards wouldn’t affect the process of trace decay.
c. CORRECT Having participants count backwards ensures that they do not rehearse the list of letters so that an accurate estimate of the duration of short-term memory can be ascertained.
d. Incorrect Having participants count backwards may or may not improve their concentration, so this is not the best answer.

The correct answer is: to keep them from rehearsing the list of letters.

122
Q

Dr. Jay tells his client, Sandy Y., to smoke cigarettes only at 10:00 a.m., 1:00 p.m., and 8:00 p.m., to smoke only while sitting in particular chairs at home and in the office, and to smoke only a certain brand of cigarettes (one that Sandy doesn’t particularly like). Dr. Jay is using the behavioral technique known as:
Select one:

A. self-monitoring.
B. stimulus control.
C. overcorrection.
D. response cost.

A

A number of operant techniques are used to reduce the frequency and/or strength of a behavior. One of them, stimulus control, involves stimulus discrimination, or learning to respond differently in the presence of different stimuli. When a behavior is under the control of stimuli in this manner, the behavior is said to be under stimulus control.

a. Incorrect Self-monitoring involves having a client record information about the frequency and conditions of a target behavior. For Sandy Y., self-monitoring might be initially used (i.e, she might record information about where and when she smokes) in order to determine what kind of program would be most effective for reducing her smoking.
b. CORRECT Sandy Y. has been instructed to limit (control) the number and kind of stimuli that are associated with cigarette smoking. By doing so, smoking will become associated with (under the control of) a restricted number of stimuli.
c. Incorrect Overcorrection involves applying a penalty following an undesirable behavior and then having the individual practice more appropriate behaviors in an exaggerated fashion. It is most useful for the treatment of socially inappropriate and self-injurious behaviors.
d. Incorrect Response cost involves the removal of a specific positive reinforcement each time a behavior is performed. If response cost were to be used with Sandy Y., she might have to give up an hour of watching television each time she smokes.

The correct answer is: stimulus control.

123
Q

The three overlapping stages of Meichenbaum and Jaremko’s (1982) stress inoculation training are:
Select one:

A. formulation, problem focus, and termination.
B. self-monitoring, self-evaluation, and self-reinforcement.
C. cognitive modeling, overt instruction, and covert instruction.
D. conceptualization, skills acquisition, and application.

A

Stress inoculation training was designed to help people cope more effectively with stress by increasing their coping skills.

a. Incorrect These are not the three stages of stress inoculation training.
b. Incorrect These are the three targets of Rehm’s self-control therapy.
c. Incorrect These are steps in Meichenbaum’s self-instructional training.
d. CORRECT Unfortunately, the names given to the three stages vary somewhat in the literature, so you need to be flexible when looking for the correct answer to a question on this technique. The first stage of stress inoculation training is ordinarily referred to as the conceptualization, education, or cognitive phase; the second stage as the skills application or training phase or the skills acquisition and rehearsal phase; and the third phase as the application or application and follow-through phase.

The correct answer is: conceptualization, skills acquisition, and application.

124
Q

A parent is concerned about her 6-year old son’s thumb sucking. After reading several articles on behavioral techniques for eliminating self-reinforcing behaviors, she decides to spend an hour a day with her son using a strategy recommended in one of the articles. It involves setting a timer for 5 minutes and, during a one-hour period, giving her son a quarter for each 5-minute period that he does not suck his thumb but instead plays with the toys she has made available for him. The mother is using which of the following strategies?
Select one:

A. differential reinforcement
B. time-out
C. overcorrection
D. shaping

A

In this situation, the mother is using a behavioral technique to eliminate an undesirable behavior.

a. CORRECT Differential reinforcement involves reducing an undesirable self-reinforcing behavior by providing a reinforcer after each predefined interval of time that the individual does not engage in that behavior but, instead, engages in other (different) behaviors.
b. Incorrect Time-out involves removing the individual from all sources of positive reinforcement for a period of time following an undesirable behavior to eliminate that behavior.
c. Incorrect Overcorrection involves having the individual correct the consequences of an undesirable behavior and/or practice corrective (alternative) behaviors.
d. Incorrect Shaping involves teaching a new behavior by reinforcing successive approximations to that behavior.

The correct answer is: differential reinforcement

125
Q

Every time a child in an institutional setting has a tantrum, the only staff member with whom the child has a positive relationship calmly but firmly tells the child to stop his behavior. The child’s behavior increases rather than decreases over time as a result of the staff member’s behavior. This is an example of:
Select one:

A. positive reinforcement of the misbehavior.
B. negative reinforcement of the misbehavior.
C. positive punishment for the misbehavior.
D. negative punishment for the misbehavior.

A

While the staff member’s intention is to decrease the boy’s behavior, she is actually increasing it. This is apparently because the desirable, reinforcing properties of the staff member’s presence and attention are stronger than any aversive effect of what she is saying to him.

a. CORRECT Positive reinforcement involves the application of a stimulus following a behavior in order to increase that behavior. Here, the staff member’s presence is reinforcing the child’s misbehavior. Often, parents who find that their attempts at punishment fail to decrease a behavior are unaware that the punishment actually contains reinforcing properties, such as attention.
b. Incorrect Negative reinforcement is the removal of a stimulus following a behavior in order to increase that behavior. In this case, a stimulus (the staff member’s attention) is being applied rather than removed.
c. Incorrect Punishment is used to reduce or eliminate a behavior. In this case, the child’s tantrums are occurring more often, and the staff member’s behavior is apparently reinforcing rather than punishing.
d. Incorrect See explanation for response c.

The correct answer is: positive reinforcement of the misbehavior.

126
Q

Interoceptive exposure involves:
Select one:

A. exposure in imagination to objects or situations that evoke anxiety.
B. exposure to bodily sensations associated with anxiety reactions.
C. prolonged continuous exposure to a feared stimulus in vivo.
D. graded (graduated) exposure to a feared stimulus first in imagination, then in vivo.

A

Interoceptive exposure has been found useful for reducing anxiety associated with panic attacks, PTSD, and other anxiety-related disorders.

a. Incorrect See explanation for response b.
b. CORRECT Interoceptive exposure involves exposing the individual to internal bodily sensations (e.g., elevated heart rate, hyperventilation) associated with a panic attack or other anxiety response by having him/her inhale carbon dioxide, spin in a chair, etc.
c. Incorrect See explanation for response b.
d. Incorrect See explanation for response b.

The correct answer is: exposure to bodily sensations associated with anxiety reactions.

127
Q

An “extinction burst” occurs when:
Select one:

A. a primary reinforcer loses its reinforcing value.
B. a previously non-reinforced behavior is reinforced.
C. reinforcement for a previously reinforced behavior is removed.
D. an extinguished behavior is accidentally reinforced.

A

In operant conditioning (which is apparently being asked about in this question since all of the answers mention reinforcement), extinction occurs when reinforcement for a previously reinforced behavior is removed.

a. Incorrect See explanation for response c.
b. Incorrect See explanation for response c.
c. CORRECT This is the only response that describes the extinction process. The term “extinction burst” refers to the temporary increase in behavior that occurs when reinforcement is removed from a previously reinforced behavior.
d. Incorrect See explanation for response c.

The correct answer is: reinforcement for a previously reinforced behavior is removed.

128
Q

Which of the following intermittent schedules of reinforcement produces the highest rate of responding and the greatest resistance to extinction?
Select one:

A. fixed interval
B. fixed ratio
C. variable interval
D. variable ratio

A

Knowing the schedule of reinforcement of slot machines may have helped you identify the correct answer to this question.

a. Incorrect See explanation for response d.
b. Incorrect See explanation for response d.
c. Incorrect See explanation for response d.
d. CORRECT When using a variable ratio schedule, a reinforcer is provided after a variable number of responses. Of the four intermittent schedules, it is associated with the highest rate of responding and the greatest resistance to extinction once the reinforcer is no longer provided.

The correct answer is: variable ratio

129
Q

Fourteen-year-old Kevin Kendall frequently curses when he’s with his friends because they respond positively when he does so. However, Kevin never uses those words when he’s at home with his family because his parents and siblings become very upset when he uses them. Kevin’s differential use of curse words in different settings illustrates the concept of:
Select one:

A. stimulus control.
B. partial reinforcement.
C. shaping.
D. overcorrection.

A

In the situation described in this question, Kevin has learned to swear in some situations but not in others because of the different consequences for doing so in those situations.

a. CORRECT A response is brought under stimulus control when the person learns to respond in situations in which reinforcement is likely but not in situations in which no reinforcement (or punishment) is likely.
b. Incorrect Partial reinforcement refers to the use of an intermittent (partial) schedule of reinforcement.
c. Incorrect Shaping is a type of reinforcement in which successive approximations to the desired response are reinforced.
d. Incorrect Overcorrection is a behavioral technique that is used to reduce an undesirable behavior and replace it with a more desirable one.

The correct answer is: stimulus control.

130
Q

A 2-1/2 year old whines until his mother picks him up. In this situation, the mother’s behavior is being controlled by which of the following?
Select one:

A. positive reinforcement
B. negative reinforcement
C. positive punishment
D. negative punishment

A

In this situation, the mother’s “picking-up-her-child” behavior occurs because it is reinforced by the cessation of her child’s whining.

a. Incorrect Positive reinforcement occurs when a behavior increases because of the stimulus that is applied following the behavior.
b. CORRECT The mother’s behavior increases (“reinforcement”) because of the stimulus that is removed following the behavior (“negative”).
c. Incorrect Positive punishment occurs when a behavior decreases because of the application of a stimulus following the behavior.
d. Incorrect Negative punishment occurs when a behavior decreases because of the removal or cessation of a stimulus following the behavior.

The correct answer is: negative reinforcement

131
Q

A psychologist tells the parents of an 8-year old to give him a “time-out” each time he torments his little sister. The parents find that, over time, the child needs less and less time in the time-out to calm down and apologize for his behavior. The boy’s behavior is being controlled by which of the following?
Select one:

A. negative punishment
B. negative reinforcement
C. stimulus control
D. stimulus fading

A

Time-out involves removing the child from all sources of positive reinforcment for a prespecified period of time following a behavior in order to reduce that behavior.

a. CORRECT Negative punishment involves taking a stimulus away following a behavior in order to decrease that behavior, which is what occurs in time-out.
b. Incorrect Negative reinforcement involves taking a stimulus away following a behavior in order to increase the behavior.
c. Incorrect A behavior is brought under stimulus control when it is elicited by the presence of a cue or prompt.
d. Incorrect Fading refers to the gradual removal of prompts.

The correct answer is: negative punishment

132
Q

Research by Diana Baumrind found that children and adolescents whose parents were _____________ were most likely to be antisocial, use drugs and alcohol, and achieve low scores on cognitive tests.
Select one:

A. low in control and low in warmth
B. low in control and high in warmth
C. high in control and low in warmth
D. high in control and high in warmth

A

D. Baumrind distinguished between four parenting styles based on level of control and warmth. See the Lifespan Development chapter of the written study materials for additional information about these styles.

a. CORRECT As defined by Baumrind, REJECTING/NEGLECTING parents are low in control and low in warmth. Their children have the worst outcomes - e.g., they are antisocial, use drugs and alcohol, and score low on cognitive tests.
b. Incorrect PERMISSIVE/LAISSEZ FAIR parents are low in control and high in warmth. Their children also have poor outcomes but not as poor as those of children of rejecting/neglecting parents. They are immature, impulsive, and self-centered and low in achievement-orientation.
c. Incorrect AUTHORITARIAN parents are high in control and low in warmth. Their children tend to be aggressive, mistrusting, and dependent and have low levels of self-esteem and academic achievement.
d. Incorrect AUTHORITATIVE parents are high in control and warmth. Their children have the best outcomes. They are mature, resilient, achievement-oriented, and responsible, and do well in school.

The correct answer is: low in control and low in warmth

133
Q

In a research study, a one-year old is placed on the shallow side of a visual cliff, while the baby’s mother stands on the deep side. On alternate trials, the mother is instructed to smile at her infant or to show a fearful face. The baby’s different responses to his/her mother’s facial expressions will be reflective of:
Select one:

A. attachment.
B. empathy.
C. impulsivity.
D. temperament

A

This is a difficult question since the visual cliff is probably most associated with the study of depth perception in infants.

a. CORRECT The visual cliff has also been used to study attachment and other developmental phenomena. The mother’s behavior is a variable in this study (she either smiles or makes a fearful face). If attachment has developed, the mother’s facial expression will serve as a signal of safety or danger to the infant. This is referred to as SOCIAL REFERENCING.
b. Incorrect See explanation for response a.
c. Incorrect See explanation for response a.
d. Incorrect See explanation for response a.

The correct answer is: attachment.

134
Q

Which of the following is true regarding skeletal age?
Select one:

A. Girls are behind boys in terms of skeletal age from birth, which accounts for their shorter height in adulthood.
B. Girls are ahead of boys in terms of skeletal age from birth, which explains why girls reach their full height before boys do.
C. Boys and girls are similar in terms of skeletal age until puberty, when the skeletal age of boys begins to increase more rapidly.
D. Boys and girls are similar in terms of skeletal age at all ages, which confirms that skeletal age is unrelated to height in adulthood.

A

Skeletal age is a measure of bone maturation and is determined through X-rays of the long bones of the body, which provide information on the degree to which ossification (hardening of soft tissues into bones) has occurred.

a. Incorrect See explanation for response b.
b. CORRECT Girls are ahead of boys in terms of skeletal age from birth, and this gap increases throughout infancy in childhood and explains why girls reach their full height before boys do. See, e.g., L. E. Berk, Development through the lifespan, 2004, Boston, Pearson Education.
c. Incorrect See explanation for response b.
d. Incorrect See explanation for response b.

The correct answer is: Girls are ahead of boys in terms of skeletal age from birth, which explains why girls reach their full height before boys do.

135
Q

A child in the preoperational stage of development:
Select one:

A. carefully follows the rules when playing a game.
B. views death as a “bogeyman” that can be outwitted.
C. thinks objects “disappear” when they have been hidden from view.
D. treats objects as symbols of things (e.g., dolls as babies).

A

According to Piaget, a child passes through four stages of cognitive development:

SENSORIMOTOR
PREOPERATIONAL
CONRETE OPERATIONAL
FORMAL OPERATIONAL

See the Lifespan Development chapter for additional information about these stages.

a. Incorrect This is more characteristic of the concrete operational stage.
b. Incorrect This usually doesn’t occur until around age 7, when the child is in the concrete operational stage.
c. Incorrect This is characteristic of the sensorimotor period of development.
d. CORRECT The manipulation of symbols is an essential characteristic of the preoperational stage of development.

The correct answer is: treats objects as symbols of things (e.g., dolls as babies).

136
Q

According to Kohlberg’s cognitive theory, gender identity development involves three stages that occur in the following order:
Select one:

A. stability, constancy, identity
B. constancy, stability, identity
C. identity, constancy, stability
D. identity, stability, constancy

A

Kohlberg’s cognitive model distinguishes between three stages of gender identity development: identity, stability, and constancy.

a. Incorrect See explanation for response d.
b. Incorrect See explanation for response d.
c. Incorrect See explanation for response d.
d. CORRECT According to Kohlberg, a child recognizes that he/she is a boy/girl by age three (gender identity). Subsequently the child realizes that gender is stable over time and then, by age seven, realizes that gender is constant across different situations.

The correct answer is: identity, stability, constancy

137
Q

Soon after birth, infants express, through facial expression:
Select one:

A. discomfort, interest, and surprise.
B. distress, interest, and disgust.
C. joy, disgust, and fear.
D. joy, anger, and surprise.

A

Parents and researchers agree that infants exhibit a variety of emotions through their facial expressions.

a. Incorrect See explanation for response b.
b. CORRECT By 3 to 4 months of age, infants express at least four emotions through their facial expressions – i.e., interest, sadness, distress, and disgust. Anger, joy, surprise, and fear do not emerge until about 6 to 8 months.
c. Incorrect See explanation for response b.
d. Incorrect See explanation for response b.

The correct answer is: distress, interest, and disgust.

138
Q

Children’s understanding of race as a physical, social, and biological category is usually first evident by ____ years of age.
Select one:

A. 4
B. 6
C. 10
D. 13

A

Although children show awareness of racial differences by age 3 or 4, a sophisticated understanding of race does not develop until later.

a. Incorrect See explanation for response c.
b. Incorrect See explanation for response c.
c. CORRECT This is the finding reported by M. N. Alejandro-Wright for both White and African-American children, and it is consistent with the work of other investigators (The children’s conception of racial classification: A socio-cognitive developmental model, in M. B. Spencer et al. (Eds.), Beginnings: The social and affective development of Black children, Hillsdale, NJ, Erlbaum, 1985).
d. Incorrect See explanation for response c.

The correct answer is: 10

139
Q

The most prominent risk factor for drug abuse in adolescence is:
Select one:

A. parent and family member drug use.
B. availability of drugs.
C. prior use of gateway drugs.
D. depression.

A

Not surprisingly, past behavior is the best predictor of future behavior.

a. Incorrect Although drug use by parents or other family members is a good predictor of adolescent drug abuse, it’s not as accurate as prior drug use.
b. Incorrect This also is not as strong of a predictor as prior drug use.
c. CORRECT Use of tobacco, alcohol, and/or marijuana (which are referred to as “gateway” drugs in the literature) is the single-best predictor of illicit drug use and drug abuse by adolescents. See M. A. Pentz, Primary prevention of adolescent drug abuse, in C. B. Fisher and R. M. Lerner (Eds.), Applied developmental psychology, McGraw-Hill, New York, 1994.
d. Incorrect Depression is not as predictive as previous drug use.

The correct answer is: prior use of gateway drugs.

140
Q

Tom is not colorblind but he has a brother who is colorblind. Tom’s wife, Alice, is not colorblind and does not have any relatives who are colorblind. Which of the following describes the likelihood that the children of Tom and Alice will be colorblind?
Select one:

A. Their male children only are likely to be colorblind.
B. Their female children only are likely to be colorblind.
C. Their male and female children may or may not be colorblind.
D. Neither their male nor their female children will be colorblind.

A

To identify the correct answer to this question, you have to be aware that colorblindness is a recessive trait that is carried on the X chromosome.

a. Incorrect For a son to be colorblind, he would have to inherit the colorblind gene from his mother. This will not occur in this case since there is no history of colorblindness in Alice’s family.
b. Incorrect For a daughter to be colorblind, she would have to inherit the colorblind gene from her father and her mother. Since there’s no history of colorblindness in Alice’s family, this would not occur. (However, a daughter would carry the trait if she inherits the gene from her father.)
c. Incorrect See explanation for answers a and b.
d. CORRECT Since the gene for colorblindness is carried on the X chromosome, colorblindness would have to come from the mother for sons and from both the mother and father for daughters. However, since there’s no history of colorblindness in the mother’s family, this will not occur.

The correct answer is: Neither their male nor their female children will be colorblind.

141
Q

In Ainsworth’s “strange situation,” a one-year old shows little distress when her mother leaves the room and ignores her when she returns. Most likely, this mother is:
Select one:

A. neglectful or physically abusive.
B. depressed.
C. impatient or overstimulating.
D. a single parent.

A

The child in the question is exhibiting insecure/avoidant attachment.

a. Incorrect Abuse and neglect are most associated with the disorganized/disoriented attachment pattern.
b. Incorrect Maternal depression may lead to insecure attachment, but you can’t draw this conclusion from the information given (i.e., the mother may or may not be depressed).
c. CORRECT Research by Ainsworth and her colleagues found that babies exhibiting insecure/avoidant attachment often have mothers who tend to be either very impatient and nonresponsive or, alternatively, overstimulating.
d. Incorrect Single parenthood has not been linked to the insecure/avoidant attachment pattern.

The correct answer is: impatient or overstimulating.

142
Q

As described by Piaget, a tertiary circular reaction occurs when an infant:
Select one:

A. repeats an interesting or enjoyable action that involves his/her own body.
B. deliberately varies an action to discover or observe the consequences of that action.
C. attempts to reproduce a pleasurable action that involves another person or an object.
D. combines cognitive schemes into more complex action sequences.

A

Piaget defined circular reactions as the behaviors that are responsible for the development and modification of cognitive schemas during the sensorimotor stage. He described these reactions as involving six substages.

a. Incorrect This answer describes primary circular actions (substage 2).
b. CORRECT This answer accurately describes a tertiary circular reaction which can be viewed as “miniature experiments” that involve engaging in specific actions to observe their consequences (substage 5).
c. Incorrect This answer describes secondary circular actions (substage 3).
d. Incorrect This answer describes coordinated secondary circular reactions (substage 4).

The correct answer is: deliberately varies an action to discover or observe the consequences of that action.

143
Q

Data collected by the U. S. Department of Health and Human Services has shown that the perpetrators of child abuse and neglect are most often:
Select one:

A. a daycare provider or babysitter
B. an unmarried partner of a parent
C. a parent
D. a relative other than a parent

A

Data on child maltreatment are collected by the National Child Abuse and Neglect System (NCANDIS) from child protective services in all 50 states, the District of Columbia, and Puerto Rico.

a. Incorrect See explanation for answer c.
b. Incorrect See explanation for answer c.
c. CORRECT The 2012 data revealed that 80.3% of the perpetrators of child maltreatment were parents, with 88.5% of the parents being biological parents (U. S. Department of Health and Human Services, Child Maltreatment: 2012, Washington, DC, U. S. Government Printing Office, 2012).
d. Incorrect See explanation for answer c.

The correct answer is: a parent

144
Q

When the sole of an infant’s foot is stroked, her big toe bends back toward the top of her foot and her other toes fan out. This is referred to as the _______ reflex.
Select one:

A. root
B. palmar
C. Moro
D. Babinski

A

Newborns exhibit a number of reflexes that disappear within the first year of life. For the exam, you want to be familiar with the four reflexes listed in the answers to this question.

a. Incorrect The root reflex (rooting) occurs when the corner of a baby’s mouth is touched and, in response, the baby turns his/her head in the direction of the touch and opens his/her mouth.
b. Incorrect As its name implies, the palmar reflex (also known as the palmar grasp reflex) occurs when an object is placed in the palm of an infant’s hand or his/her palm is stroked.
c. Incorrect The Moro reflex is also known as the startle reflex and occurs when, in response to a loud noise or sudden movement, the baby throws his/her head back, extends his/her arms and legs, and then brings his/her arms and legs inward.
d. CORRECT The Babinski reflex is also referred to as the plantar reflex and is one of a few early reflexes that does not seem to serve a function. It disappears at about 8 to 12 months of age.

The correct answer is: Babinski

145
Q

A number of researchers have attempted to determine the conditions associated with positive outcomes for children who were “at risk” at birth and during early infancy. Their studies suggest that children who recover from early deficiencies:
Select one:

A. have higher-than-average levels of intelligence.
B. exhibit good social responsiveness during infancy.
C. are placed in special education classes on entering school.
D. have mothers who do not work.

A

One of the most commonly-cited studies on the outcomes for at-risk children is the longitudinal study by E. Werner and R. Smith (Vulnerable but invincible, New York, McGraw-Hill, 1982). That study and others have identified several factors that predict positive outcomes for high risk (vulnerable) children.

a. Incorrect While significantly lower-than-average IQs are associated with a poor outcome, many children with average IQs have a positive outcome.
b. CORRECT Children with positive outcomes tend to be described as very active and socially responsive during infancy, and many studies have found that a positive outcome is likely to the extent that the child is able to successfully elicit attention and appropriate care from his/her caretakers.
c. Incorrect Learning disabilities (with or without special education) are associated with poorer outcomes.
d. Incorrect Many of the mothers in the Werner and Smith study worked outside the home. However, this did not seem to have a negative effect since the children often had close relationships with their grandmothers and/or siblings who acted as caretakers.

The correct answer is: exhibit good social responsiveness during infancy.

146
Q

As described by __________, internalization involves an internal reconstruction of an external operation.
Select one:

A. Piaget
B. Erikson
C. Vygotsky
D. Elkind

A

This is a difficult question, but you may have been able to identify the correct answer if you remembered that Vygotsky viewed cognitive development as being initially interpersonal (which refers to the child’s interactions with others) and then intrapersonal (which occurs when the child internalizes what he or she has learned).

a. Incorrect See explanation for response c.
b. Incorrect See explanation for response c.
c. CORRECT Internalization is a key concept in Vygotsky’s theory of cognitive development. For example, he described language development as a process involving a transition from social (external) speech to private (self-directed) speech to internal speech. In other words, speech first serves a social function (social speech); is then used to direct, plan, and evaluate one’s own actions (private speech); and then acts as a tool for thinking (internal speech).
d. Incorrect See explanation for response c.

The correct answer is: Vygotsky

147
Q

In his study of moral development, Kohlberg presented research participants with stories that posed a conflict between two moral dilemmas. In the most famous of these stories, the “________ dilemma,” participants were presented with a choice between the value of obeying the law and the value of human life.
Select one:

A. Clever Hans
B. Good Boy/Good Girl
C. Hawthorne
D. Heinz

A

Kohlberg’s theory of moral development focuses on how individuals reason about moral dilemmas.

a. Incorrect See explanation for response d.
b. Incorrect See explanation for response d.
c. Incorrect See explanation for response d.
d. CORRECT The Heinz dilemma required research participants to reason about a dilemma in which a man must choose between not stealing a drug and stealing the drug to save his wife’s life.

The correct answer is: Heinz

148
Q

Research investigating the effectiveness of training first-graders in the use of rehearsal strategies has found that:
Select one:

A. these children learn to apply rehearsal to the current task but do not generalize this knowledge to other tasks.
B. these children learn to apply rehearsal to the current task and often generalize this knowledge to other tasks.
C. these children learn to apply rehearsal to the current task only when they anticipate external reinforcement for doing so.
D. these children are unable to learn to apply rehearsal to the current or future tasks.

A

The studies have consistently shown that children do not spontaneously and consistently use rehearsal until about age 9 or 10.

a. CORRECT This is the result reported, for example, by T. J. Keeney et al. (Spontaneous and induced verbal rehearsal in a recall task, Child Development, 38, 953-966, 1967). The first-graders in their study could be taught to use rehearsal for a specific task; however, they did not generalize this knowledge to other tasks.
b. Incorrect See explanation above.
c. Incorrect See explanation above.
d. Incorrect See explanation above.

The correct answer is: these children learn to apply rehearsal to the current task but do not generalize this knowledge to other tasks.

149
Q

As described by Piaget, the ability of a child to see someone else perform a behavior and subsequently perform that behavior him/herself depends on the development of which of the following abilities?
Select one:

A. mental representation
B. assimilation
C. transductive reasoning
D. centration

A

The ability to imitate the behavior of a model after a time delay of hours or days is referred to as DEFERRED IMITATION.

a. CORRECT Piaget believed that deferred imitation does not occur until about 18 months of age and depends on the ability to represent objects and actions symbolically - i.e., to form a MENTAL REPRESENTATION of the object or action.
b. Incorrect Assimilation refers to the incorporation of new information into existing knowledge. Although assimilation may be involved in the ability to imitate a model, it does not account for deferred imitation.
c. Incorrect Transductive (precausal) reasoning is one of the limitations of the preoperational stage and reflects an incomplete understanding of cause and effect.
d. Incorrect As defined by Piaget, centration is another limitation of the preoperational stage and refers to the tendency to focus on the most noticeable feature of an object.

The correct answer is: mental representation

150
Q

Based on the empirical research, the best conclusion that can be drawn about the effects of maternal depression on infant development is that children of depressed mothers:
Select one:

A. are at no higher risk for future psychopathology than children whose mothers are not depressed.
B. are at higher risk for future psychopathology although symptoms are not usually first apparent until adolescence.
C. are at higher risk for emotional and behavioral problems during the preschool years but are essentially indistinguishable from children whose mothers are not depressed in subsequent years.
D. are at higher risk for psychopathology and may show symptoms of disturbance as early as three months of age.

A

Children of depressed mothers are at higher risk for emotional and behavioral problems, although the exact nature and severity of the problems depend on several factors including early mother-child interactions.

a. Incorrect See explanation for response d.
b. Incorrect See explanation for response d.
c. Incorrect See explanation for response d.
d. CORRECT There is some evidence that signs of distress (e.g., elevated heart rate and greater right frontal lobe EEG asymmetry) are apparent by the time the infant is three months of age.

The correct answer is: are at higher risk for psychopathology and may show symptoms of disturbance as early as three months of age.

151
Q

Longitudinal research by Gerald Patterson and his colleagues has shown that parents of aggressive children rely on power assertion to enforce their standards. In addition, the discipline of these parents:
Select one:

A. is consistent and often accompanied by withdrawal of physical affection.
B. is inconsistent and often not clearly linked to the child’s behavior.
C. is inconsistent but, when applied, usually occurs immediately after the child’s misbehavior.
D. is consistent and is usually accompanied by humiliating verbal messages.

A

Patterson and his colleagues have found that parents of highly aggressive children tend to rely on physical punishment.

a. Incorrect See explanation for response b.
b. CORRECT Patterson et al. also found that parents of aggressive children frequently fail to connect their punishments to the child’s behavior, do not state clear rules, and do not consistently punish violation of rules.
c. Incorrect See explanation for response b.
d. Incorrect See explanation for response b.

The correct answer is: is inconsistent and often not clearly linked to the child’s behavior.

152
Q

Longitudinal research by the National Institute of Child Health and Human Development (NICHD) has found a positive relationship between the number of hours a young child spends in daycare and the rates of:
Select one:

A. Oppositional Defiant Disorder, Conduct Disorder, and other clinical disorders.
B. disobedience and other behavioral problems.
C. language and academic skill deficits.
D. language and academic skill deficits and behavioral problems.

A

The NICHD research has found that participation in high-quality early child care is actually associated with enhanced pre-academic (cognitive) skills and language performance at age 4-1/2. However, it is also associated with some negative consequences.

a. Incorrect The NICHD research found that children who attend daycare are not at higher risk for serious (clinical) behavioral problems.
b. CORRECT Children who attend daycare tend to exhibit a higher degree of externalizing behaviors and conflicts with adults even when quality, type, and stability of the daycare is controlled. However, these behaviors are within the normal range.
c. Incorrect As noted above, participation in daycare is associated with better pre-academic and language skills.
d. Incorrect See explanation above.

The correct answer is: disobedience and other behavioral problems.

153
Q

According to Piaget’s theory of cognitive development, emergence of the ability to create mental representations of reality occurs between the ages of:
Select one:

A. 4 to 8 months.
B. 8 to 12 months.
C. 12 to 18 months.
D. 18 to 24 months.

A

Piaget proposed that the ability to form mental representations occurs at the end of the sensorimotor stage.

a. Incorrect From 4 to 8 months of age, the child is in the SECONDARY CIRCULAR REACTIONS substage of the sensorimotor stage.
b. Incorrect From 8 to 12 months, the child is in the COORDINATED SECONDARY CIRCULAR reactions substage.
c. Incorrect From 12 to 18 months, the child is in the TERTIARY CIRCULAR reactions substage.
d. CORRECT From 18 to 24 months, the child is in the MENTAL REPRESENTATION SUBSTAGE, which is characterized by the development of the ability to form internal representations of objects and events. (Note that some recent research suggests that deferred imitation and problem solving - which depend on mental representation - occur at an earlier age than Piaget proposed.)

The correct answer is: 18 to 24 months.

154
Q

Metcalfe and Mischel (1999) use a “hot/cool” system to explain which of the following?
Select one:

A. delay of gratification
B. risk and resilience
C. aggression
D. attitude change

A

For the exam, you want to know that Mischel and colleagues are well-known for their research on delay of gratification.

a. CORRECT J. Metcalfe and W. Mischel propose that delay of gratification is related to “hot” and “cool” systems in the brain (A hot/cool system analysis of delay of gratification: Dynamics of willpower, Psychological Review, 106, 3-19, 1999). The hot system is emotional and demands immediate gratification, while the cool system is cognitive and, in certain conditions, can divert attention away from the hot (arousal-based) features of a reward, resulting in an increased ability to delay gratification. According to these investigators, the hot system develops earlier than the cool system, which explains why the ability to delay gratification increases with increasing age.
b. Incorrect See explanation for response a.
c. Incorrect See explanation for response a.
d. Incorrect See explanation for response a.

The correct answer is: delay of gratification

155
Q

When using the Adult Attachment Interview, a mother will be classified as ____________ if she describes her parents in positive terms (e.g., “Dad was a very loving person”) but either reports negative interactions with her parents or says she can’t remember any early interactions with them.
Select one:

A. dismissing
B. preoccupied
C. unresolved/disorganized
D. resistant

A

The Adult Attachment Interview categorizes respondents in terms of three main categories - autonomous, dismissing, and preoccupied.

a. CORRECT Dismissing individuals describe their parents in positive terms that are not substantiated by descriptions of actual interactions.
b. Incorrect Preoccupied individuals are confused, angry, or passively preoccupied with attachment figures.
c. Incorrect The Adult Attachment Interview includes a fourth category (unresolved/disorganized) that is assigned to respondents who have an unresolved experience related to loss or abuse. It is assigned in conjunction with one of the three categories listed above.
d. Incorrect This is not one of the scoring categories for the Adult Attachment Interview.

The correct answer is: dismissing

156
Q

Language and social interactions are the key elements in _________ theory of cognitive development.
Select one:

A. E. Gibson’s
B. L. Vygotsky’s
C. W. Perry
D. M. Rutter

A

Only one of the theorists listed emphasized the roles of both language and social interactions in cognitive development.

a. Incorrect Eleanor Gibson is best known for her studies on perceptual development in infants.
b. CORRECT Vygotsky’s sociocultural theory focuses on the impact of language (e.g., the internalization of dialogues with peers and adults) and social interactions on cognitive development. For additional information about Vygotsky’s theory, see the Lifespan Development chapter of the written study materials.
c. Incorrect Perry’s research focused on changes in cognitive development from adolescence to early adulthood, and he distinguished between dualistic and relativistic thinking.
d. Incorrect Rutter is probably best known for his work on risk and resilience.

The correct answer is: L. Vygotsky’s

157
Q

As described by Piaget, a child in the autonomous stage of moral development believes that:
Select one:

A. rule violations will be punished.
B. rules can be changed by consensus.
C. rules can be changed by authorities only.
D. rules are made to be broken.

A

Piaget distinguished between two stages of moral development–HETERONOMOUS AND AUTONOMOUS

a. Incorrect This belief is more characteristic of the heteronomous stage.
b. CORRECT Children in the autonomous stage believe that rules can be changed by consensus.
c. Incorrect This is more characteristic of the heteronomous stage.
d. Incorrect This doesn’t describe the autonomous stage as accurately as response b.

The correct answer is: rules can be changed by consensus.

158
Q

According to Erikson, successful resolution of the final stage of psychosocial development involves which of the following?
Select one:

A. identity
B. intimacy
C. wisdom
D. hope

A

The final stage in Erikson’s stages of psychosocial development involves a conflict between integrity versus despair.

a. Incorrect The development of a coherent identity is the goal of adolescence.
b. Incorrect Developing intimate relationships is a goal during young adulthood.
c. CORRECT Successful resolution of the psychosocial conflict of late adulthood (ages 65 and over) involves developing a broader philosophical and moral perspective (“wisdom”) that younger adults often lack.
d. Incorrect “Hope” is not associated with this stage.

The correct answer is: wisdom

159
Q

Based on the results of their longitudinal study of married women, Gorchoff, John, and Hebson (2008) concluded that the transition to an “empty nest” is usually accompanied by:
Select one:

A. an increase in marital satisfaction thats related to the quantity (but not quality) of time spent with their partners.
B. an increase in marital satisfaction thats related to the quality (but not quantity) of time spent with their partners.
C. a decrease in marital satisfaction thats related to the quantity (but not quality) of time spent with their partners.
D. a decrease in marital satisfaction thats related to the quality (but not quantity) of time spent with their partners.

A

The results of S. M. Gorchoff, O. P. John, and R. Helson’s research confirmed earlier studies finding that marital satisfaction tends to increase when children leave home [Contextualizing marital satisfaction during middle age: An 18-year longitudinal study, Psychological Science, 19(11), 1194-2000, 2008].

a. Incorrect See explanation for answer b.
b. CORRECT In addition to finding that women reported increased marital satisfaction when their children left home, these investigators found that the increase was due to an increase in the quality of interactions with their partners rather than the quantity of time spent with their partners.
c. Incorrect See explanation for answer b.
d. Incorrect See explanation for answer b.

The correct answer is: an increase in marital satisfaction thats related to the quality (but not quantity) of time spent with their partners.

160
Q

When working with a “split-brain” patient, you would notice that he or she has the most difficulty with regard to which of the following?
Select one:

A. reflexive functions
B. executive functions
C. memory functions
D. sensory functions

A

Split-brain patients have had their corpus callosum severed, usually as a treatment for severe epilepsy.

a. Incorrect See explanation for response d.
b. Incorrect See explanation for response d.
c. Incorrect See explanation for response d.
d. CORRECT A primary function of the corpus callosum is to transfer sensory (and some motor) information from one hemisphere to the other. When the corpus callosum is severed, messages from the right brain cannot be transferred to the left brain and vice versa, and this deficit is most apparent in tasks involving sensory input.

The correct answer is: sensory functions

161
Q

An MRI of a patient with Huntington’s disease is most likely to show atrophy in which brain structure?
Select one:

A. caudate nucleus
B. suprachiasmatic nucleus
C. septum
D. medulla

A

Huntington’s disease (chorea) is a basal ganglia disorder, and the basal ganglia are involved in the control of motor movements.

a. CORRECT The caudate nucleus and putamen are structures of the basal ganglia and have been found to be the brain areas most severely affected by Huntington’s disease.
b. Incorrect See explanation above.
c. Incorrect See explanation above.
d. Incorrect See explanation above.

The correct answer is: caudate nucleus

162
Q

Wernicke’s area is located in the:
Select one:

A. occipital lobe.
B. temporal lobe.
C. frontal lobe.
D. parietal lobe.

A

Wernicke’s area is one of the primary speech areas.

a. Incorrect The occipital lobe mediates vision and visual perception.
b. CORRECT Wernicke’s area is located in the temporal lobe. Damage to this area causes Wernicke’s (receptive) aphasia.
c. Incorrect Broca’s area (the other primary speech area) is located in the frontal lobe.
d. Incorrect The parietal lobe contains the somatosensory cortex.

The correct answer is: temporal lobe.

163
Q

Following a head injury, Jake J., age 24, exhibits a period of post-traumatic amnesia that persists for nearly one hour. Two days later, Jake is still experiencing a number of symptoms including headache, fatigue, irritability, visual disturbances, and impaired attention. Jake does not have a history of a prior head injury or psychiatric problems. In terms of prognosis, Jake can expect:
Select one:

A. recovery of all neuropsychological functions and resolution of most or all symptoms within two to four weeks.
B. recovery of most or all neuropsychological functions and resolution of most or all symptoms within three to six months.
C. significant impairment of some neuropsychological functions and the presence of other symptoms for at least nine to 12 months.
D. significant impairment of most neuropsychological functions and the presence of other symptoms for an indefinite period of time.

A

Post-traumatic amnesia (PTA) refers to the anterograde amnesia that occurs following a traumatic brain injury. The duration of PTA is considered to be a good predictor of outcomes following a brain injury, although the outcomes actually vary considerably from individual to individual.

a. Incorrect See explanation for response b.
b. CORRECT Several categorization systems are available for defining severity of a traumatic brain injury based on the duration of the PTA. However, a frequently used system identifies a duration of less than one hour as a mild brain injury. In terms of recovery, most individuals experiencing a mild brain injury recover cognitive and behavioral functions and experience a resolution of other symptoms within 3 to 6 months, although a minority of individuals continues to experience some symptoms for an extended period of time. Factors associated with an increased risk for long-term impairment include female gender, previous head trauma, and history of a neurological or psychiatric problem. See, e.g., J. Ponsford et al., Factors influencing outcome following mild traumatic brain injury in adults, Journal of the International Neuropsychological Society, 6(5), 568-579, 2000.
c. Incorrect People who experience a mild head injury usually return to premorbid (or near premorbid) levels of functioning within several months. However, those who experience a moderate or severe injury are likely to experience long-term symptoms and impairments in multiple areas of functioning. Note that the research on outcomes following traumatic brain injury has produced inconsistent results and, consequently, there is only limited agreement among the experts regarding the outcomes associated with all severity levels of brain injury.
d. Incorrect See explanation for responses b and c.

The correct answer is: recovery of most or all neuropsychological functions and resolution of most or all symptoms within three to six months.

164
Q

A patient taking the neuroleptic clozapine exhibits several symptoms including tachycardia, muscle rigidity, hyperthermia, altered consciousness, and autonomic dysfunction. The best action in this case would be to:
Select one:

A. gradually reduce the drug dose until the symptoms are alleviated.
B. immediately stop the drug and administer fluids and electrolytes.
C. switch the patient to a traditional antipsychotic drug.
D. check to see what other drugs the patient is taking since clozapine does not produce these symptoms.

A

The symptoms listed in the question are characteristic of neuroleptic malignant syndrome (NMS), which is a rare disorder that can occur with any class of neuroleptic agent.

a. Incorrect See explanation for response b.
b. CORRECT NMS can be fatal unless it is recognized early and the medication is immediately stopped.
c. Incorrect See explanation for response b.
d. Incorrect See explanation for response b.

The correct answer is: immediately stop the drug and administer fluids and electrolytes.

165
Q

An advantage of risperidone is that it:
Select one:

A. is less likely to produce extrapyramidal side effects than are the traditional antipsychotics.
B. is not associated with weight gain and sexual dysfunction.
C. does not produce neuroleptic malignant syndrome.
D. has a rapid onset of beneficial (therapeutic) effects.

A

Resperidone is classified as an atypical antipsychotic.

a. CORRECT Although extrapyramidal side effects may occur, they are much less common with resperidone than with the typical (traditional) antipsychotics.
b. Incorrect Weight gain and sexual dysfunction are common side effects of resperidone.
c. Incorrect Resperidone can produce NMS, although this side effect is rare.
d. Incorrect One problem with resperidone is that it has a delayed onset.

The correct answer is: is less likely to produce extrapyramidal side effects than are the traditional antipsychotics.

166
Q

L-dopa acts as a:
Select one:

A. dopamine agonist.
B. dopamine antagonist.
C. dopamine neurotoxin.
D. dopamine metabolite.

A

Some chemicals act as either agonists or antagonists in the brain.

a. CORRECT L-dopa is a precursor to dopamine, which means that it is transformed to dopamine in the brain. An agonist is a chemical that increases the amount or action of a neurotransmitter. Consequently, L-dopa is a dopamine agonist.
b. Incorrect An antagonist reduces the amount or availability of a neurotransmitter in the brain.
c. Incorrect See explanation for response a.
d. Incorrect See explanation for response a.

The correct answer is: dopamine agonist.

167
Q

To reduce the symptoms of tardive dyskinesia, which of the following would be most effective?
Select one:

A. a drug that decreases dopamine levels
B. a drug that increases norepinephrine levels
C. a drug that decreases GABA levels
D. a drug that increases serotonin levels

A

The cause of tardive dyskinesia (TD) seems to be complex and is not entirely understood. However, there is evidence that it is related to dopamine oversensitivity and/or GABA depletion.

a. CORRECT There is evidence that dopamine depleting drugs alleviate some symptoms of tardive dyskinesia.
b. Incorrect In fact, the research has shown the opposite - i.e., drugs that decrease norepinephrine levels suppress the symptoms of TD.
c. Incorrect Drugs that increase GABA (e.g., GABA agonists - especially benzodiazepine) have beneficial effects for many patients with TD.
d. Incorrect Research on serotonin has found that SSRIs and other serotonergic drugs are not useful as a treatment for TD.

The correct answer is: a drug that decreases dopamine levels

168
Q

Which of the following describes the correct sequence of events that occur during an action potential?
Select one:

A. Potassium (K+) enters the cell and the cell depolarizes; then sodium (Na+) leaves the cell and the cell repolarizes.
B. Potassium (K+) leaves the cell and the cell hyperpolarizes; then sodium (Na+) enters the cell and the cell repolarizes.
C. Sodium (Na+) enters the cell and the cell depolarizes; then potassium (K+) leaves the cell and the cell repolarizes.
D. Sodium (Na+) leaves the cell and the cell hyperpolarizes; then potassium enters the cell and the cell repolarizes.

A

An action potential (nerve impulse) occurs when a neuron “fires” - i.e., when an electrical signal is sent through the axon and triggers the release of a neurotransmitter from the axon terminal into the synaptic cleft.

a. Incorrect See explanation for response c.
b. Incorrect See explanation for response c.
c. CORRECT An action potential is caused by the exchange of ions across the neuron’s semipermeable membrane. Specifically, with sufficient stimulation, the cell’s sodium channels open and sodium ions rush into the cell, creating a state of depolarization (which means that the neuron has become more positively charged). Subsequently, the cell’s potassium channels open and potassium rushes out of the cell, which reverses the depolarization and causes repolarization of the cell.
d. Incorrect See explanation for response c.

The correct answer is: Sodium (Na+) enters the cell and the cell depolarizes; then potassium (K+) leaves the cell and the cell repolarizes.

169
Q

Drugs that block the activity of __________ produce dry mouth, blurred vision, postural hypotension, tachycardia, and sedation.
Select one:

A. 5-HT
B. ACh
C. glycine
D. glutamate

A

The symptoms listed in the question are collectively referred to as “anticholinergic effects.”

a. Incorrect See explanation for response b.
b. CORRECT ACh (acetylcholine) is one of the neurotransmitters classified as “cholinergic.” Drugs that block acetylcholine activity produce anticholinergic effects.
c. Incorrect Glycine is found in the spinal cord and exerts an inhibitory effect.
d. Incorrect Glutamate is associated with learning and memory and the effects of alcohol.

The correct answer is: ACh

170
Q

Recent studies suggest that, for patients who develop tardive dyskinesia as a result of long-term neuroleptic use:
Select one:

A. symptoms are always irreversible.
B. symptoms are alleviated with a dopamine agonist in most cases.
C. symptoms may eventually improve to some degree following neuroleptic withdrawal.
D. symptoms worsen over time following neuroleptic withdrawal even when withdrawal is gradual.

A

Tardive dyskinesia is a serious side effect of the antipsychotic drugs.

a. Incorrect See explanation for response c.
b. Incorrect See explanation for response c.
c. CORRECT Although early research suggests that tardive dyskinesia is irreversible, recently-published longitudinal studies indicate that, in 30 to 60% of cases, symptoms improve after five years following withdrawal of the drug. This is especially true for younger people and when the drug is withdrawn gradually. Note also that continued use of a low-dose of neuroleptics can, at least temporarily, suppress the symptoms of tardive dyskinesia.
d. Incorrect See explanation for response c.

The correct answer is: symptoms may eventually improve to some degree following neuroleptic withdrawal.

171
Q

A common migraine:
Select one:

A. begins with an aura and is usually accompanied by nausea.
B. is constant and non-throbbing and may be exacerbated by bright lights.
C. does not begin with an aura and may be exacerbated by bending over or lifting.
D. is sharp and unilateral and is usually accompanied by autonomic symptoms.

A

Migraine headaches are classified as common or classic.

a. Incorrect Classic migraines begin with an aura.
b. Incorrect Migraines involve throbbing pain.
c. CORRECT A number of things can exacerbate the pain of a migraine including lifting or bending over. In contrast to classic migraines, common migraines do not begin with an aura.
d. Incorrect This sounds more like a cluster headache than a migraine headache.

The correct answer is: does not begin with an aura and may be exacerbated by bending over or lifting.

172
Q

The complete loss of color vision resulting from a lack of functioning cone cells is referred to as:
Select one:

A. color agnosia.
B. akinetopsia.
C. prosopagnosia.
D. achromatopsia.

A

You may have been able to identify the correct answer to this question by knowing that “chroma” refers to color.

a. Incorrect Color agnosia is the inability to name or discriminate between colors.
b. Incorrect Akinetopsia is the inability to perceive moving objects as the result of brain damage.
c. Incorrect Prosopagnosia is the inability to recognize familiar faces.
d. CORRECT Total color blindness, or complete achromatopsia, is an autosomal recessive disorder that is caused by a lack of functioning of the cone cells.

The correct answer is: achromatopsia.

173
Q

The research on facial expression of emotion has demonstrated that:
Select one:

A. facial expression is highly variable from culture to culture, implying that it is a learned behavior.
B. facial expressions among infants are similar from culture to culture, but by age 4 or 5, most expression is culturally determined.
C. there are several universal facial expressions that may be mediated in terms of intensity by cultural factors.
D. some cultures share similar facial expressions but there are few, if any, universal expressions.

A

Much of the research on emotion and the facial expression of emotion has confirmed universal elements.

a. Incorrect See explanation for response c.
b. Incorrect See explanation for response c.
c. CORRECT According to the universality hypothesis, there are a certain number of universal emotions (the number varies somewhat from expert to expert but is around seven). The facial expressions for these emotions are recognized cross-culturally, although there are differences in the intensity of facial expressions, which seem to be the result of cultural factors.
d. Incorrect See explanation for response c.

The correct answer is: there are several universal facial expressions that may be mediated in terms of intensity by cultural factors.

174
Q

Anomia is a likely outcome of damage to which of the following areas of the brain?
Select one:

A. somatosensory cortex
B. mammillary bodies
C. Wernickes area
D. Papezs circuit

A

Knowing that anomia is a symptom of some types of aphasia and/or that it involves an inability to name familiar objects and people would have helped you identify the correct answer to this question.

a. Incorrect The somatosensory cortex is located in the parietal lobe and is involved in pressure, temperature, pain, and proprioception.
b. Incorrect The mammillary bodies are connected to the hypothalamus and play a role in learning and memory.
c. CORRECT Wernicke’s area is a major language area in the brain, and damage causes deficits in language comprehension and production, including an inability to name familiar objects and people.
d. Incorrect Papez’s circuit was proposed as a brain mechanism (circuit) that mediates the experience and expression of emotion.

The correct answer is: Wernickes area

175
Q

Following a head injury, a middle-aged man experiences a loss of sensation in the fingers of his left hand. Most likely the damage involves the:
Select one:

A. postcentral gyrus.
B. precentral gyrus.
C. lateral fissure.
D. precentral sulcus.

A

Loss of sensation due to brain injury is likely to involve the somatosensory cortex.

a. CORRECT The somatosensory cortex is located on the postcentral gyrus in the parietal lobe.
b. Incorrect See explanation above.
c. Incorrect See explanation above.
d. Incorrect See explanation above.

The correct answer is: postcentral gyrus.

176
Q

In comparison to conventional (traditional) neuroleptics, risperidone (Risperdal), an atypical neuroleptic:
Select one:

A. is more likely to produce tardive dyskinesia.
B. is less likely to produce tardive dyskinesia.
C. is equally likely to produce tardive dyskinesia.
D. is less useful for alleviating tardive dyskinesia.

A

One of the advantages of many of the newer (atypical) neuroleptics is that they are associated with fewer negative side effects than the traditional neuroleptics are.

a. Incorrect See explanation for response b.
b. CORRECT Although long-term use of risperidone can produce tardive dyskinesia, the risk for this side effect is much lower for this drug than for the traditional neuroleptics.
c. Incorrect See explanation for response b.
d. Incorrect See explanation for response b.

The correct answer is: is less likely to produce tardive dyskinesia.

177
Q

Kandel and Schwartz (1982) studied habituation in Aplysia, an invertebrate marine animal, in order to obtain information on:
Select one:

A. neuronal processes underlying memory.
B. environmental factors that impact sensory memory.
C. neuronal processes underlying sexual behavior.
D. environmental factors that affect goal-directed behavior.

A

The term “habituation” refers to a decrease in response strength that occurs as the result of repeated stimulation. It is considered the simplest type of learning.

a. CORRECT Kandel and Schwartz’s research with Aplysia indicated that habituation is due to a decrease in the release of a neurotransmitter.
b. Incorrect See explanation above.
c. Incorrect See explanation above.
d. Incorrect See explanation above.

The correct answer is: neuronal processes underlying memory.

178
Q

Which of the following would NOT be useful for alleviating the psychotic symptoms associated with schizophrenia or other psychotic disorder?
Select one:

A. risperidone (Risperdal)
B. olanzapine (Zyprexa)
C. chlorpromazine (Thorazine)
D. carbamazepine (Tegretol)

A

To identify the correct response to this question, you need to know that, of the four drugs listed in the answers, only one is not an antipsychotic drug.

a. Incorrect Risperidone is an atypical antipsychotic drug.
b. Incorrect Olanzapine is also an atypical antipsychotic.
c. Incorrect Chlorpromazine is a traditional antipsychotic.
d. CORRECT Carbamazepine is an anticonvulsant drug that is also useful for treating Bipolar Disorder.

The correct answer is: carbamazepine (Tegretol)

179
Q

Which of the following individuals is at highest risk for migraine headache?
Select one:

A. a 35 year old female who is perfectionistic and ambitious
B. a 60 year old female who is sensitive and conscientious
C. a 35 year old male who is competitive and stressed
D. a 50 year old male who exercises regularly and drinks alcohol

A

Migraine headaches are more common in females and have been linked with certain personality characteristics.

a. CORRECT The risk for migraine is higher for females than males; the onset of migraines is between puberty and mid-life; and personality characteristics that have been linked to migraines include perfectionism, neatness, restraint, and ambitiousness.
b. Incorrect See explanation for response a.
c. Incorrect See explanation for response a.
d. Incorrect See explanation for response a.

The correct answer is: a 35 year old female who is perfectionistic and ambitious

180
Q

For most patients with Parkinson’s Disease, depression:
Select one:

A. is only slightly more common than it is in the general population and seems to be a reaction to the diagnosis.
B. appears to be a reaction to the diagnosis and usually first appears when motor symptoms begin to interfere with daily functioning.
C. appears to be endogenous to the disorder and may precede motor symptoms, especially in younger patients.
D. appears to be endogenous to the disorder and first appears in the later stages as cognitive impairments increase.

A

The debate over the relationship between Parkinson’s Disease and depression has not been entirely resolved. However, there seems to be more evidence that it is endogenous to the disorder rather than simply a reaction to it.

a. Incorrect Depression is more common in those with Parkinson’s Disease than in the general population and, as noted above, it is currently believed to be endogenous to the disorder.
b. Incorrect As noted, depression is probably not just a reaction to the disorder.
c. CORRECT One source of evidence for the endogenous nature of depression in Parkinson’s Disease is the occurrence of a prodromal “parkinsonian personality,” which is characterized by melancholia, introversion, and pessimism. The early appearance of depressive symptoms is most common in younger patients and those with a family history of the disorder.
d. Incorrect As noted above, depression is an early appearing symptom in some patients.

The correct answer is: appears to be endogenous to the disorder and may precede motor symptoms, especially in younger patients.

181
Q

What is the purpose of Practical Clinical Trials?
Select one:

A. To evaluate the efficacious value of interventions that are delivered in laboratory settings.
B. To evaluate the effectiveness of community interventions for potential recipients.
C. To evaluate the effects of interventions delivered under typical community conditions.
D. To evaluate the strength and valence of experimental drugs in a community setting to determine their impact.

A

The correct answer is C. Practical (or pragmatic) Clinical Trials (PCTs) are randomized studies conducted in practice settings. PCTs complement and increase the information provided by efficacy clinical trials, which evaluate interventions under ideal experimental conditions by carefully selecting suitable patients, adopting a placebo control, and utilizing intensive assessment batteries to detect statistically meaningful variations. Answers A, B and D are incorrect as the interventions are not conducted in a laboratory, community interventions are not being evaluated, and what is being evaluated is not strength and valence but rather effect.

The correct answer is: To evaluate the effects of interventions delivered under typical community conditions.

182
Q

A __________ seizure affects movement and sensation, usually on one side of the body, without a loss of consciousness.
Select one:

A. petit mal
B. atonic
C. clonic
D. simple partial

A

There are many different types of seizures. Each involves a different etiology and combination of symptoms.

a. Incorrect Petit mal (absence) seizures involve a loss of consciousness without prominent motor symptoms.
b. Incorrect Atonic seizures involve a sudden loss of muscle tone that causes the individual to fall down.
c. Incorrect Clonic seizures are characterized by jerky movements.
d. CORRECT A simple partial seizure begins on one side of the brain and, at least initially, affects only one side of the body. It does not involve a loss of consciousness.

The correct answer is: simple partial

183
Q

Which of the following describes the most likely outcome for a one-year-old child who sustains left-hemisphere brain injury that involves extensive damage to Broca’s area?
Select one:

A. The child will have substantial deficits in language production and comprehension throughout his/her life.
B. The child will have substantial deficits in language production throughout his/her life but normal language comprehension.
C. The child will eventually exhibit language abilities in the low-normal to normal range due to a takeover of language functions by the right hemisphere.
D. The child will eventually exhibit language abilities in the low-normal to normal range due to synaptogenesis in Brocas area.

A

Research on brain plasticity has shown that left hemisphere damage during early childhood does not usually produce clinically significant language dysfunction.

a. Incorrect See explanation for response c.
b. Incorrect See explanation for response c.
c. CORRECT A young child who sustains damage to the language areas in the left hemisphere may show delays in language development but, in most cases, will eventually exhibit language abilities within the low-normal to normal range due to the assumption of language functions by the right hemisphere. However, the child may also experience deficits in some right-hemisphere functions and/or a drop in IQ. See, e.g., D. Riva and L. Cazzaniga, Late effects of unilateral brain lesions sustained before and after age one, Neuropsychologia, 24, 423-428, 1986.
d. Incorrect See explanation for response c.

The correct answer is: The child will eventually exhibit language abilities in the low-normal to normal range due to a takeover of language functions by the right hemisphere.

184
Q

A tumor in the medial hypothalamus is most likely to produce which of the following?
Select one:

A. indifference and apathy
B. outbursts of aggressive behavior
C. deficits in recent long-term memory
D. receptive aphasia

A

The hypothalamus has been linked to a number of functions including maintaining the body’s homeostasis, regulating the release of hormones from the pituitary and other endocrine glands, and mediating aggressive responses.

a. Incorrect Indifference and apathy are the likely consequences of lesions in certain areas in the right hemisphere of the cerebral cortex.
b. CORRECT Lesions in certain areas of the hypothalamus (including the medial hypothalamus) have been linked to rage reactions and other forms of aggressive behavior. See, e.g., D. F. Swaab, The human hypothalamus: Basic and human aspects, Elsevier Health Sciences, Amsterdam, 2003.
c. Incorrect Memory loss may be caused by damage to the hippocampus, thalamus, or prefrontal cortex.
d. Incorrect Receptive (Wernicke’s) aphasia is caused by damage to Wernicke’s area, which is located in the dominant temporal lobe.

The correct answer is: outbursts of aggressive behavior

185
Q

Intelligence test items assessing which of the following provide a measure of crystallized intelligence?
Select one:

A. language comprehension
B. short-term memory
C. sequential reasoning
D. problem solving

A

Horn and Cattell (1966) divided general intellectual ability into two components: crystallized and fluid.

a. CORRECT As defined by Horn and Cattell, crystallized intelligence refers to acquired knowledge and skills and is affected by educational and cultural experiences. Language comprehension is a measure of crystallized intelligence.
b. Incorrect Items assessing short-term memory, sequential reasoning, and puzzle solving measure fluid intelligence, which depends on information processing skills and is less influenced than crystallized intelligence by educational and cultural experiences.
c. Incorrect See explanations for responses a and b.
d. Incorrect See explanations for responses a and b.

The correct answer is: language comprehension

186
Q

The SAT Reasoning Test is used to:
Select one:

A. help skilled and semi-skilled workers choose an occupation.
B. help high school students choose a college major.
C. predict the graduate school success of college seniors.
D. predict the college success of high school seniors.

A

Knowing that SAT refers to the Scholastic Assessment Test may have helped you identify the correct answer to this question.

a. Incorrect See explanation for response d.
b. Incorrect See explanation for response d.
c. Incorrect See explanation for response d.
d. CORRECT The Scholastic Assessment Test is an admissions test that is used to predict the college success of high school seniors. It consists of two parts – the SAT Reasoning Test and the SAT Subject Tests.

The correct answer is: predict the college success of high school seniors.

187
Q

The best conclusion that can be drawn from research on the effects of examiner race on the cognitive test performance of African American children is that:
Select one:

A. White examiners have significant adverse effects on the test performance of these children.
B. White examiners often have positive effects on the test performance of these children.
C. White examiners may have a positive, a negative, or no effect on the test performance of these children.
D. White examiners have a negative effect on the test performance of these children only when the examiners are unable to adopt a “colorblind” perspective.

A

his is a controversial issues in the literature, and the results of the research have not been consistent.

a. Incorrect See explanation for response c.
b. Incorrect See explanation for response c.
c. CORRECT This is probably the best conclusion that can be drawn on the basis of the inconsistent research results. The inconsistency has led some experts to conclude that other factors (e.g., rapport and attitudes) are more important, although “colorblindness” has not been identified as one of these factors.
d. Incorrect See explanation for response c.

The correct answer is: White examiners may have a positive, a negative, or no effect on the test performance of these children.

188
Q

With regard to test bias, a slope bias occurs when:
Select one:

A. a predictor’s validity coefficient differs for different groups.
B. a predictor’s reliability coefficient differs for different groups.
C. members of one group consistently score higher than members of another group on the predictor.
D. members of one group consistently score higher than members of another group on the criterion.

A

For the exam, you want to be familiar with slope and intercept bias, which are described in the Psychological Assessment chapter of the written study materials.

a. CORRECT A slope bias occurs when there is differential validity – i.e., when the validity coefficients for a predictor differ for different groups.
b. Incorrect See explanation for response a.
c. Incorrect See explanation for response a.
d. Incorrect See explanation for response a.

The correct answer is: a predictor’s validity coefficient differs for different groups.

189
Q

Research investigating the usefulness of the serial position effect for detecting malingering provides some evidence that, in comparison to patients with traumatic brain injury, volunteers instructed to malinger (i.e., to feign brain injury) are:
Select one:

A. less likely to exhibit a primacy effect.
B. less likely to exhibit a recency effect.
C. less likely to exhibit a primacy and a recency effect.
D. more likely to exhibit a primacy and a recency effect.

A

This is a difficult question because relatively few studies address the topic it is asking about (the use of the serial position effect to detect malingering), and those studies have not provided entirely consistent results. While several studies have found that suppression of primacy effects is characteristic of malingerers but not brain-injured patients, a few other studies suggest that malingerers and brain-injured patients do not consistently differ with regard to primacy (or recency) effects.

a. CORRECT Some studies have found that malingerers are less likely than patients with TBI to exhibit a primacy effect (i.e., are less likely to recall words from the beginning of the word list) but are equally likely to exhibit a recency effect (i.e., are equally likely to recall words from the end of the list). See, e.g., J. A. Suhr, Malingering, coaching, and the serial position effect, Archives of Clinical Neuropsychology, 17(1), 69-77, 2002.
b. Incorrect See explanation for response a.
c. Incorrect See explanation for response a.
d. Incorrect See explanation for response a.
The correct answer is: less likely to exhibit a primacy effect.

190
Q

A person is most likely to be happy in a job that involves working with tools and objects, requires physical strength, and provides regular feedback in the form of practical results if his/her highest score is on the __________ subscale of Holland’s Self-Directed Search.
Select one:

A. investigative
B. mechanical
C. conventional
D. realistic

A

Holland distinguished between six personality/job environment themes.

a. Incorrect A person who scores high on the investigative theme has scientific, mathematical, analytic, or scholarly interests or skills.
b. Incorrect Mechanical is not one of Holland’s themes.
c. Incorrect A person who scores high on the conventional theme has interests or skills in keeping records, organizing data, attending to detail, or following through on others’ instructions.
d. CORRECT A person who scores high on the realistic theme enjoys or does well in technical, physical, mechanical, and/or outdoor activities.

The correct answer is: realistic

191
Q

In the context of the Rorschach test, form quality indicates:
Select one:

A. what aspect of the inkblot determined the response.
B. the degree of congruence between the response and reality.
C. the extent to which the response is based on an unusual or common detail.
D. the extent to which form is well integrated with other determinants.

A

As its name suggests, form quality refers to the form (shape) of the inkblot.

a. Incorrect See explanation for response b.
b. CORRECT Form quality refers to the degree to which an examinee’s response matches the actual form of the inkblot and provides information on the degree of congruence between the examinee’s response and reality.
c. Incorrect See explanation for response b.
d. Incorrect See explanation for response b.

The correct answer is: the degree of congruence between the response and reality.

192
Q

The Wonderlic Personnel Test-Revised (WPT-R) is a(n):
Select one:

A. brief test of mental ability for adults.
B. measure of global and facet job satisfaction.
C. test of English-language proficiency.
D. apparatus test of psychomotor skills.

A

Many of the licensing exam questions on specific tests will (like this one) require you to be familiar with the test’s purpose.

a. CORRECT In addition to knowing that the Wonderlic is a 12-minute test of mental ability for adults, you want to know that it is used to assist employers with hiring decisions.
b. Incorrect See explanation for response a.
c. Incorrect See explanation for response a.
d. Incorrect See explanation for response a.

The correct answer is: brief test of mental ability for adults.

193
Q

The Cattell-Horn-Carroll theory of cognitive abilities combines the Cattell-Horn theory of fluid intelligence and Carroll’s three stratum theory of intelligence and distinguishes between ___ broad abilities and over ___ narrow abilities.
Select one:

A. 5; 25
B. 8; 40
C. 10; 70
D. 15; 90

A

The Cattell-Horn-Carroll (CHC) theory has influenced the development a several intelligence tests including the fifth edition of the Stanford-Binet.

a. Incorrect See explanation for answer c.
b. Incorrect See explanation for answer c.
c. CORRECT The CHC theory distinguishes between 10 broad cognitive abilities and over 70 narrow cognitive abilities, with each broad ability being composed of several related but distinct narrow abilities. The ten broad abilities are fluid intelligence, crystallized intelligence, quantitative knowledge, reading/writing ability, short-term memory, visual processing, auditory processing, long-term storage and retrieval, processing speed, and correct decision speed/reaction time.
d. Incorrect See explanation for answer c.

The correct answer is: 10; 70

194
Q

A social psychologist tells his male and female subjects the following story about Jack and Jane: On Saturday, Jack fixes his car’s transmission and Jane sews. Both complete their tasks successfully. On Sunday, Jack decorates his apartment and Jane cuts down a tree in her front yard. Again, both complete their tasks successfully. Which of the following statements best describes the likely results of this research?
Select one:

A. Males and females will attribute Jack’s success in fixing the transmission and Jane’s success in sewing to ability, but will attribute Jack’s success in decorating and Jane’s success in cutting down a tree to luck.
B. Males will attribute Jack’s success on both tasks and Jane’s success in sewing to ability, but females will attribute Jack’s success in fixing the transmission and Jane’s success in sewing to ability and Jack’s success in decorating and Jane’s success in cutting down a tree to luck.
C. Males and females will both attribute Jack’s success in both tasks to ability and Jane’s success in both tasks to luck.
D. Males and females will both attribute Jack’s success in both tasks and Jane’s success in sewing to ability, but will attribute Jane’s success in cutting down a tree to luck.

A

Deaux and Emswiller (1974) found that attributions related to achievement vary, depending on the sex of the achiever. Specifically, they found that (1) both males and females viewed ability as the cause of success for males on traditionally masculine tasks, (2) both males and females viewed luck as the cause of success for females on traditionally masculine tasks, (3) both males and females viewed ability as the cause of success for males and females on traditionally feminine tasks.
a. Incorrect See explanation for response d.

b. Incorrect See explanation for response d.
c. Incorrect See explanation for response d.
d. CORRECT This response is in line with the research mentioned above. Jack’s success in both fixing a transmission (a traditionally masculine task) and decorating (a traditionally feminine task) will be attributed to ability; Jane’s success in sewing (a traditionally feminine task) will be attributed to ability, but her success in cutting down a tree (a traditionally masculine task) will be attributed to luck.

The correct answer is: Males and females will both attribute Jack’s success in both tasks and Jane’s success in sewing to ability, but will attribute Jane’s success in cutting down a tree to luck.

195
Q

A listener is LEAST likely to change her attitude as the result of a communicator’s message if:
Select one:

A. the listener accidentally overhears the message.
B. the listener has prior knowledge about the content of the message.
C. the communicator is arguing against his or her own best interests.
D. there is a moderate discrepancy between the listener’s initial position and the communicator’s position.

A

Much research has been conducted on communicator, audience, and message characteristics that influence persuadibility. Though you should be familiar with such research, you can also sometimes use logic or your own experience to derive answers to questions such as this one.

a. Incorrect “Accidental” messages are associated with greater attitude change than intended messages.
b. CORRECT Research suggests that listeners are better able to resist a persuasive message if they have been forewarned about its content.
c. Incorrect A communicator arguing against his/her own best interest is more likely to change the attitudes of listeners than a communicator arguing to advance his/her own interests.
d. Incorrect A moderate discrepancy in position between the listener and communicator is associated with greater attitude change than either a high or low discrepancy.

The correct answer is: the listener has prior knowledge about the content of the message.

196
Q

Byrne’s (1971) law of attraction proposes that people prefer spending time with others who are similar to them in terms of attitudes, and he links this preference to which of the following?
Select one:

A. reinforcement
B. cognitive schemas
C. social comparison theory
D. self-monitoring

A

The correct answer is A. Byrne’s law of attraction proposes that there is positive linear relationship between level of attraction and similarity in attitudes (The attraction paradigm, New York, Academic Press, 1971). Byrne proposes that people are reinforced for interacting with others who have similar attitudes because doing so confirms their effectiveness in interpreting the world and produces positive affect.

The correct answer is: reinforcement

197
Q

People using the ____________ heuristic base their judgments on resemblances and similarities.
Select one:

A. availability
B. representativeness
C. simulation
D. anchoring

A

Kahneman and Tversky (1984) identified the four heuristics (mental shortcuts) listed in the answers.

a. Incorrect The availability heuristic involves judging the likelihood or frequency of an event based on how easy it is to retrieve information relevant to the event from long-term memory.
b. CORRECT The representativeness heuristic involves judging the likelihood or frequency of an event based on the extent to which it resembles the “typical” case. (Even if you aren’t familiar with the representativeness heuristic, you may have been able to identify the correct answer to this question since things that resemble or are similar to members of a category can be viewed as being representative of that category.)
c. Incorrect The simulation heuristic involves judging the likelihood or frequency of an event based on how easy it is to mentally simulate (imagine) the event.
d. Incorrect The anchoring and adjustment heuristic involves judging the likelihood or frequency of an event by using an anchoring (starting) point and making adjustments up or down from that point.

The correct answer is: representativeness

198
Q

Research on bystander apathy (Latane and Darley, 1968) found that a victim is most likely to obtain help from a bystander when there is:
Select one:

A. only one bystander.
B. between three and six bystanders.
C. between seven and 10 bystanders.
D. more than 12 bystanders.

A

A number of factors influence whether bystanders will assist the apparent victim of an accident, crime, or other emergency situation including the number of bystanders.

a. CORRECT The research has found that bystanders are less likely to offer help when in the presence of others than when alone and that the greater the number of bystanders, the less likely anyone will offer assistance.
b. Incorrect See explanation for response a.
c. Incorrect See explanation for response a.
d. Incorrect See explanation for response a.

The correct answer is: only one bystander.

199
Q

According to the elaboration likelihood model, the recipient of a persuasive message is more likely to use the central route of information processing when:
Select one:

A. the recipient of the message has an external locus of control.
B. the person delivering the message is a well-liked celebrity.
C. the message has some relevance to the recipient of the message.
D. the message is weak.

A

The elaboration likelihood model (Petty and Cacioppo, 1980) distinguishes between central and peripheral processing routes.

a. Incorrect Locus of control is not part of the elaboration likelihood model.
b. Incorrect This situation is more likely to lead to peripheral processing of a persuasive message.
c. CORRECT A person is more likely to use the central route when he/she is highly motivated. Relevance of the message affects motivation.
d. Incorrect A weak message is more likely to result in peripheral processing of the message.

The correct answer is: the message has some relevance to the recipient of the message.

200
Q

Hewig et al. (2008) compared the gaze patterns of heterosexual men and women as they looked at pictures of members of the opposite sex and found that:
Select one:

A. females and males both gazed at the face of a member of the opposite sex for the longest period of time.
B. females gazed at the face of a male for the longest period of time, while males gazed at the breasts of a female for the longest period of time.
C. females gazed at the hip/buttocks region of a male for the longest period of time, while males gazed at the face of a female for the longest period of time.
D. females and males both gazed at the hip/buttocks region of a member of the opposite sex for the longest period of time.

A

This question is an example of the two or three “distant galaxy” questions that you’re likely to encounter on the licensing exam.

a. CORRECT J. Hewig et al. found males and females both gazed at the face of a member of the opposite sex for the longest period of time. However, after an initial face-scan, men looked earlier and longest at the female’s breasts, while women looked earlier at the male’s legs. These investigators note that their findings are consistent with previous research using other methodologies (Gender differences for specific body regions when looking at men and women, Journal of Nonverbal Behavior, 32(2), 67-78, 2008).
b. Incorrect See explanation for response a.
c. Incorrect See explanation for response a.
d. Incorrect See explanation for response a.

The correct answer is: females and males both gazed at the face of a member of the opposite sex for the longest period of time.

201
Q

A number of studies have been conducted in the last decade to examine the interaction between cognition and affect. One consistent finding of this research is that:
Select one:

A. people in a positive mood perceive, encode, and retrieve positive, neutral, and negative information with equal efficiency and more efficiently than people in a negative mood.
B. people in a positive mood perceive, encode, and retrieve neutral information most efficiently, while people in a negative mood perceive, encode, and retrieve positive and negative information most efficiently.
C. people in a positive mood perceive, encode, and retrieve positive information more efficiently, while people in a negative mood perceive, encode, and retrieve negative information more efficiently.
D. there is no consistent relationship between mood and the perception, encoding, and retrieval of positive or negative information.

A

Most studies support the principle of “mood congruency.”

a. Incorrect See explanation for response c.
b. Incorrect See explanation for response c.
c. CORRECT Except in special circumstances (e.g., when current mood is attributed to an experimental manipulation), there is a consistent relationship between affect and cognition. For example, depressed people are more likely to recall pessimistic or negative events and make pessimistic social judgments.
d. Incorrect See explanation for response c.

The correct answer is: people in a positive mood perceive, encode, and retrieve positive information more efficiently, while people in a negative mood perceive, encode, and retrieve negative information more efficiently.

202
Q

You are a community psychologist working with two rival gangs. Which of the following represents the intervention that is most likely to decrease the hostility between members of the groups?
Select one:

A. talk to them about the different theories of aggression so that they understand the processes behind their actions
B. have the gangs organize a protest against police brutality in their neighborhood
C. give each group false information about the other group
D. give each group no information about the other group

A

This question is related to research about the reduction of conflict.

a. Incorrect Involved explanations of psychological processes have not been shown to be effective in reducing conflict; indeed, they are rarely, if ever, effective in any therapeutic setting.
b. CORRECT Conflict can sometimes be defused by de-emphasizing the current conflict and focusing on superordinate, common goals shared by the parties.
c. Incorrect Sometimes, persuasive information is effective in reducing conflict. However, this information should be factual, not false. In addition, the receiving party must be willing to consider the information; such willingness is unlikely with gang members.
d. Incorrect See explanation for response c.

The correct answer is: have the gangs organize a protest against police brutality in their neighborhood

203
Q

Studies looking at the impact of gender on reactions to crowding have found that:
Select one:

A. women cope with crowding better than men do in both laboratory and residential settings.
B. men cope with crowding better than women do in both laboratory and residential settings.
C. women cope with crowding better than men do in laboratory settings but men cope with crowding better in residential settings.
D. men cope with crowding better than women do in laboratory settings but women cope with crowding better in residential settings.

A

Studies on the relationship between gender and the effects of crowding have produced somewhat inconsistent results.

a. Incorrect See explanation for response c.
b. Incorrect See explanation for response c.
c. CORRECT A review of the research indicates that the inconsistencies are due largely to the setting in which the research was conducted. Women tend to respond better than men in crowded laboratory (experimental) situations, while men do better in naturalistic settings.
d. Incorrect See explanation for response c.

The correct answer is: women cope with crowding better than men do in laboratory settings but men cope with crowding better in residential settings.

204
Q

To use scores obtained on the eight subscales of the Parental Activities Scale to predict scores on the five subscales of the Child Adjustment Profile, you would use:
Select one:

A. multiple regression.
B. canonical correlation.
C. multiple discriminant analysis.
D. cluster analysis

A

In this situation, multiple X scores will be used to predict multiple Y scores.

a. Incorrect Multiple regression is used when scores on two or more X variables will be used to predict a single Y score.
b. CORRECT Canonical correlation is an extension of multiple regression that is appropriate when there is more than one Y variable.
c. Incorrect Multiple discriminant analysis is used when the Y variable is nominal and two or more X variables will be used to place the examinee into a category on Y.
d. Incorrect Cluster analysis is used to identify or confirm homogeneous subgroups.

The correct answer is: canonical correlation.

205
Q

__________ is used to determine the minimum sample size needed for a study, given a particular level of significance, expected effect size, and other factors.
Select one:

A. Power analysis
B. Meta-analysis
C. Incremental analysis
D. Sensitivity analysis

A

Knowing that statistical power is affected by sample size would have helped you identify the correct answer to this question.

a. CORRECT A common use of power analysis is to determine the minimum sample size needed, given the desired level of power, type of statistical test, level of significance, and expected effect size.
b. Incorrect See explanation for response a.
c. Incorrect See explanation for response a.
d. Incorrect See explanation for response a.

The correct answer is: Power analysis

206
Q

A moderator variable:
Select one:

A. is responsible for (causes) the relationship between independent and dependent variables.
B. affects the direction or strength of the relationship between independent and dependent variables.
C. is applied sequentially to participants during the course of the research study.
D. is the outcome variable that is predicted in a correlational research study.

A

For the exam, you want to be familiar with the variables described in the answers to this question.

a. Incorrect This describes a mediator variable.
b. CORRECT A moderator variable affects the direction or strength of the relationship between independent and dependent variables. For example, if the results of a research study indicate that EMG biofeedback is more effective for tension headaches than for migraine or cluster headaches, type of headache is a moderator variable - i.e., type of headache moderates the effects of EMG feedback on headache symptoms.
c. Incorrect This does not describe a moderator variable.
d. Incorrect In correlational research, the dependent (outcome) variable is often referred to as the criterion variable.

The correct answer is: affects the direction or strength of the relationship between independent and dependent variables.

207
Q

Multiple regression analysis has several advantages over the analysis of variance including all of the following except:
Select one:

A. it eliminates the need for a two-stage analysis involving a global significance test that is followed, when appropriate, by “fine grain” significance tests.
B. it is not limited to categorical (or categorized continuous) variables.
C. it enables the researcher to determine if entering additional independent variables affects the dependent variable beyond the effects found for previously entered variables.
D. it allows the researcher to estimate the effects of the independent variables on the dependent variable with the effects of measurement error removed.

A

At least one reason for R. A. Fisher’s development of the ANOVA was to have a mathematically simpler alternative to multiple regression for analyzing the effects of qualitative variables. However, with the availability of computers, the need for the ANOVA has been questioned.

a. Incorrect This is an advantage of multiple regression. All of the information is provided with a single analysis as opposed to the two-step process required when using the ANOVA.
b. Incorrect This is also an advantage of multiple regression. It can be used for independent variables measured on any scale of measurement and doesn’t require that continuous variables be categorized (which has the effect of reducing the usefulness of the data).
c. Incorrect The “stepwise” method of multiple regression does have this advantage.
d. CORRECT This is not an advantage of either multiple regression or the ANOVA.

The correct answer is: it allows the researcher to estimate the effects of the independent variables on the dependent variable with the effects of measurement error removed.

208
Q

In a __________ distribution of scores, the mean is less than the median and the median is less than the mode.
Select one:

A. leptokurtic
B. platykurtic
C. negatively skewed
D. positively skewed

A

In a normal distribution, the mean, median, and mode are equal to the same value. In contrast, in a skewed distribution, the distribution is asymmetrical and the mean, median and mode have different values due to the presence of a few outliers in the positive or negative end of the distribution.

a. Incorrect The term leptokurtic is used to describe a distribution of scores that is more “peaked” than the normal distribution.
b. Incorrect The term platykurtic is used to describe a distribution of scores that is flatter than the normal distribution.
c. CORRECT In a negatively skewed distribution, the mean is affected most by the few outliers in the negative (low) end of the distribution and, therefore, has the lowest value.
d. Incorrect In a positively skewed distribution, the mean is larger than the median which is larger than the mode.

The correct answer is: negatively skewed

209
Q

The probability of making a Type I error is equal to:
Select one:

A. alpha.
B. 1 minus alpha.
C. beta.
D. 1 minus beta.

A

Alpha is also known as the level of significance, and its value is set by the researcher prior to analyzing the data collected in a research study.

a. CORRECT Alpha determines the probability of making a Type I error, which occurs when a true null hypothesis is rejected.
b. Incorrect One minus alpha is the probability of retaining a true null hypothesis.
c. Incorrect Beta is hypothetical (i.e., is not set by the researcher) and refers to the probability of making a Type II error, which occurs when a false null hypothesis is retained.
d. Incorrect One minus beta is also hypothetical. It refers to the probability of rejecting a false null hypothesis and is also referred to as power.

The correct answer is: alpha.

210
Q

“Sampling error” is due to:
Select one:

A. the unreliability of the test.
B. the invalidity of the test.
C. random factors that produce a nonrepresentative sample.
D. non-random factors that produce a nonrepresentative sample.

A

Whenever a sample is drawn from a population, there is a good chance that the sample will not be entirely representative of that population.

a. Incorrect See explanation for response c.
b. Incorrect See explanation for response c.
c. CORRECT As the result of random (chance) factors, the sample may not “mirror” the population in terms of important characteristics. This is referred to as sampling error.
d. Incorrect See explanation for response c.

The correct answer is: random factors that produce a nonrepresentative sample.

211
Q

A researcher uses a factorial ANOVA to statistically analyze the effects of four types of training strategies and three levels of self-efficacy on a measure of job performance. The results indicate that there are significant main effects of each independent variable and a significant interaction. The researcher will conclude that:
Select one:

A. training is effective regardless of level of self-efficacy.
B. each type of training is equally effective for each level of self-efficacy.
C. the most effective type of training depends on level of self-efficacy.
D. overall, training is effective only for people with a certain level of self-efficacy.

A

To answer this question correctly, you need to know that an interaction occurs when the effects of one independent variable differ for different levels of another independent variable.

a. Incorrect See explanation for response c.
b. Incorrect See explanation for response c.
c. CORRECT Because the interaction is significant, this means that the effects of training type may differ for different levels of self-efficacy –e.g., training method #1 might be most effective for people with low self-efficacy, while training method #2 might be most effective for people with high self-efficacy.
d. Incorrect See explanation for response c.

The correct answer is: the most effective type of training depends on level of self-efficacy.

212
Q

To determine the degree of association between pregnancy status (pregnant or not) and the number of items selected on a stress event checklist, you would use which of the following?
Select one:

A. Pearson r
B. Spearman rho
C. biserial
D. point biserial

A

In this situation, you are correlating a variable representing a true dichotomy (pregnant versus not pregnant) with a variable representing a ratio (continuous) scale of measurement (number of stress events).

a. Incorrect The Pearson r is appropriate when both variables are measured on a continuous scale.
b. Incorrect Spearman rho is appropriate when both variables are expressed in terms of ranks.
c. Incorrect The biserial correlation coefficient is used when one variable is an artificial dichotomy (a continuous variable that has been artificially or arbitrarily dichotomized) and the other variable is continuous.
d. CORRECT The point biserial correlation coefficient is appropriate when one variable is a true dichotomy and the other is continuous.

The correct answer is: point biserial

213
Q

Your research study involves assessing the effects of two independent variables on three dependent variables. In this situation, you would choose to conduct a MANOVA to analyze the data you collect rather than separate factorial ANOVAs in order to:
Select one:

A. maximize experimental variance.
B. control an extraneous variable.
C. increase statistical power.
D. control the experimentwise error rate.

A

The multivariate analysis of variance (MANOVA) is used to simultaneously assess the effects of one or more independent variables on two or more dependent variables.

a. Incorrect See explanation for response d.
b. Incorrect See explanation for response d.
c. Incorrect See explanation for response d.
d. CORRECT The more statistical comparisons made within a research study, the greater the likelihood of making a Type I error. By using the MANOVA to simultaneously assess the effects of the independent variable(s) on the dependent variables, the fewer the total number of statistical comparisons, and the lower the probability of making a Type I error. (The word “error” in the term experimentwise error rate refers to a Type I error.)

The correct answer is: control the experimentwise error rate.

214
Q

A researcher would use the Solomon four-group design to:
Select one:

A. reduce practice effects.
B. eliminate demand characteristics.
C. evaluate the effects of pretesting.
D. determine if there are any selection biases.

A

The Solomon four-group design combines the pretest-posttest control group design with the posttest only control group design.

a Incorrect See explanation for response c.

b. Incorrect See explanation for response c.
c. CORRECT The purpose of Solomon four-group design is to evaluate the effects of pretesting on a study’s internal and external validity.
d. Incorrect See explanation for response c.

The correct answer is: evaluate the effects of pretesting.

215
Q

In a normal distribution, approximately ____% of scores fall between the scores that are plus and minus two standard deviations from the mean.
Select one:

A. 50
B. 68
C. 95
D. 99

A

For the exam, you want to be familiar with the areas under the normal curve so that you can answer questions like this one. See the Statistics and Research Design chapter for a description of areas under the normal curve.

a. Incorrect See explanation for response c.
b. Incorrect About 68% of scores fall between the scores that are plus and minus one standard deviation from the mean.
c. CORRECT About 95% of scores fall between the scores that are plus and minus two standard deviations from the mean.
d. Incorrect About 99% of scores fall between the scores that are plus and minus three standard deviations from the mean.

The correct answer is: 95

216
Q

A predictor’s ___________ is calculated by dividing the number of true positives by the number of true positives plus false negatives.
Select one:

A. positive predictive value
B. negative predictive value
C. sensitivity
D. specificity

A

The accuracy of a predictor can be described in terms of its sensitivity, specificity, positive predictive value, and negative predictive value.

a. Incorrect A predictor’s positive predictive value indicates the probability that an individual identified as a positive is a true positive. It is calculated by dividing the number of true positives by the number of true and false positives.
b. Incorrect A predictor’s negative predictive value indicates the probability that an individual identified as a negative is a true negative. It is calculated by dividing the number of true negatives by the number of true and false negatives.
c. CORRECT A predictor’s sensitivity refers to the proportion of individuals in the validation sample who have the characteristic measured by the predictor and were accurately identified by the predictor as having that characteristic. It provides an index of the predictor’s ability to identify true positives. Sensitivity is calculated by dividing the number of true positives by the number of true positives plus false negatives.
d. Incorrect A predictor’s specificity refers to the proportion of individuals in the validation sample without the characteristic measured by the predictor who were correctly identified by the predictor as not having that characteristic. It provides an index of the predictor’s ability to identify true negatives. Specificity is calculated by dividing the number of true negatives by the number of true negatives plus false positives.

The correct answer is: sensitivity

217
Q

In a normal distribution of scores that has a mean of 120 and a standard deviation of 12, a raw score of 138 is equivalent to a z score of:
Select one:

A. 0.75
B. 1
C. 1.5
D. 2.25

A

To identify the correct answer to this question, you need to know that z scores indicate how far a raw score is below or above the mean.

a. Incorrect See explanation for response c.
b. Incorrect See explanation for response c.
c. CORRECT In this distribution, the mean is 120 and the standard deviation is 12. Therefore, a raw score of 138 is 1.5 standard deviations above the mean and is equivalent to a z score of 1.5.
d. Incorrect See explanation for response c.

The correct answer is: 1.5

218
Q

To assess the reliability of a characteristic that fluctuates in severity or intensity over time, you would be best advised to use which of the following?
Select one:

A. coefficient of equivalence
B. coefficient of stability
C. coefficient of determination
D. coefficient of internal consistency

A

If the characteristic measured by a test fluctuates over time (i.e., is a “state”), it would not be appropriate to assess the test’s reliability using a method that requires administering the test (or alternative forms of the test) at different times.

a. Incorrect A coefficient of equivalence is obtained when equivalent forms reliability is used. In most cases, it is necessary to administer the different forms at different times; therefore, this method would not be the best for a test that measures a characteristic that fluctuates over time.
b. Incorrect The coefficient of stability is obtained when using test-retest reliability, which would also be inappropriate for a measure of a characteristic that fluctuates over time.
c. Incorrect The coefficient of determination is not a measure of reliability.
d. CORRECT A coefficient of internal consistency would be appropriate for a test that measures a characteristic that fluctuates over time since it requires administering the test only once.

The correct answer is: coefficient of internal consistency

219
Q

A factor matrix indicates that Test A has a factor loading of .40 on Factor I and a factor loading of .30 on Factor II. Assuming the factors are orthogonal, what is the communality for Test A?
Select one:

A. 0.1
B. 0.25
C. 0.45
D. 0.7

A

As described in the Test Construction chapter of the written study materials, when factors are orthogonal (uncorrelated), a test’s factor loadings can be squared and summed to calculate the communality (the amount of variability in test scores explained by the identified factors).

a. Incorrect See explanation for response b.
b. CORRECT Test A’s communality is equal to .40 squared (.16) plus .30 squared (.09), which is .25.
c. Incorrect See explanation for response b.
d. Incorrect See explanation for response b.

The correct answer is: 0.25

220
Q

A predictor’s criterion-related validity coefficient is .70. This means that ____% of variability in criterion scores is explained by variability in predictor scores.
Select one:

A. 30
B. 49
C. 51
D. 70

A

The criterion-related validity coefficient is interpreted like any other correlation coefficient for two variables – i.e., it is squared to obtain a measure of shared variability.

a. Incorrect See explanation for response b.
b. CORRECT A validity coefficient of .70 indicates that 49% (.70 squared) of variability is shared by the predictor and criterion – or, put another way, that 49% of variability in criterion scores is explained by variability in predictor scores.
c. Incorrect See explanation for response b.
d. Incorrect See explanation for response b.

The correct answer is: 49

221
Q

The coefficient of stability is useful for:
Select one:

A. assessing the reliability of a test that is administered on two different occasions to the same group of examinees.
B. assessing the reliability of two versions of a test that are administered to the same group of examinees.
C. evaluating the validity of a test across administrations of the test at two different times.
D. evaluating the validity of a test across different groups of examinees.

A

To answer this question, you need to know that the “coefficient of stability” is another name for the test-retest reliability coefficient.

a. CORRECT The coefficient of stability indicates the degree of consistency (reliability) of a test across time.
b. Incorrect See explanation above.
c. Incorrect See explanation above.
d. Incorrect See explanation above.

The correct answer is: assessing the reliability of a test that is administered on two different occasions to the same group of examinees.

222
Q

A test developer uses factor analysis to evaluate the construct validity of his newly developed test of self-efficacy. The results indicate that the measure’s communality is .40. This means that:
Select one:

A. 16% of variability in test scores is accounted for by each identified factor.
B. 40% of variability in test scores is accounted for by each identified factor.
C. 16% of variability in test scores is accounted for by all of the identified factors.
D. 40% of variability in test scores is accounted for by all of the identified factors.

A

In factor analysis, a test’s communality indicates its “common variance,” or the amount of variability in test scores that is explained by the factors that the test shares in common with the other tests included in the analysis.

a. Incorrect See explanation for response d.
b. Incorrect See explanation for response d.
c. Incorrect See explanation for response d.
d. CORRECT A test’s communality is interpreted directly as the amount of variability in test scores explained by the identified factors. Therefore, when the communality is .40, this means that 40% of variability in test scores is explained by the identified factors.

The correct answer is: 40% of variability in test scores is accounted for by all of the identified factors.

223
Q

A distribution of test scores is normally shaped and has a mean of 100 and standard error of measurement of 5. For an examinee who obtains a score of 103 on the test, the 68% confidence interval is:
Select one:

A. 95 to 105.
B. 98 to 108.
C. 93 to 113.
D. 90 to 115.

A

To calculate the 68% confidence interval, one standard error is added to and subtracted from the examinee’s test score.

a. Incorrect See explanation for response b.
b. CORRECT Adding and subtracting 5 (the standard error) to and from the examinee’s score of 103 produces a confidence interval that ranges from 98 to 108.
c. Incorrect See explanation for response b.
d. Incorrect See explanation for response b.

The correct answer is: 98 to 108.

224
Q

The Spearman-Brown formula is used to:
Select one:

A. estimate the effects of increasing or decreasing the length of a test on its reliability coefficient.
B. estimate what a predictors validity coefficient would be if the predictor and/or criterion were perfectly reliable.
C. determine the range within which an examinees true score is likely to fall given his/her obtained score.
D. determine if adding or subtracting a predictor to the multiple regression equation will have a significant effect on its predictive accuracy.

A

For the exam, you want to have the Spearman-Brown formula linked to reliability.

a. CORRECT The Spearman-Brown formula is also known as the Spearman-Brown prophecy formula and is used to estimate the effects of adding or subtracting items to a test on its reliability coefficient. For example, if a 50-item test has a reliability coefficient of .80, the Spearman-Brown formula could be used to estimate what the test’s reliability coefficient would be if the number of items was doubled.
b. Incorrect This describes the correction for attenuation formula.
c. Incorrect The standard error of measurement is used to construct a confidence interval around an examinee’s obtained test score. The confidence interval indicates the range within which the examinees true score is likely to fall.
d. Incorrect The coefficient of multiple determination provides this information.

The correct answer is: estimate the effects of increasing or decreasing the length of a test on its reliability coefficient.

225
Q

To ensure that a work sample has adequate content validity, you would:
Select one:

A. make sure that the skills required by the work sample represent the skill domain required by the job.
B. make sure that performance on the work sample accurately distinguishes between successful and unsuccessful workers.
C. determine if performance on the work sample corresponds to current on-the-job performance.
D. determine if performance on the work sample accurately predicts on-the-job performance in the future.

A

A measure has content validity when its items represent the content (or behavior) domain the test was designed to measure.

a. CORRECT This is an accurate description of content validity.
b. Incorrect This technique would be used to establish construct or criterion-related validity.
c. Incorrect This sounds like concurrent validity (a type of criterion-related validity).
d. Incorrect This sounds like predictive validity (another type of criterion-related validity).

The correct answer is: make sure that the skills required by the work sample represent the skill domain required by the job.